Migraine relief in 20 minutes using eyedrops?

Article Type
Changed
Wed, 06/15/2022 - 10:38
Display Headline
Migraine relief in 20 minutes using eyedrops?

ILLUSTRATIVE CASE

A 35-year-old woman with no significant past medical history presents for follow-up of migraine. At the previous visit, she was prescribed sumatriptan for abortive therapy. However, she has been having significant adverse effect intolerance from the oral formulation, and the nasal formulation is cost prohibitive. What can you recommend as an alternative abortive therapy for this patient’s migraine?

Migraine is among the most common causes of disability worldwide, affecting more than 10% of the global population.2 The prevalence of migraine is between 2.6% and 21.7% across multiple countries.3 On a scale of 0% to 100%, disability caused by migraine is 43.3%, comparable to the first 2 days after an acute myocardial infarction (42.2%) and severe dementia (43.8%).4

Abortive therapy for acute migraine includes nonsteroidal anti-inflammatory drugs (NSAIDs), triptans, ergots, and antiemetics. However, these options are predominantly administered by mouth; non-oral formulations tend to be cost prohibitive and difficult to obtain.

Nausea and vomiting, common components of migraine (that are included in International Classification of Headache Disorders, 3rd edition [ICHD-3] criteria for migraine5) present obstacles to effective oral administration if experienced by the patient. In addition, for migraine refractory to first-line treatments, abortive options—including the recently approved calcitonin gene-related peptide (CGRP) receptor antagonists ubrogepant and rimegepant—are also cost prohibitive, potentially costing more than $1000 for 10 tablets (100 mg), depending on insurance coverage.6

Two oral beta-blockers, propranolol and timolol, are approved by the US Food and Drug Administration for migraine prophylaxis. Unfortunately, oral beta-blockers are ineffective for abortive treatment.7 Ophthalmic timolol is typically used in the treatment of glaucoma, but there have been case reports describing its benefits in acute migraine treatment.8,9 In addition, ophthalmic timolol is far cheaper than medications such as ubrogepant.10 A 2014 case series of 7 patients discussed ophthalmic beta-blockers as an effective and possibly cheaper option for acute migraine treatment.8 A randomized, crossover, placebo-controlled pilot study of 198 migraine attacks in 10 participants using timolol eyedrops for abortive therapy found timolol was not significantly more effective than placebo.9 However, it was an underpowered pilot study, with a lack of masking and an imperfect placebo. The trial discussed here was a controlled, prospective study investigating topical beta-blockers for acute migraine treatment.

STUDY SUMMARY

Crossover study achieved primary endpoint in pain reduction

This randomized, single-center, double-masked, crossover trial compared timolol maleate ophthalmic solution 0.5% with placebo among 43 patients ages 12 or older presenting with a diagnosis of migraine based on ICHD-3 (beta) criteria. Patients were eligible if they had not taken any antimigraine medications for at least 1 month prior to the study and were excluded if they had taken systemic beta-blockers at baseline, or had asthma, bradyarrhythmias, or cardiac dysfunction.

Patients were randomized 1:1 to treatment with timolol maleate 0.5% eyedrops or placebo. At the earliest onset of migraine, patients used 1 drop of timolol maleate 0.5% or placebo in each eye; if they experienced no relief after 10 minutes, they used a second drop or matching placebo. Patients were instructed to score their headache pain on a 10-point scale prior to using the eyedrops and then again 20 minutes after treatment. If a patient had migraine with aura, they were asked to use the eyedrops at the onset of the aura but measure their score at headache onset. If no headaches developed within 20 minutes of the aura, the episode was not included for analysis. All patients were permitted to use their standard oral rescue medication if no relief occurred after 20 minutes of pain onset.

Continue to: The groups were observed...

 

 

The groups were observed for 3 months and then followed for a 1-month washout period, during which they received no study medications. The groups were then crossed over to the other treatment and were observed for another 3 months. The primary outcome was a reduction in pain score by 4 or more points, or to 0 on a 10-point pain scale, 20 minutes after treatment. The secondary outcome was nonuse of oral rescue medication.

The primary outcome was achieved in 233 of 284 (82%) timolol-treated migraines, compared to 38 of 271 (14%) placebo-treated migraines.

Forty-three patients were included in a modified intention-to-treat analysis. The primary outcome was achieved in 233 of 284 (82%) timolol-treated migraines, compared to 38 of 271 (14%) placebo-treated migraines (percentage difference = 68 percentage points; 95% CI, 62-74 percentage points; P < .001). The mean pain score at the onset of migraine attacks was 6.01 for those treated with timolol and 5.93 for those treated with placebo. Patients treated with timolol had a reduction in pain of 5.98 points, compared with 0.93 points after using placebo (difference = 5.05; 95% CI, 4.19-5.91). No attacks included in the data required oral rescue medications, and there were no systemic adverse effects from the timolol eyedrops.

 

WHAT’S NEW

Evidence of benefit as abortive therapy for acute migraine

This randomized controlled trial (RCT) showed evidence to support timolol maleate ophthalmic solution 0.5% vs placebo for treatment of acute migraine by significantly reducing pain when taken at the onset of an acute migraine attack.

CAVEATS

Single-center trial, measuring limited response time

The generalizability of this RCT is limited because it was a single-center trial with a study population from a single region in India. It is unknown whether pain relief, adverse effects, or adherence would differ for the global population. Additionally, only migraines with headache were included in the analysis, limiting non-headache migraine subgroup-directed treatment. Also, this trial evaluated only the response to treatment at 20 minutes, and it is unknown if pain response continued for several hours. Headaches that began more than 20 minutes after the onset of aura were not evaluated.

CHALLENGES TO IMPLEMENTATION

Timolol’s systemic adverse effects require caution

Systemic beta-blocker effects (eg, bradycardia, hypotension, drowsiness, and bronchospasm) from topical timolol have been reported. Caution should be used when prescribing timolol for patients with current cardiovascular and pulmonary conditions. 

ACKNOWLEDGEMENT

The PURLs Surveillance System was supported in part by Grant Number UL1RR024999 from the National Center for Research Resources, a Clinical Translational Science Award to the University of Chicago. The content is solely the responsibility of the authors and does not necessarily represent the official views of the National Center for Research Resources or the National Institutes of Health.

Files
References
  1. Kurian A, Reghunadhan I, Thilak P, et al. Short-term efficacy and safety of topical β-blockers (timolol maleate ophthalmic solution, 0.5%) in acute migraine: a randomized crossover trial. JAMA Ophthalmol. 2020;138:1160-1166. doi: 10.1001/jamaophthalmol.2020.3676
  2. Global Burden of Disease Study 2013 Collaborators. Global, regional, and national incidence, prevalence, and years lived with disability for 301 acute and chronic diseases and injuries in 188 countries, 1990-2013: a systematic analysis for the Global Burden of Disease Study 2013. Lancet. 2015;386:743-800. doi: 10.1016/S0140-6736(15)60692-4
  3. Yeh WZ, Blizzard L, Taylor BV. What is the actual prevalence of migraine? Brain Behav. 2018;8:e00950. doi: 10.1002/brb3.950
  4. Leonardi M, Raggi A. Burden of migraine: international perspectives. Neurol Sci. 2013;34(suppl 1):S117-S118. doi: 10.1007/s10072-013-1387-8
  5. Headache Classification Committee of the International Headache Society (IHS). The International Classification of Headache Disorders, 3rd edition (beta version). Cephalalgia. 2013;33:629-808. doi: 10.1177/0333102413485658
  6. Ubrogepant. GoodRx. Accessed May 23, 2022. www.goodrx.com/ubrogepant
  7. Orr SL, Friedman BW, Christie S, et al. Management of adults with acute migraine in the emergency department: the American Headache Society evidence assessment of parenteral pharmacotherapies. Headache. 2016;56:911-940. doi: 10.1111/head.12835
  8. 8. Migliazzo CV, Hagan JC III. Beta blocker eye drops for treatment of acute migraine. Mo Med. 2014;111:283-288.
  9. 9. Cossack M, Nabrinsky E, Turner H, et al. Timolol eyedrops in the treatment of acute migraine attacks: a randomized crossover study. JAMA Neurol. 2018;75:1024-1025. doi: 10.1001/jamaneurol.2018.0970
  10. 10. Timolol. GoodRx. Accessed May 23, 2022. www.goodrx.com/timolol
Article PDF
Author and Disclosure Information

UPMC St. Margaret Family Medicine Residency Program, Pittsburgh, PA

DEPUTY EDITOR
Anne Mounsey, MD

Department of Family Medicine, University of North Carolina, Chapel Hill

Issue
The Journal of Family Practice - 71(5)
Publications
Topics
Page Number
222-223,226
Sections
Files
Files
Author and Disclosure Information

UPMC St. Margaret Family Medicine Residency Program, Pittsburgh, PA

DEPUTY EDITOR
Anne Mounsey, MD

Department of Family Medicine, University of North Carolina, Chapel Hill

Author and Disclosure Information

UPMC St. Margaret Family Medicine Residency Program, Pittsburgh, PA

DEPUTY EDITOR
Anne Mounsey, MD

Department of Family Medicine, University of North Carolina, Chapel Hill

Article PDF
Article PDF

ILLUSTRATIVE CASE

A 35-year-old woman with no significant past medical history presents for follow-up of migraine. At the previous visit, she was prescribed sumatriptan for abortive therapy. However, she has been having significant adverse effect intolerance from the oral formulation, and the nasal formulation is cost prohibitive. What can you recommend as an alternative abortive therapy for this patient’s migraine?

Migraine is among the most common causes of disability worldwide, affecting more than 10% of the global population.2 The prevalence of migraine is between 2.6% and 21.7% across multiple countries.3 On a scale of 0% to 100%, disability caused by migraine is 43.3%, comparable to the first 2 days after an acute myocardial infarction (42.2%) and severe dementia (43.8%).4

Abortive therapy for acute migraine includes nonsteroidal anti-inflammatory drugs (NSAIDs), triptans, ergots, and antiemetics. However, these options are predominantly administered by mouth; non-oral formulations tend to be cost prohibitive and difficult to obtain.

Nausea and vomiting, common components of migraine (that are included in International Classification of Headache Disorders, 3rd edition [ICHD-3] criteria for migraine5) present obstacles to effective oral administration if experienced by the patient. In addition, for migraine refractory to first-line treatments, abortive options—including the recently approved calcitonin gene-related peptide (CGRP) receptor antagonists ubrogepant and rimegepant—are also cost prohibitive, potentially costing more than $1000 for 10 tablets (100 mg), depending on insurance coverage.6

Two oral beta-blockers, propranolol and timolol, are approved by the US Food and Drug Administration for migraine prophylaxis. Unfortunately, oral beta-blockers are ineffective for abortive treatment.7 Ophthalmic timolol is typically used in the treatment of glaucoma, but there have been case reports describing its benefits in acute migraine treatment.8,9 In addition, ophthalmic timolol is far cheaper than medications such as ubrogepant.10 A 2014 case series of 7 patients discussed ophthalmic beta-blockers as an effective and possibly cheaper option for acute migraine treatment.8 A randomized, crossover, placebo-controlled pilot study of 198 migraine attacks in 10 participants using timolol eyedrops for abortive therapy found timolol was not significantly more effective than placebo.9 However, it was an underpowered pilot study, with a lack of masking and an imperfect placebo. The trial discussed here was a controlled, prospective study investigating topical beta-blockers for acute migraine treatment.

STUDY SUMMARY

Crossover study achieved primary endpoint in pain reduction

This randomized, single-center, double-masked, crossover trial compared timolol maleate ophthalmic solution 0.5% with placebo among 43 patients ages 12 or older presenting with a diagnosis of migraine based on ICHD-3 (beta) criteria. Patients were eligible if they had not taken any antimigraine medications for at least 1 month prior to the study and were excluded if they had taken systemic beta-blockers at baseline, or had asthma, bradyarrhythmias, or cardiac dysfunction.

Patients were randomized 1:1 to treatment with timolol maleate 0.5% eyedrops or placebo. At the earliest onset of migraine, patients used 1 drop of timolol maleate 0.5% or placebo in each eye; if they experienced no relief after 10 minutes, they used a second drop or matching placebo. Patients were instructed to score their headache pain on a 10-point scale prior to using the eyedrops and then again 20 minutes after treatment. If a patient had migraine with aura, they were asked to use the eyedrops at the onset of the aura but measure their score at headache onset. If no headaches developed within 20 minutes of the aura, the episode was not included for analysis. All patients were permitted to use their standard oral rescue medication if no relief occurred after 20 minutes of pain onset.

Continue to: The groups were observed...

 

 

The groups were observed for 3 months and then followed for a 1-month washout period, during which they received no study medications. The groups were then crossed over to the other treatment and were observed for another 3 months. The primary outcome was a reduction in pain score by 4 or more points, or to 0 on a 10-point pain scale, 20 minutes after treatment. The secondary outcome was nonuse of oral rescue medication.

The primary outcome was achieved in 233 of 284 (82%) timolol-treated migraines, compared to 38 of 271 (14%) placebo-treated migraines.

Forty-three patients were included in a modified intention-to-treat analysis. The primary outcome was achieved in 233 of 284 (82%) timolol-treated migraines, compared to 38 of 271 (14%) placebo-treated migraines (percentage difference = 68 percentage points; 95% CI, 62-74 percentage points; P < .001). The mean pain score at the onset of migraine attacks was 6.01 for those treated with timolol and 5.93 for those treated with placebo. Patients treated with timolol had a reduction in pain of 5.98 points, compared with 0.93 points after using placebo (difference = 5.05; 95% CI, 4.19-5.91). No attacks included in the data required oral rescue medications, and there were no systemic adverse effects from the timolol eyedrops.

 

WHAT’S NEW

Evidence of benefit as abortive therapy for acute migraine

This randomized controlled trial (RCT) showed evidence to support timolol maleate ophthalmic solution 0.5% vs placebo for treatment of acute migraine by significantly reducing pain when taken at the onset of an acute migraine attack.

CAVEATS

Single-center trial, measuring limited response time

The generalizability of this RCT is limited because it was a single-center trial with a study population from a single region in India. It is unknown whether pain relief, adverse effects, or adherence would differ for the global population. Additionally, only migraines with headache were included in the analysis, limiting non-headache migraine subgroup-directed treatment. Also, this trial evaluated only the response to treatment at 20 minutes, and it is unknown if pain response continued for several hours. Headaches that began more than 20 minutes after the onset of aura were not evaluated.

CHALLENGES TO IMPLEMENTATION

Timolol’s systemic adverse effects require caution

Systemic beta-blocker effects (eg, bradycardia, hypotension, drowsiness, and bronchospasm) from topical timolol have been reported. Caution should be used when prescribing timolol for patients with current cardiovascular and pulmonary conditions. 

ACKNOWLEDGEMENT

The PURLs Surveillance System was supported in part by Grant Number UL1RR024999 from the National Center for Research Resources, a Clinical Translational Science Award to the University of Chicago. The content is solely the responsibility of the authors and does not necessarily represent the official views of the National Center for Research Resources or the National Institutes of Health.

ILLUSTRATIVE CASE

A 35-year-old woman with no significant past medical history presents for follow-up of migraine. At the previous visit, she was prescribed sumatriptan for abortive therapy. However, she has been having significant adverse effect intolerance from the oral formulation, and the nasal formulation is cost prohibitive. What can you recommend as an alternative abortive therapy for this patient’s migraine?

Migraine is among the most common causes of disability worldwide, affecting more than 10% of the global population.2 The prevalence of migraine is between 2.6% and 21.7% across multiple countries.3 On a scale of 0% to 100%, disability caused by migraine is 43.3%, comparable to the first 2 days after an acute myocardial infarction (42.2%) and severe dementia (43.8%).4

Abortive therapy for acute migraine includes nonsteroidal anti-inflammatory drugs (NSAIDs), triptans, ergots, and antiemetics. However, these options are predominantly administered by mouth; non-oral formulations tend to be cost prohibitive and difficult to obtain.

Nausea and vomiting, common components of migraine (that are included in International Classification of Headache Disorders, 3rd edition [ICHD-3] criteria for migraine5) present obstacles to effective oral administration if experienced by the patient. In addition, for migraine refractory to first-line treatments, abortive options—including the recently approved calcitonin gene-related peptide (CGRP) receptor antagonists ubrogepant and rimegepant—are also cost prohibitive, potentially costing more than $1000 for 10 tablets (100 mg), depending on insurance coverage.6

Two oral beta-blockers, propranolol and timolol, are approved by the US Food and Drug Administration for migraine prophylaxis. Unfortunately, oral beta-blockers are ineffective for abortive treatment.7 Ophthalmic timolol is typically used in the treatment of glaucoma, but there have been case reports describing its benefits in acute migraine treatment.8,9 In addition, ophthalmic timolol is far cheaper than medications such as ubrogepant.10 A 2014 case series of 7 patients discussed ophthalmic beta-blockers as an effective and possibly cheaper option for acute migraine treatment.8 A randomized, crossover, placebo-controlled pilot study of 198 migraine attacks in 10 participants using timolol eyedrops for abortive therapy found timolol was not significantly more effective than placebo.9 However, it was an underpowered pilot study, with a lack of masking and an imperfect placebo. The trial discussed here was a controlled, prospective study investigating topical beta-blockers for acute migraine treatment.

STUDY SUMMARY

Crossover study achieved primary endpoint in pain reduction

This randomized, single-center, double-masked, crossover trial compared timolol maleate ophthalmic solution 0.5% with placebo among 43 patients ages 12 or older presenting with a diagnosis of migraine based on ICHD-3 (beta) criteria. Patients were eligible if they had not taken any antimigraine medications for at least 1 month prior to the study and were excluded if they had taken systemic beta-blockers at baseline, or had asthma, bradyarrhythmias, or cardiac dysfunction.

Patients were randomized 1:1 to treatment with timolol maleate 0.5% eyedrops or placebo. At the earliest onset of migraine, patients used 1 drop of timolol maleate 0.5% or placebo in each eye; if they experienced no relief after 10 minutes, they used a second drop or matching placebo. Patients were instructed to score their headache pain on a 10-point scale prior to using the eyedrops and then again 20 minutes after treatment. If a patient had migraine with aura, they were asked to use the eyedrops at the onset of the aura but measure their score at headache onset. If no headaches developed within 20 minutes of the aura, the episode was not included for analysis. All patients were permitted to use their standard oral rescue medication if no relief occurred after 20 minutes of pain onset.

Continue to: The groups were observed...

 

 

The groups were observed for 3 months and then followed for a 1-month washout period, during which they received no study medications. The groups were then crossed over to the other treatment and were observed for another 3 months. The primary outcome was a reduction in pain score by 4 or more points, or to 0 on a 10-point pain scale, 20 minutes after treatment. The secondary outcome was nonuse of oral rescue medication.

The primary outcome was achieved in 233 of 284 (82%) timolol-treated migraines, compared to 38 of 271 (14%) placebo-treated migraines.

Forty-three patients were included in a modified intention-to-treat analysis. The primary outcome was achieved in 233 of 284 (82%) timolol-treated migraines, compared to 38 of 271 (14%) placebo-treated migraines (percentage difference = 68 percentage points; 95% CI, 62-74 percentage points; P < .001). The mean pain score at the onset of migraine attacks was 6.01 for those treated with timolol and 5.93 for those treated with placebo. Patients treated with timolol had a reduction in pain of 5.98 points, compared with 0.93 points after using placebo (difference = 5.05; 95% CI, 4.19-5.91). No attacks included in the data required oral rescue medications, and there were no systemic adverse effects from the timolol eyedrops.

 

WHAT’S NEW

Evidence of benefit as abortive therapy for acute migraine

This randomized controlled trial (RCT) showed evidence to support timolol maleate ophthalmic solution 0.5% vs placebo for treatment of acute migraine by significantly reducing pain when taken at the onset of an acute migraine attack.

CAVEATS

Single-center trial, measuring limited response time

The generalizability of this RCT is limited because it was a single-center trial with a study population from a single region in India. It is unknown whether pain relief, adverse effects, or adherence would differ for the global population. Additionally, only migraines with headache were included in the analysis, limiting non-headache migraine subgroup-directed treatment. Also, this trial evaluated only the response to treatment at 20 minutes, and it is unknown if pain response continued for several hours. Headaches that began more than 20 minutes after the onset of aura were not evaluated.

CHALLENGES TO IMPLEMENTATION

Timolol’s systemic adverse effects require caution

Systemic beta-blocker effects (eg, bradycardia, hypotension, drowsiness, and bronchospasm) from topical timolol have been reported. Caution should be used when prescribing timolol for patients with current cardiovascular and pulmonary conditions. 

ACKNOWLEDGEMENT

The PURLs Surveillance System was supported in part by Grant Number UL1RR024999 from the National Center for Research Resources, a Clinical Translational Science Award to the University of Chicago. The content is solely the responsibility of the authors and does not necessarily represent the official views of the National Center for Research Resources or the National Institutes of Health.

References
  1. Kurian A, Reghunadhan I, Thilak P, et al. Short-term efficacy and safety of topical β-blockers (timolol maleate ophthalmic solution, 0.5%) in acute migraine: a randomized crossover trial. JAMA Ophthalmol. 2020;138:1160-1166. doi: 10.1001/jamaophthalmol.2020.3676
  2. Global Burden of Disease Study 2013 Collaborators. Global, regional, and national incidence, prevalence, and years lived with disability for 301 acute and chronic diseases and injuries in 188 countries, 1990-2013: a systematic analysis for the Global Burden of Disease Study 2013. Lancet. 2015;386:743-800. doi: 10.1016/S0140-6736(15)60692-4
  3. Yeh WZ, Blizzard L, Taylor BV. What is the actual prevalence of migraine? Brain Behav. 2018;8:e00950. doi: 10.1002/brb3.950
  4. Leonardi M, Raggi A. Burden of migraine: international perspectives. Neurol Sci. 2013;34(suppl 1):S117-S118. doi: 10.1007/s10072-013-1387-8
  5. Headache Classification Committee of the International Headache Society (IHS). The International Classification of Headache Disorders, 3rd edition (beta version). Cephalalgia. 2013;33:629-808. doi: 10.1177/0333102413485658
  6. Ubrogepant. GoodRx. Accessed May 23, 2022. www.goodrx.com/ubrogepant
  7. Orr SL, Friedman BW, Christie S, et al. Management of adults with acute migraine in the emergency department: the American Headache Society evidence assessment of parenteral pharmacotherapies. Headache. 2016;56:911-940. doi: 10.1111/head.12835
  8. 8. Migliazzo CV, Hagan JC III. Beta blocker eye drops for treatment of acute migraine. Mo Med. 2014;111:283-288.
  9. 9. Cossack M, Nabrinsky E, Turner H, et al. Timolol eyedrops in the treatment of acute migraine attacks: a randomized crossover study. JAMA Neurol. 2018;75:1024-1025. doi: 10.1001/jamaneurol.2018.0970
  10. 10. Timolol. GoodRx. Accessed May 23, 2022. www.goodrx.com/timolol
References
  1. Kurian A, Reghunadhan I, Thilak P, et al. Short-term efficacy and safety of topical β-blockers (timolol maleate ophthalmic solution, 0.5%) in acute migraine: a randomized crossover trial. JAMA Ophthalmol. 2020;138:1160-1166. doi: 10.1001/jamaophthalmol.2020.3676
  2. Global Burden of Disease Study 2013 Collaborators. Global, regional, and national incidence, prevalence, and years lived with disability for 301 acute and chronic diseases and injuries in 188 countries, 1990-2013: a systematic analysis for the Global Burden of Disease Study 2013. Lancet. 2015;386:743-800. doi: 10.1016/S0140-6736(15)60692-4
  3. Yeh WZ, Blizzard L, Taylor BV. What is the actual prevalence of migraine? Brain Behav. 2018;8:e00950. doi: 10.1002/brb3.950
  4. Leonardi M, Raggi A. Burden of migraine: international perspectives. Neurol Sci. 2013;34(suppl 1):S117-S118. doi: 10.1007/s10072-013-1387-8
  5. Headache Classification Committee of the International Headache Society (IHS). The International Classification of Headache Disorders, 3rd edition (beta version). Cephalalgia. 2013;33:629-808. doi: 10.1177/0333102413485658
  6. Ubrogepant. GoodRx. Accessed May 23, 2022. www.goodrx.com/ubrogepant
  7. Orr SL, Friedman BW, Christie S, et al. Management of adults with acute migraine in the emergency department: the American Headache Society evidence assessment of parenteral pharmacotherapies. Headache. 2016;56:911-940. doi: 10.1111/head.12835
  8. 8. Migliazzo CV, Hagan JC III. Beta blocker eye drops for treatment of acute migraine. Mo Med. 2014;111:283-288.
  9. 9. Cossack M, Nabrinsky E, Turner H, et al. Timolol eyedrops in the treatment of acute migraine attacks: a randomized crossover study. JAMA Neurol. 2018;75:1024-1025. doi: 10.1001/jamaneurol.2018.0970
  10. 10. Timolol. GoodRx. Accessed May 23, 2022. www.goodrx.com/timolol
Issue
The Journal of Family Practice - 71(5)
Issue
The Journal of Family Practice - 71(5)
Page Number
222-223,226
Page Number
222-223,226
Publications
Publications
Topics
Article Type
Display Headline
Migraine relief in 20 minutes using eyedrops?
Display Headline
Migraine relief in 20 minutes using eyedrops?
Sections
PURLs Copyright
Copyright © 2022. The Family Physicians Inquiries Network. All rights reserved.
Inside the Article

PRACTICE CHANGER

Consider timolol maleate 0.5% eyedrops as a quick and effective abortive therapy for migraine.1

STRENGTH OF RECOMMENDATION

B: Based on a single randomized controlled trial.1

Kurian A, Reghunadhan I, Thilak P, et al. Short-term efficacy and safety of topical β-blockers (timolol maleate ophthalmic solution, 0.5%) in acute migraine: a randomized crossover trial. JAMA Ophthalmol. 2020;138:1160-1166.

Disallow All Ads
Content Gating
No Gating (article Unlocked/Free)
Alternative CME
Disqus Comments
Default
Use ProPublica
Hide sidebar & use full width
render the right sidebar.
Conference Recap Checkbox
Not Conference Recap
Clinical Edge
Display the Slideshow in this Article
Medscape Article
Display survey writer
Reuters content
Disable Inline Native ads
WebMD Article
Article PDF Media
Media Files

Getting a jump on recovery from sports-related concussion

Article Type
Changed
Wed, 04/13/2022 - 13:30
Display Headline
Getting a jump on recovery from sports-related concussion

ILLUSTRATIVE CASE

A 16-year-old girl with no chronic medical illness presents to your office with her parents after sustaining a head injury at a soccer game over the weekend. She collided with another player while attempting to head the ball. Immediately afterward, she was taken off the field and assessed. She was confused but had a normal level of consciousness and denied vision changes, weakness or tingling in her arms or legs, severe headache, or neck pain. Further testing revealed dizziness and abnormal balance. Her confusion and abnormal balance resolved after 1 day. She has had a mild headache and light sensitivity since the event. She otherwise feels well at rest in the office. She wants to recover quickly but safely and has heard conflicting statements about whether she should completely rest or start back to light activity now.

Sports-related concussions (SRCs) are highly prevalent in the United States, with as many as 3.8 million cases annually. Of those, 1.1 to 1.9 million cases are in children 18 years old or younger.2,3 SRCs are defined by the Concussion in Sport Group (CISG) 2017 consensus statement as involving the following criteria: (1) direct or indirect trauma anywhere on the body with force transmitted to the head; (2) rapid or delayed symptom presentation, typically with spontaneous resolution; (3) functional rather than structural injury; and (4) occurrence with or without loss of consciousness with stepwise symptom resolution.4

SRCs do not have a proven, effective treatment and can have short- or long-term consequences. Initial treatment includes removing athletes from play immediately after an event. The American Academy of Neurology recommends that athletes not return to play until the concussion is resolved, as judged by a health care provider, and the athlete is asymptomatic when off medication.2

The CISG recommends a 6-step approach, with each step taking at least 24 hours.4 The final step is a return to normal activity.4 This working group recommended extensive study of rehabilitation programs involving subsymptom threshold exercise (ie, exercise performed at a level that does not exacerbate symptoms) before implementation as routine practice. Evidence from a 2015 study suggests that following strict rest for 5 days until complete symptom resolution may prolong recovery compared with rest for only 1 to 2 days.5 Additionally, strict rest did not show a difference in neurocognitive or balance outcomes in that study, and the authors noted it may also negatively impact academic, sports, and social function in adolescents.5 This study looked at the potential benefit of subsymptom threshold exercise during recovery from SRC.1

STUDY SUMMARY

Light aerobic exercise may help speed recovery

This multicenter, prospective, parallel, randomized clinical trial compared subsymptom threshold aerobic exercise to ­placebo-like stretching. Patients were included if they were ages 13 to 18 years and presented within 10 days of an SRC, as diagnosed using the CISG criteria. Exclusion criteria included focal neurologic deficits; history of moderate or severe traumatic brain injury; inability to exercise due to orthopedic injury, cervical spine injury, diabetes, or heart disease; increased cardiac risk; or low postconcussion symptom severity. Patients with a diagnosis of and treatment with medication for ­attention-deficit/hyperactivity disorder (ADHD), depression, anxiety, or learning disorder were excluded, as were patients with a history of more than 3 previous concussions.

It’s unclear whether subsymptom threshold exercise is safe and effective in adolescents with a history of multiple concussions, as those with > 3 concussions were excluded from this study.

Patients in the aerobic exercise group were instructed to use a stationary bike or treadmill (or equivalent walking or jogging if they did not have access to this equipment) at a prescribed heart rate. The target heart rate was 80% of the heart rate achieved during initial assessment with the Buffalo Concussion Treadmill Test (BCTT).6 Patients in this group were instructed to exercise for 20 minutes or to the point at which their symptoms increased by 2 points (on a 10-point scale) from pre-exercise levels, whichever came first, with rest prescribed at all other times.

For the placebo-like group, a stretching instruction booklet was provided, with the goal of achieving a heart rate that was not significantly elevated. Participants in this group were told to perform the stretches for 20 minutes daily. Of note, researchers ensured the level of physician and research staff attention was similar for each patient, regardless of treatment group, to prevent intervention bias. Additionally, interventions were not initiated prior to 48 hours from the time of injury.

Continue to: The primary outcome...

 

 

The primary outcome was number of days to recovery since the date of injury. This was defined as symptom resolution to normal (as evaluated by a physician blinded to the study group) and by the patient’s ability to exercise to exhaustion without symptom exacerbation on the BCTT. Secondary outcomes measured the proportion of patients with delayed recovery (defined as recovery requiring > 30 days) and daily symptom scores.

Of 165 patients meeting the inclusion criteria, 52 patients were excluded prior to randomization (12 patients chose not to participate, 39 were excluded for lack of symptoms, and 1 withdrew due to severe symptoms on the BCTT). A total of 113 were randomized to either group, and 103 patients completed the study (10 patients did not complete the study or had another illness during the intervention). The study analysis included 52 patients in the aerobic exercise group and 51 in the placebo-like stretching group. The study was powered to detect a significant difference in recovery time.

Patients were about equally divided by sex, with a mean age of 15 years. Patients who had no previous concussion made up 50% of the aerobic group and 57% of the stretching group. The average time since injury was similar in the aerobic and stretching groups (4.9 days and 4.8 days, respectively). The aerobic exercise group recovered in a median of 13 days (interquartile range [IQR] = 10-18.5 days) compared with a median of 17 days (IQR = 13-23 days) for the stretching group (P = .009). The incidence of delayed recovery (> 30 days) was higher in the stretching group (n = 7) compared with the aerobic exercise group (n = 2) but was not statistically significant. Daily symptom reporting occurred at a high rate in both groups, with patients stating that they performed their prescribed exercise 89% of the time. No adverse events were reported.

 

WHAT’S NEW

First high-quality study to support evidence for early light activity

This is the first high-quality study of subsymptom threshold exercise for SRC. Its findings add to the growing body of evidence that early engagement in light aerobic activity that does not provoke symptoms (but not fully returning to sports activity) can aid in recovery from an SRC.

CAVEATS

Narrow study population limits application of findings

It is unclear if subsymptom threshold exercise is safe and effective in adolescents with a history of multiple concussions, as those with more than 3 concussions were excluded from this study. Additionally, patients with comorbidities such as ADHD, depression, anxiety, or learning disorders were not included in this study, which limits the application of these findings. The generalizability of this study is limited in younger children, adults, those with increased cardiovascular risk, and in patients with concussions that are not sports related.

CHALLENGES TO IMPLEMENTATION

More real-world studies needed to confirm benefit

The majority of adolescent athletes in this study completed the subsymptom threshold exercise in a monitored environment with trainers, heart rate monitors, and access to equipment, limiting the study’s generalizability. Additionally, physicians need to be familiar with the BCTT to assign heart rate goals and assess improvement. The study environment may be feasible for some but not others. Studies evaluating real-world settings with athletes self-monitoring for symptom threshold with stepwise evaluations are needed and may be more broadly applicable.

ACKNOWLEDGEMENT

The PURLs Surveillance System was supported in part by Grant Number UL1RR024999 from the National Center for Research Resources, a Clinical Translational Science Award to the University of Chicago. The content is solely the responsibility of the authors and does not necessarily represent the official views of the National Center for Research Resources or the National Institutes of Health.

Files
References

1. Leddy JJ, Haider MN, Ellis MJ, et al. Early subthreshold aerobic exercise for sport-related concussion: a randomized clinical trial. JAMA Pediatr. 2019;173:319-325. doi: 10.1001/jamapediatrics.2018.4397

2. Giza CC, Kutcher JS, Ashwal S, et al. Summary of evidence-based guideline update: evaluation and management of concussion in sports: report of the Guideline Development Subcommittee of the American Academy of Neurology. Neurology. 2013;80:2250-2257. doi: 10.1212/WNL.0b013e31828d57dd

3. Bryan MA, Rowhani-Rahbar A, Comstock RD, et al; Seattle Sports Concussion Research Collaborative. Sports- and recreation-related concussions in US youth. Pediatrics. 2016;138:e20154635. doi: 10.1542/peds.2015-4635

4. McCrory P, Meeuwisse W, Dvořák J, et al. Consensus statement on concussion in sport—the 5th international conference on concussion in sport held in Berlin, October 2016. Br J Sports Med. 2017;51:838-847. doi: 10.1136/bjsports-2017-097699

5. Thomas DG, Apps JN, Hoffmann RG, et al. Benefits of strict rest after acute concussion: a randomized controlled trial. Pediatrics. 2015;135:213-223.

6. Leddy JJ, Haider MN, Willer BS. Buffalo Concussion Treadmill Test (BCTT) – Instruction Manual. Accessed March 16, 2022. https://cdn-links.lww.com/permalink/jsm/a/jsm_2020_01_28_haider_19-313_sdc1.pdf

Article PDF
Author and Disclosure Information

UPMC St. Margaret, Pittsburgh, PA

DEPUTY EDITOR
Katherine Hale, PharmD, BCPS, MFA

Department of Nursing, Heritage University, Toppenish, WA

Issue
The Journal of Family Practice - 71(3)
Publications
Topics
Page Number
135-137
Sections
Files
Files
Author and Disclosure Information

UPMC St. Margaret, Pittsburgh, PA

DEPUTY EDITOR
Katherine Hale, PharmD, BCPS, MFA

Department of Nursing, Heritage University, Toppenish, WA

Author and Disclosure Information

UPMC St. Margaret, Pittsburgh, PA

DEPUTY EDITOR
Katherine Hale, PharmD, BCPS, MFA

Department of Nursing, Heritage University, Toppenish, WA

Article PDF
Article PDF

ILLUSTRATIVE CASE

A 16-year-old girl with no chronic medical illness presents to your office with her parents after sustaining a head injury at a soccer game over the weekend. She collided with another player while attempting to head the ball. Immediately afterward, she was taken off the field and assessed. She was confused but had a normal level of consciousness and denied vision changes, weakness or tingling in her arms or legs, severe headache, or neck pain. Further testing revealed dizziness and abnormal balance. Her confusion and abnormal balance resolved after 1 day. She has had a mild headache and light sensitivity since the event. She otherwise feels well at rest in the office. She wants to recover quickly but safely and has heard conflicting statements about whether she should completely rest or start back to light activity now.

Sports-related concussions (SRCs) are highly prevalent in the United States, with as many as 3.8 million cases annually. Of those, 1.1 to 1.9 million cases are in children 18 years old or younger.2,3 SRCs are defined by the Concussion in Sport Group (CISG) 2017 consensus statement as involving the following criteria: (1) direct or indirect trauma anywhere on the body with force transmitted to the head; (2) rapid or delayed symptom presentation, typically with spontaneous resolution; (3) functional rather than structural injury; and (4) occurrence with or without loss of consciousness with stepwise symptom resolution.4

SRCs do not have a proven, effective treatment and can have short- or long-term consequences. Initial treatment includes removing athletes from play immediately after an event. The American Academy of Neurology recommends that athletes not return to play until the concussion is resolved, as judged by a health care provider, and the athlete is asymptomatic when off medication.2

The CISG recommends a 6-step approach, with each step taking at least 24 hours.4 The final step is a return to normal activity.4 This working group recommended extensive study of rehabilitation programs involving subsymptom threshold exercise (ie, exercise performed at a level that does not exacerbate symptoms) before implementation as routine practice. Evidence from a 2015 study suggests that following strict rest for 5 days until complete symptom resolution may prolong recovery compared with rest for only 1 to 2 days.5 Additionally, strict rest did not show a difference in neurocognitive or balance outcomes in that study, and the authors noted it may also negatively impact academic, sports, and social function in adolescents.5 This study looked at the potential benefit of subsymptom threshold exercise during recovery from SRC.1

STUDY SUMMARY

Light aerobic exercise may help speed recovery

This multicenter, prospective, parallel, randomized clinical trial compared subsymptom threshold aerobic exercise to ­placebo-like stretching. Patients were included if they were ages 13 to 18 years and presented within 10 days of an SRC, as diagnosed using the CISG criteria. Exclusion criteria included focal neurologic deficits; history of moderate or severe traumatic brain injury; inability to exercise due to orthopedic injury, cervical spine injury, diabetes, or heart disease; increased cardiac risk; or low postconcussion symptom severity. Patients with a diagnosis of and treatment with medication for ­attention-deficit/hyperactivity disorder (ADHD), depression, anxiety, or learning disorder were excluded, as were patients with a history of more than 3 previous concussions.

It’s unclear whether subsymptom threshold exercise is safe and effective in adolescents with a history of multiple concussions, as those with > 3 concussions were excluded from this study.

Patients in the aerobic exercise group were instructed to use a stationary bike or treadmill (or equivalent walking or jogging if they did not have access to this equipment) at a prescribed heart rate. The target heart rate was 80% of the heart rate achieved during initial assessment with the Buffalo Concussion Treadmill Test (BCTT).6 Patients in this group were instructed to exercise for 20 minutes or to the point at which their symptoms increased by 2 points (on a 10-point scale) from pre-exercise levels, whichever came first, with rest prescribed at all other times.

For the placebo-like group, a stretching instruction booklet was provided, with the goal of achieving a heart rate that was not significantly elevated. Participants in this group were told to perform the stretches for 20 minutes daily. Of note, researchers ensured the level of physician and research staff attention was similar for each patient, regardless of treatment group, to prevent intervention bias. Additionally, interventions were not initiated prior to 48 hours from the time of injury.

Continue to: The primary outcome...

 

 

The primary outcome was number of days to recovery since the date of injury. This was defined as symptom resolution to normal (as evaluated by a physician blinded to the study group) and by the patient’s ability to exercise to exhaustion without symptom exacerbation on the BCTT. Secondary outcomes measured the proportion of patients with delayed recovery (defined as recovery requiring > 30 days) and daily symptom scores.

Of 165 patients meeting the inclusion criteria, 52 patients were excluded prior to randomization (12 patients chose not to participate, 39 were excluded for lack of symptoms, and 1 withdrew due to severe symptoms on the BCTT). A total of 113 were randomized to either group, and 103 patients completed the study (10 patients did not complete the study or had another illness during the intervention). The study analysis included 52 patients in the aerobic exercise group and 51 in the placebo-like stretching group. The study was powered to detect a significant difference in recovery time.

Patients were about equally divided by sex, with a mean age of 15 years. Patients who had no previous concussion made up 50% of the aerobic group and 57% of the stretching group. The average time since injury was similar in the aerobic and stretching groups (4.9 days and 4.8 days, respectively). The aerobic exercise group recovered in a median of 13 days (interquartile range [IQR] = 10-18.5 days) compared with a median of 17 days (IQR = 13-23 days) for the stretching group (P = .009). The incidence of delayed recovery (> 30 days) was higher in the stretching group (n = 7) compared with the aerobic exercise group (n = 2) but was not statistically significant. Daily symptom reporting occurred at a high rate in both groups, with patients stating that they performed their prescribed exercise 89% of the time. No adverse events were reported.

 

WHAT’S NEW

First high-quality study to support evidence for early light activity

This is the first high-quality study of subsymptom threshold exercise for SRC. Its findings add to the growing body of evidence that early engagement in light aerobic activity that does not provoke symptoms (but not fully returning to sports activity) can aid in recovery from an SRC.

CAVEATS

Narrow study population limits application of findings

It is unclear if subsymptom threshold exercise is safe and effective in adolescents with a history of multiple concussions, as those with more than 3 concussions were excluded from this study. Additionally, patients with comorbidities such as ADHD, depression, anxiety, or learning disorders were not included in this study, which limits the application of these findings. The generalizability of this study is limited in younger children, adults, those with increased cardiovascular risk, and in patients with concussions that are not sports related.

CHALLENGES TO IMPLEMENTATION

More real-world studies needed to confirm benefit

The majority of adolescent athletes in this study completed the subsymptom threshold exercise in a monitored environment with trainers, heart rate monitors, and access to equipment, limiting the study’s generalizability. Additionally, physicians need to be familiar with the BCTT to assign heart rate goals and assess improvement. The study environment may be feasible for some but not others. Studies evaluating real-world settings with athletes self-monitoring for symptom threshold with stepwise evaluations are needed and may be more broadly applicable.

ACKNOWLEDGEMENT

The PURLs Surveillance System was supported in part by Grant Number UL1RR024999 from the National Center for Research Resources, a Clinical Translational Science Award to the University of Chicago. The content is solely the responsibility of the authors and does not necessarily represent the official views of the National Center for Research Resources or the National Institutes of Health.

ILLUSTRATIVE CASE

A 16-year-old girl with no chronic medical illness presents to your office with her parents after sustaining a head injury at a soccer game over the weekend. She collided with another player while attempting to head the ball. Immediately afterward, she was taken off the field and assessed. She was confused but had a normal level of consciousness and denied vision changes, weakness or tingling in her arms or legs, severe headache, or neck pain. Further testing revealed dizziness and abnormal balance. Her confusion and abnormal balance resolved after 1 day. She has had a mild headache and light sensitivity since the event. She otherwise feels well at rest in the office. She wants to recover quickly but safely and has heard conflicting statements about whether she should completely rest or start back to light activity now.

Sports-related concussions (SRCs) are highly prevalent in the United States, with as many as 3.8 million cases annually. Of those, 1.1 to 1.9 million cases are in children 18 years old or younger.2,3 SRCs are defined by the Concussion in Sport Group (CISG) 2017 consensus statement as involving the following criteria: (1) direct or indirect trauma anywhere on the body with force transmitted to the head; (2) rapid or delayed symptom presentation, typically with spontaneous resolution; (3) functional rather than structural injury; and (4) occurrence with or without loss of consciousness with stepwise symptom resolution.4

SRCs do not have a proven, effective treatment and can have short- or long-term consequences. Initial treatment includes removing athletes from play immediately after an event. The American Academy of Neurology recommends that athletes not return to play until the concussion is resolved, as judged by a health care provider, and the athlete is asymptomatic when off medication.2

The CISG recommends a 6-step approach, with each step taking at least 24 hours.4 The final step is a return to normal activity.4 This working group recommended extensive study of rehabilitation programs involving subsymptom threshold exercise (ie, exercise performed at a level that does not exacerbate symptoms) before implementation as routine practice. Evidence from a 2015 study suggests that following strict rest for 5 days until complete symptom resolution may prolong recovery compared with rest for only 1 to 2 days.5 Additionally, strict rest did not show a difference in neurocognitive or balance outcomes in that study, and the authors noted it may also negatively impact academic, sports, and social function in adolescents.5 This study looked at the potential benefit of subsymptom threshold exercise during recovery from SRC.1

STUDY SUMMARY

Light aerobic exercise may help speed recovery

This multicenter, prospective, parallel, randomized clinical trial compared subsymptom threshold aerobic exercise to ­placebo-like stretching. Patients were included if they were ages 13 to 18 years and presented within 10 days of an SRC, as diagnosed using the CISG criteria. Exclusion criteria included focal neurologic deficits; history of moderate or severe traumatic brain injury; inability to exercise due to orthopedic injury, cervical spine injury, diabetes, or heart disease; increased cardiac risk; or low postconcussion symptom severity. Patients with a diagnosis of and treatment with medication for ­attention-deficit/hyperactivity disorder (ADHD), depression, anxiety, or learning disorder were excluded, as were patients with a history of more than 3 previous concussions.

It’s unclear whether subsymptom threshold exercise is safe and effective in adolescents with a history of multiple concussions, as those with > 3 concussions were excluded from this study.

Patients in the aerobic exercise group were instructed to use a stationary bike or treadmill (or equivalent walking or jogging if they did not have access to this equipment) at a prescribed heart rate. The target heart rate was 80% of the heart rate achieved during initial assessment with the Buffalo Concussion Treadmill Test (BCTT).6 Patients in this group were instructed to exercise for 20 minutes or to the point at which their symptoms increased by 2 points (on a 10-point scale) from pre-exercise levels, whichever came first, with rest prescribed at all other times.

For the placebo-like group, a stretching instruction booklet was provided, with the goal of achieving a heart rate that was not significantly elevated. Participants in this group were told to perform the stretches for 20 minutes daily. Of note, researchers ensured the level of physician and research staff attention was similar for each patient, regardless of treatment group, to prevent intervention bias. Additionally, interventions were not initiated prior to 48 hours from the time of injury.

Continue to: The primary outcome...

 

 

The primary outcome was number of days to recovery since the date of injury. This was defined as symptom resolution to normal (as evaluated by a physician blinded to the study group) and by the patient’s ability to exercise to exhaustion without symptom exacerbation on the BCTT. Secondary outcomes measured the proportion of patients with delayed recovery (defined as recovery requiring > 30 days) and daily symptom scores.

Of 165 patients meeting the inclusion criteria, 52 patients were excluded prior to randomization (12 patients chose not to participate, 39 were excluded for lack of symptoms, and 1 withdrew due to severe symptoms on the BCTT). A total of 113 were randomized to either group, and 103 patients completed the study (10 patients did not complete the study or had another illness during the intervention). The study analysis included 52 patients in the aerobic exercise group and 51 in the placebo-like stretching group. The study was powered to detect a significant difference in recovery time.

Patients were about equally divided by sex, with a mean age of 15 years. Patients who had no previous concussion made up 50% of the aerobic group and 57% of the stretching group. The average time since injury was similar in the aerobic and stretching groups (4.9 days and 4.8 days, respectively). The aerobic exercise group recovered in a median of 13 days (interquartile range [IQR] = 10-18.5 days) compared with a median of 17 days (IQR = 13-23 days) for the stretching group (P = .009). The incidence of delayed recovery (> 30 days) was higher in the stretching group (n = 7) compared with the aerobic exercise group (n = 2) but was not statistically significant. Daily symptom reporting occurred at a high rate in both groups, with patients stating that they performed their prescribed exercise 89% of the time. No adverse events were reported.

 

WHAT’S NEW

First high-quality study to support evidence for early light activity

This is the first high-quality study of subsymptom threshold exercise for SRC. Its findings add to the growing body of evidence that early engagement in light aerobic activity that does not provoke symptoms (but not fully returning to sports activity) can aid in recovery from an SRC.

CAVEATS

Narrow study population limits application of findings

It is unclear if subsymptom threshold exercise is safe and effective in adolescents with a history of multiple concussions, as those with more than 3 concussions were excluded from this study. Additionally, patients with comorbidities such as ADHD, depression, anxiety, or learning disorders were not included in this study, which limits the application of these findings. The generalizability of this study is limited in younger children, adults, those with increased cardiovascular risk, and in patients with concussions that are not sports related.

CHALLENGES TO IMPLEMENTATION

More real-world studies needed to confirm benefit

The majority of adolescent athletes in this study completed the subsymptom threshold exercise in a monitored environment with trainers, heart rate monitors, and access to equipment, limiting the study’s generalizability. Additionally, physicians need to be familiar with the BCTT to assign heart rate goals and assess improvement. The study environment may be feasible for some but not others. Studies evaluating real-world settings with athletes self-monitoring for symptom threshold with stepwise evaluations are needed and may be more broadly applicable.

ACKNOWLEDGEMENT

The PURLs Surveillance System was supported in part by Grant Number UL1RR024999 from the National Center for Research Resources, a Clinical Translational Science Award to the University of Chicago. The content is solely the responsibility of the authors and does not necessarily represent the official views of the National Center for Research Resources or the National Institutes of Health.

References

1. Leddy JJ, Haider MN, Ellis MJ, et al. Early subthreshold aerobic exercise for sport-related concussion: a randomized clinical trial. JAMA Pediatr. 2019;173:319-325. doi: 10.1001/jamapediatrics.2018.4397

2. Giza CC, Kutcher JS, Ashwal S, et al. Summary of evidence-based guideline update: evaluation and management of concussion in sports: report of the Guideline Development Subcommittee of the American Academy of Neurology. Neurology. 2013;80:2250-2257. doi: 10.1212/WNL.0b013e31828d57dd

3. Bryan MA, Rowhani-Rahbar A, Comstock RD, et al; Seattle Sports Concussion Research Collaborative. Sports- and recreation-related concussions in US youth. Pediatrics. 2016;138:e20154635. doi: 10.1542/peds.2015-4635

4. McCrory P, Meeuwisse W, Dvořák J, et al. Consensus statement on concussion in sport—the 5th international conference on concussion in sport held in Berlin, October 2016. Br J Sports Med. 2017;51:838-847. doi: 10.1136/bjsports-2017-097699

5. Thomas DG, Apps JN, Hoffmann RG, et al. Benefits of strict rest after acute concussion: a randomized controlled trial. Pediatrics. 2015;135:213-223.

6. Leddy JJ, Haider MN, Willer BS. Buffalo Concussion Treadmill Test (BCTT) – Instruction Manual. Accessed March 16, 2022. https://cdn-links.lww.com/permalink/jsm/a/jsm_2020_01_28_haider_19-313_sdc1.pdf

References

1. Leddy JJ, Haider MN, Ellis MJ, et al. Early subthreshold aerobic exercise for sport-related concussion: a randomized clinical trial. JAMA Pediatr. 2019;173:319-325. doi: 10.1001/jamapediatrics.2018.4397

2. Giza CC, Kutcher JS, Ashwal S, et al. Summary of evidence-based guideline update: evaluation and management of concussion in sports: report of the Guideline Development Subcommittee of the American Academy of Neurology. Neurology. 2013;80:2250-2257. doi: 10.1212/WNL.0b013e31828d57dd

3. Bryan MA, Rowhani-Rahbar A, Comstock RD, et al; Seattle Sports Concussion Research Collaborative. Sports- and recreation-related concussions in US youth. Pediatrics. 2016;138:e20154635. doi: 10.1542/peds.2015-4635

4. McCrory P, Meeuwisse W, Dvořák J, et al. Consensus statement on concussion in sport—the 5th international conference on concussion in sport held in Berlin, October 2016. Br J Sports Med. 2017;51:838-847. doi: 10.1136/bjsports-2017-097699

5. Thomas DG, Apps JN, Hoffmann RG, et al. Benefits of strict rest after acute concussion: a randomized controlled trial. Pediatrics. 2015;135:213-223.

6. Leddy JJ, Haider MN, Willer BS. Buffalo Concussion Treadmill Test (BCTT) – Instruction Manual. Accessed March 16, 2022. https://cdn-links.lww.com/permalink/jsm/a/jsm_2020_01_28_haider_19-313_sdc1.pdf

Issue
The Journal of Family Practice - 71(3)
Issue
The Journal of Family Practice - 71(3)
Page Number
135-137
Page Number
135-137
Publications
Publications
Topics
Article Type
Display Headline
Getting a jump on recovery from sports-related concussion
Display Headline
Getting a jump on recovery from sports-related concussion
Sections
PURLs Copyright
Copyright © 2022. The Family Physicians Inquiries Network. All rights reserved.
Inside the Article

PRACTICE CHANGER

Recommend subsymptom threshold exercise in adolescents with a sports-related concussion. Early return to light aerobic activity not only seems safe but may help speed recovery compared with stretching alone in this patient population.

STRENGTH OF RECOMMENDATION

B: Based on a single multicenter, prospective, randomized clinical trial1

Leddy JJ, Haider MN, Ellis MJ, et al. Early subthreshold aerobic exercise for sport-related concussion: a randomized clinical trial. JAMA Pediatr. 2019;173:319-325. doi: 10.1001/jamapediatrics.2018.4397

Disallow All Ads
Content Gating
No Gating (article Unlocked/Free)
Alternative CME
Disqus Comments
Default
Use ProPublica
Hide sidebar & use full width
render the right sidebar.
Conference Recap Checkbox
Not Conference Recap
Clinical Edge
Display the Slideshow in this Article
Medscape Article
Display survey writer
Reuters content
Disable Inline Native ads
WebMD Article
Article PDF Media
Media Files

An alternative regimen to reduce risk of asthma exacerbations

Article Type
Changed
Fri, 03/12/2021 - 11:26
Display Headline
An alternative regimen to reduce risk of asthma exacerbations

ILLUSTRATIVE CASE

A 37-year-old woman with moderate persistent asthma, controlled on the ICS fluticasone (110 μg twice a day) presents to you for an annual exam. She uses her rescue albuterol inhaler a few times per month. Her last exacerbation was 2 years ago. She has never smoked. She is concerned about continuing to take an ICS every day. What alternative regimen would you recommend for this patient?

According to the Centers for Disease Control and Prevention, asthma affected 24.7 million children and adults in the United States in 2018, accounting for 9.8 million physician visits and 1.6 million emergency department (ED) visits.2 The National Institutes of Health (NIH) asthma care guidelines, updated in 2020, recommend a SABA prn as step 1 for intermittent asthma, along with nonpharmacologic management.3 Once a patient has persistent asthma, treatment escalation to step 2 calls for use of daily maintenance inhalers as the preferred treatment option.3

However, the 2020 Global Initiative for Asthma (GINA) warns that an as-needed SABA does not protect patients from severe exacerbations, and regular use of a SABA alone (> 3 inhalers/year) can increase the risk of exacerbations.4 A meta-analysis and systematic review from 2018 showed that using an ICS/LABA—scheduled and prn for rescue—had lower risk of asthma exacerbations compared with scheduled ICS/LABA with SABA prn for rescue in patients with ­moderate-to-severe persistent asthma.5 Interestingly, the updated 2020 NIH guidelines have adopted this strategy. SABA use prn is no longer recommended for rescue in mild and moderate persistent asthma, and the guidelines now suggest that ICS/LABA be used as rescue in addition to daily medication.3

Although evidence has been mounting for adding the as-needed ICS/LABA for rescue in patients on daily medication, the mainstay has been to provide a SABA prn for rescue use.5 Confusing matters more, evidence is emerging that as-needed ICS/LABA for rescue alone in certain patients is safe and effective. The randomized controlled Novel START study, an open-label, parallel-group study, compared ICS/LABA prn vs scheduled ICS with SABA prn vs SABA alone prn in adult patients with intermittent or mild persistent asthma.6 ICS/LABA prn prevented more exacerbations and provided better daily control than as-needed SABA alone.6 In addition, ICS/LABA as needed resulted in fewer severe exacerbations but potentially poorer daily control than ICS with SABA as needed.6

The PRACTICAL study investigated treatment of patients with intermittent, mild persistent, and moderate persistent asthma.1

STUDY SUMMARY

ICS/LABA prn reduced risk of severe exacerbations

The randomized controlled PRACTICAL study was a 52-week, open-label, parallel-group, superiority trial in New Zealand that compared as-needed ICS/LABA (n = 437) to scheduled ICS plus as-needed SABA (n = 448). Patients were 18 to 75 years old, with a diagnosis of asthma. Applying NIH guideline definitions, these patients would fall into intermittent, mild persistent, or moderate persistent asthma categories, and were on either as-needed SABA alone or a scheduled low- to moderate-dose ICS plus an as-needed SABA in the previous 12 weeks.

Patients on an as-needed SABA prerandomization had to have at least 1 of the following: (1) asthma symptoms or need for a SABA at least twice in the past 4 weeks; (2) at least 1 nighttime awakening due to asthma in the past 4 weeks; or (3) a severe exacerbation requiring oral corticosteroids in the past year. Patients on scheduled ICS plus SABA prn prerandomization were required to have either: (1) low or moderate ICS dosing with partly or well-controlled asthma; or (2) if uncontrolled, poor inhaler technique or adherence.

Continue to: Patients in the ICS/LABA group...

 

 

Patients in the ICS/LABA group were given budesonide 200 µg/formoterol 6 µg, 1 puff prn, and patients in the ICS plus as-needed SABA group were given budesonide 200 µg, 1 puff twice daily, and terbutaline 250 µg, 2 puffs prn. All patients received an asthma action plan that provided guidance on when to seek medical care if asthma worsened, as well as a log to note urgent medical visits and use of systemic corticosteroids. A subset of patients had adherence and dosing monitored by electronic inhaler usage monitors. Patients were seen at 0, 4, 16, 28, 40, and 52 weeks.

Outcomes. The primary outcome was the number of severe exacerbations per patient per year, defined as treatment with oral corticosteroids for ≥ 3 days or ED visit or hospital admission requiring systemic corticosteroids. Among the secondary outcomes were number of moderate and severe exacerbations per patient per year (defined as an unplanned medical visit: primary care, ED, hospital admission, and any duration of steroids); time to first severe exacerbation; assessment with the Asthma Control Questionnaire (ACQ-5); adverse outcomes; and quantity of ICS used (analysis done only for the subset with electronic inhaler monitoring).

This study represents a compelling, real-world look at emerging asthma recommendations.

ACQ-5 takes the mean of 5 questions assessing asthma control in the previous week, with each question ranging from 0 (no impairment) to 6 (maximum impairment). The statistician was blinded to the primary outcome.

 

Results. The rate of severe exacerbations per patient per year was 0.119 in the as-needed ICS/LABA group vs 0.172 in the scheduled ICS plus as-needed SABA group (relative rate [RR] = 0.69; 95% confidence interval [CI], 0.48–1.00). Time to first severe asthma exacerbation was longer in the as-needed ICS/LABA group (hazard ratio = 0.60; 95% CI, 0.40–0.91). The rate of moderate and severe exacerbations per patient per year was lower in the as-needed ICS/LABA group: 0.165 vs 0.237 (RR = 0.70; 95% CI, 0.51–0.95).

ACQ-5 scores were similar at all time points (mean difference = 0.07; 95% CI, –0.03 to 0.17). Adverse events were similar between groups (most commonly nasopharyngitis in both groups). Less ICS was used in the ICS/LABA group (difference = –126.5 µg per day; 95% CI, –171.0 to –81.9).

Continue to: WHAT'S NEW

 

 

WHAT’S NEW

Study lends support to recent recommendations

This study represents a compelling, real-world look at emerging asthma recommendations. This was the first comprehensive study to show that as-needed ICS/LABA therapy prevents more moderate and severe exacerbations and lengthens the time to first severe exacerbation, compared with scheduled ICS plus SABA prn in intermittent, mild persistent, or moderate persistent asthma. These data have been incorporated into the GINA guidelines, which recommend ICS/LABA prn for step 2.

CAVEATS

Potential bias in study design

The LABA used in this study was formoterol, which has a quicker onset than other LABAs. It is likely that not all LABAs can be used the same way, and both the NIH and GINA guidelines call it out specifically. Additionally, the study’s open-label design can introduce bias but may be the only way to simulate the real-world actions of our patients. Prior studies used placebo inhalers to keep participants and providers blinded but then could not capitalize on the behavior of using only an inhaler prn (as with the ICS/LABA of this study). Finally, there is discordance between the NIH and GINA asthma guidelines on how to use these data.

CHALLENGES TO IMPLEMENTATION

Cost of ICS/LABA may limit its use

Cost is the largest barrier to implementation. Budesonide costs 6 to 10 times more than albuterol per inhaler (retail price of $281-$427 vs $17-$92, respectively).7,8 However, cost differences are likely negated for patients already on a maintenance inhaler.

 

ACKNOWLEDGEMENT

The PURLs Surveillance System was supported in part by Grant Number UL1RR024999 from the National Center For Research Resources, a Clinical Translational Science Award to the University of Chicago. The content is solely the responsibility of the authors and does not necessarily represent the official views of the National Center For Research Resources or the National Institutes of Health.

Files
References

1. Hardy J, Baggott C, Fingleton J, et al; PRACTICAL study team. Budesonide-formoterol reliever therapy versus maintenance budesonide plus terbutaline reliever therapy in adults with mild to moderate asthma (PRACTICAL): a 52-week, open-label, multicentre, superiority, randomised controlled trial. Lancet. 2019;394:919-928. Published correction appears in Lancet. 2020;395:1422.

2. Centers for Disease Control and Prevention. Summary Health Statistics: National Health Interview Survey, 2018. Accessed February 17, 2021. https://ftp.cdc.gov/pub/Health_Statistics/NCHS/NHIS/SHS/2018_SHS_Table_A-2.pdf

3. National Institutes of Health. National Heart, Lung, and Blood Institute. 2020 Focused Updates to the Asthma Management Guidelines: A Report from the National Asthma Education and Prevention Program Coordinating Committee Expert Panel Working Group. December 2020. Accessed February 17, 2021. www.nhlbi.nih.gov/health-topics/all-publications-and-resources/2020-focused-updates-asthma-management-guidelines

4. Global Initiative for Asthma (GINA). Global Strategy for Asthma Management and Prevention, 2020. Accessed February 17, 2021. www.ginasthma.org/

5. Sobieraj DM, Weeda ER, Nguyen E, et al. Association of inhaled corticosteroids and long-acting β-agonists as controller and quick relief therapy with exacerbations and symptom control in persistent asthma: a systematic review and meta-analysis. JAMA. 2018;319:1485-1496.

6. Beasley R, Holliday M, Reddel HK, et al; Novel START Study Team. Controlled trial of budesonide-formoterol as needed for mild asthma. N Engl J Med. 2019;380:2020-2030.

7. Albuterol. GoodRx. Accessed February 17, 2021. www.goodrx.com/albuterol

8. Budesonide/formoterol. GoodRx. Accessed February 17, 2021. www.goodrx.com/budesonide-formoterol

Article PDF
Author and Disclosure Information

UPMC St. Margaret, Pittsburgh, PA

DEPUTY EDITOR
Shailendra Prasad, MBBS, MPH

University of Minnesota North Memorial Family Medicine Residency Program, Minneapolis, MN

Issue
The Journal of Family Practice - 70(2)
Publications
Topics
Page Number
90-92
Sections
Files
Files
Author and Disclosure Information

UPMC St. Margaret, Pittsburgh, PA

DEPUTY EDITOR
Shailendra Prasad, MBBS, MPH

University of Minnesota North Memorial Family Medicine Residency Program, Minneapolis, MN

Author and Disclosure Information

UPMC St. Margaret, Pittsburgh, PA

DEPUTY EDITOR
Shailendra Prasad, MBBS, MPH

University of Minnesota North Memorial Family Medicine Residency Program, Minneapolis, MN

Article PDF
Article PDF

ILLUSTRATIVE CASE

A 37-year-old woman with moderate persistent asthma, controlled on the ICS fluticasone (110 μg twice a day) presents to you for an annual exam. She uses her rescue albuterol inhaler a few times per month. Her last exacerbation was 2 years ago. She has never smoked. She is concerned about continuing to take an ICS every day. What alternative regimen would you recommend for this patient?

According to the Centers for Disease Control and Prevention, asthma affected 24.7 million children and adults in the United States in 2018, accounting for 9.8 million physician visits and 1.6 million emergency department (ED) visits.2 The National Institutes of Health (NIH) asthma care guidelines, updated in 2020, recommend a SABA prn as step 1 for intermittent asthma, along with nonpharmacologic management.3 Once a patient has persistent asthma, treatment escalation to step 2 calls for use of daily maintenance inhalers as the preferred treatment option.3

However, the 2020 Global Initiative for Asthma (GINA) warns that an as-needed SABA does not protect patients from severe exacerbations, and regular use of a SABA alone (> 3 inhalers/year) can increase the risk of exacerbations.4 A meta-analysis and systematic review from 2018 showed that using an ICS/LABA—scheduled and prn for rescue—had lower risk of asthma exacerbations compared with scheduled ICS/LABA with SABA prn for rescue in patients with ­moderate-to-severe persistent asthma.5 Interestingly, the updated 2020 NIH guidelines have adopted this strategy. SABA use prn is no longer recommended for rescue in mild and moderate persistent asthma, and the guidelines now suggest that ICS/LABA be used as rescue in addition to daily medication.3

Although evidence has been mounting for adding the as-needed ICS/LABA for rescue in patients on daily medication, the mainstay has been to provide a SABA prn for rescue use.5 Confusing matters more, evidence is emerging that as-needed ICS/LABA for rescue alone in certain patients is safe and effective. The randomized controlled Novel START study, an open-label, parallel-group study, compared ICS/LABA prn vs scheduled ICS with SABA prn vs SABA alone prn in adult patients with intermittent or mild persistent asthma.6 ICS/LABA prn prevented more exacerbations and provided better daily control than as-needed SABA alone.6 In addition, ICS/LABA as needed resulted in fewer severe exacerbations but potentially poorer daily control than ICS with SABA as needed.6

The PRACTICAL study investigated treatment of patients with intermittent, mild persistent, and moderate persistent asthma.1

STUDY SUMMARY

ICS/LABA prn reduced risk of severe exacerbations

The randomized controlled PRACTICAL study was a 52-week, open-label, parallel-group, superiority trial in New Zealand that compared as-needed ICS/LABA (n = 437) to scheduled ICS plus as-needed SABA (n = 448). Patients were 18 to 75 years old, with a diagnosis of asthma. Applying NIH guideline definitions, these patients would fall into intermittent, mild persistent, or moderate persistent asthma categories, and were on either as-needed SABA alone or a scheduled low- to moderate-dose ICS plus an as-needed SABA in the previous 12 weeks.

Patients on an as-needed SABA prerandomization had to have at least 1 of the following: (1) asthma symptoms or need for a SABA at least twice in the past 4 weeks; (2) at least 1 nighttime awakening due to asthma in the past 4 weeks; or (3) a severe exacerbation requiring oral corticosteroids in the past year. Patients on scheduled ICS plus SABA prn prerandomization were required to have either: (1) low or moderate ICS dosing with partly or well-controlled asthma; or (2) if uncontrolled, poor inhaler technique or adherence.

Continue to: Patients in the ICS/LABA group...

 

 

Patients in the ICS/LABA group were given budesonide 200 µg/formoterol 6 µg, 1 puff prn, and patients in the ICS plus as-needed SABA group were given budesonide 200 µg, 1 puff twice daily, and terbutaline 250 µg, 2 puffs prn. All patients received an asthma action plan that provided guidance on when to seek medical care if asthma worsened, as well as a log to note urgent medical visits and use of systemic corticosteroids. A subset of patients had adherence and dosing monitored by electronic inhaler usage monitors. Patients were seen at 0, 4, 16, 28, 40, and 52 weeks.

Outcomes. The primary outcome was the number of severe exacerbations per patient per year, defined as treatment with oral corticosteroids for ≥ 3 days or ED visit or hospital admission requiring systemic corticosteroids. Among the secondary outcomes were number of moderate and severe exacerbations per patient per year (defined as an unplanned medical visit: primary care, ED, hospital admission, and any duration of steroids); time to first severe exacerbation; assessment with the Asthma Control Questionnaire (ACQ-5); adverse outcomes; and quantity of ICS used (analysis done only for the subset with electronic inhaler monitoring).

This study represents a compelling, real-world look at emerging asthma recommendations.

ACQ-5 takes the mean of 5 questions assessing asthma control in the previous week, with each question ranging from 0 (no impairment) to 6 (maximum impairment). The statistician was blinded to the primary outcome.

 

Results. The rate of severe exacerbations per patient per year was 0.119 in the as-needed ICS/LABA group vs 0.172 in the scheduled ICS plus as-needed SABA group (relative rate [RR] = 0.69; 95% confidence interval [CI], 0.48–1.00). Time to first severe asthma exacerbation was longer in the as-needed ICS/LABA group (hazard ratio = 0.60; 95% CI, 0.40–0.91). The rate of moderate and severe exacerbations per patient per year was lower in the as-needed ICS/LABA group: 0.165 vs 0.237 (RR = 0.70; 95% CI, 0.51–0.95).

ACQ-5 scores were similar at all time points (mean difference = 0.07; 95% CI, –0.03 to 0.17). Adverse events were similar between groups (most commonly nasopharyngitis in both groups). Less ICS was used in the ICS/LABA group (difference = –126.5 µg per day; 95% CI, –171.0 to –81.9).

Continue to: WHAT'S NEW

 

 

WHAT’S NEW

Study lends support to recent recommendations

This study represents a compelling, real-world look at emerging asthma recommendations. This was the first comprehensive study to show that as-needed ICS/LABA therapy prevents more moderate and severe exacerbations and lengthens the time to first severe exacerbation, compared with scheduled ICS plus SABA prn in intermittent, mild persistent, or moderate persistent asthma. These data have been incorporated into the GINA guidelines, which recommend ICS/LABA prn for step 2.

CAVEATS

Potential bias in study design

The LABA used in this study was formoterol, which has a quicker onset than other LABAs. It is likely that not all LABAs can be used the same way, and both the NIH and GINA guidelines call it out specifically. Additionally, the study’s open-label design can introduce bias but may be the only way to simulate the real-world actions of our patients. Prior studies used placebo inhalers to keep participants and providers blinded but then could not capitalize on the behavior of using only an inhaler prn (as with the ICS/LABA of this study). Finally, there is discordance between the NIH and GINA asthma guidelines on how to use these data.

CHALLENGES TO IMPLEMENTATION

Cost of ICS/LABA may limit its use

Cost is the largest barrier to implementation. Budesonide costs 6 to 10 times more than albuterol per inhaler (retail price of $281-$427 vs $17-$92, respectively).7,8 However, cost differences are likely negated for patients already on a maintenance inhaler.

 

ACKNOWLEDGEMENT

The PURLs Surveillance System was supported in part by Grant Number UL1RR024999 from the National Center For Research Resources, a Clinical Translational Science Award to the University of Chicago. The content is solely the responsibility of the authors and does not necessarily represent the official views of the National Center For Research Resources or the National Institutes of Health.

ILLUSTRATIVE CASE

A 37-year-old woman with moderate persistent asthma, controlled on the ICS fluticasone (110 μg twice a day) presents to you for an annual exam. She uses her rescue albuterol inhaler a few times per month. Her last exacerbation was 2 years ago. She has never smoked. She is concerned about continuing to take an ICS every day. What alternative regimen would you recommend for this patient?

According to the Centers for Disease Control and Prevention, asthma affected 24.7 million children and adults in the United States in 2018, accounting for 9.8 million physician visits and 1.6 million emergency department (ED) visits.2 The National Institutes of Health (NIH) asthma care guidelines, updated in 2020, recommend a SABA prn as step 1 for intermittent asthma, along with nonpharmacologic management.3 Once a patient has persistent asthma, treatment escalation to step 2 calls for use of daily maintenance inhalers as the preferred treatment option.3

However, the 2020 Global Initiative for Asthma (GINA) warns that an as-needed SABA does not protect patients from severe exacerbations, and regular use of a SABA alone (> 3 inhalers/year) can increase the risk of exacerbations.4 A meta-analysis and systematic review from 2018 showed that using an ICS/LABA—scheduled and prn for rescue—had lower risk of asthma exacerbations compared with scheduled ICS/LABA with SABA prn for rescue in patients with ­moderate-to-severe persistent asthma.5 Interestingly, the updated 2020 NIH guidelines have adopted this strategy. SABA use prn is no longer recommended for rescue in mild and moderate persistent asthma, and the guidelines now suggest that ICS/LABA be used as rescue in addition to daily medication.3

Although evidence has been mounting for adding the as-needed ICS/LABA for rescue in patients on daily medication, the mainstay has been to provide a SABA prn for rescue use.5 Confusing matters more, evidence is emerging that as-needed ICS/LABA for rescue alone in certain patients is safe and effective. The randomized controlled Novel START study, an open-label, parallel-group study, compared ICS/LABA prn vs scheduled ICS with SABA prn vs SABA alone prn in adult patients with intermittent or mild persistent asthma.6 ICS/LABA prn prevented more exacerbations and provided better daily control than as-needed SABA alone.6 In addition, ICS/LABA as needed resulted in fewer severe exacerbations but potentially poorer daily control than ICS with SABA as needed.6

The PRACTICAL study investigated treatment of patients with intermittent, mild persistent, and moderate persistent asthma.1

STUDY SUMMARY

ICS/LABA prn reduced risk of severe exacerbations

The randomized controlled PRACTICAL study was a 52-week, open-label, parallel-group, superiority trial in New Zealand that compared as-needed ICS/LABA (n = 437) to scheduled ICS plus as-needed SABA (n = 448). Patients were 18 to 75 years old, with a diagnosis of asthma. Applying NIH guideline definitions, these patients would fall into intermittent, mild persistent, or moderate persistent asthma categories, and were on either as-needed SABA alone or a scheduled low- to moderate-dose ICS plus an as-needed SABA in the previous 12 weeks.

Patients on an as-needed SABA prerandomization had to have at least 1 of the following: (1) asthma symptoms or need for a SABA at least twice in the past 4 weeks; (2) at least 1 nighttime awakening due to asthma in the past 4 weeks; or (3) a severe exacerbation requiring oral corticosteroids in the past year. Patients on scheduled ICS plus SABA prn prerandomization were required to have either: (1) low or moderate ICS dosing with partly or well-controlled asthma; or (2) if uncontrolled, poor inhaler technique or adherence.

Continue to: Patients in the ICS/LABA group...

 

 

Patients in the ICS/LABA group were given budesonide 200 µg/formoterol 6 µg, 1 puff prn, and patients in the ICS plus as-needed SABA group were given budesonide 200 µg, 1 puff twice daily, and terbutaline 250 µg, 2 puffs prn. All patients received an asthma action plan that provided guidance on when to seek medical care if asthma worsened, as well as a log to note urgent medical visits and use of systemic corticosteroids. A subset of patients had adherence and dosing monitored by electronic inhaler usage monitors. Patients were seen at 0, 4, 16, 28, 40, and 52 weeks.

Outcomes. The primary outcome was the number of severe exacerbations per patient per year, defined as treatment with oral corticosteroids for ≥ 3 days or ED visit or hospital admission requiring systemic corticosteroids. Among the secondary outcomes were number of moderate and severe exacerbations per patient per year (defined as an unplanned medical visit: primary care, ED, hospital admission, and any duration of steroids); time to first severe exacerbation; assessment with the Asthma Control Questionnaire (ACQ-5); adverse outcomes; and quantity of ICS used (analysis done only for the subset with electronic inhaler monitoring).

This study represents a compelling, real-world look at emerging asthma recommendations.

ACQ-5 takes the mean of 5 questions assessing asthma control in the previous week, with each question ranging from 0 (no impairment) to 6 (maximum impairment). The statistician was blinded to the primary outcome.

 

Results. The rate of severe exacerbations per patient per year was 0.119 in the as-needed ICS/LABA group vs 0.172 in the scheduled ICS plus as-needed SABA group (relative rate [RR] = 0.69; 95% confidence interval [CI], 0.48–1.00). Time to first severe asthma exacerbation was longer in the as-needed ICS/LABA group (hazard ratio = 0.60; 95% CI, 0.40–0.91). The rate of moderate and severe exacerbations per patient per year was lower in the as-needed ICS/LABA group: 0.165 vs 0.237 (RR = 0.70; 95% CI, 0.51–0.95).

ACQ-5 scores were similar at all time points (mean difference = 0.07; 95% CI, –0.03 to 0.17). Adverse events were similar between groups (most commonly nasopharyngitis in both groups). Less ICS was used in the ICS/LABA group (difference = –126.5 µg per day; 95% CI, –171.0 to –81.9).

Continue to: WHAT'S NEW

 

 

WHAT’S NEW

Study lends support to recent recommendations

This study represents a compelling, real-world look at emerging asthma recommendations. This was the first comprehensive study to show that as-needed ICS/LABA therapy prevents more moderate and severe exacerbations and lengthens the time to first severe exacerbation, compared with scheduled ICS plus SABA prn in intermittent, mild persistent, or moderate persistent asthma. These data have been incorporated into the GINA guidelines, which recommend ICS/LABA prn for step 2.

CAVEATS

Potential bias in study design

The LABA used in this study was formoterol, which has a quicker onset than other LABAs. It is likely that not all LABAs can be used the same way, and both the NIH and GINA guidelines call it out specifically. Additionally, the study’s open-label design can introduce bias but may be the only way to simulate the real-world actions of our patients. Prior studies used placebo inhalers to keep participants and providers blinded but then could not capitalize on the behavior of using only an inhaler prn (as with the ICS/LABA of this study). Finally, there is discordance between the NIH and GINA asthma guidelines on how to use these data.

CHALLENGES TO IMPLEMENTATION

Cost of ICS/LABA may limit its use

Cost is the largest barrier to implementation. Budesonide costs 6 to 10 times more than albuterol per inhaler (retail price of $281-$427 vs $17-$92, respectively).7,8 However, cost differences are likely negated for patients already on a maintenance inhaler.

 

ACKNOWLEDGEMENT

The PURLs Surveillance System was supported in part by Grant Number UL1RR024999 from the National Center For Research Resources, a Clinical Translational Science Award to the University of Chicago. The content is solely the responsibility of the authors and does not necessarily represent the official views of the National Center For Research Resources or the National Institutes of Health.

References

1. Hardy J, Baggott C, Fingleton J, et al; PRACTICAL study team. Budesonide-formoterol reliever therapy versus maintenance budesonide plus terbutaline reliever therapy in adults with mild to moderate asthma (PRACTICAL): a 52-week, open-label, multicentre, superiority, randomised controlled trial. Lancet. 2019;394:919-928. Published correction appears in Lancet. 2020;395:1422.

2. Centers for Disease Control and Prevention. Summary Health Statistics: National Health Interview Survey, 2018. Accessed February 17, 2021. https://ftp.cdc.gov/pub/Health_Statistics/NCHS/NHIS/SHS/2018_SHS_Table_A-2.pdf

3. National Institutes of Health. National Heart, Lung, and Blood Institute. 2020 Focused Updates to the Asthma Management Guidelines: A Report from the National Asthma Education and Prevention Program Coordinating Committee Expert Panel Working Group. December 2020. Accessed February 17, 2021. www.nhlbi.nih.gov/health-topics/all-publications-and-resources/2020-focused-updates-asthma-management-guidelines

4. Global Initiative for Asthma (GINA). Global Strategy for Asthma Management and Prevention, 2020. Accessed February 17, 2021. www.ginasthma.org/

5. Sobieraj DM, Weeda ER, Nguyen E, et al. Association of inhaled corticosteroids and long-acting β-agonists as controller and quick relief therapy with exacerbations and symptom control in persistent asthma: a systematic review and meta-analysis. JAMA. 2018;319:1485-1496.

6. Beasley R, Holliday M, Reddel HK, et al; Novel START Study Team. Controlled trial of budesonide-formoterol as needed for mild asthma. N Engl J Med. 2019;380:2020-2030.

7. Albuterol. GoodRx. Accessed February 17, 2021. www.goodrx.com/albuterol

8. Budesonide/formoterol. GoodRx. Accessed February 17, 2021. www.goodrx.com/budesonide-formoterol

References

1. Hardy J, Baggott C, Fingleton J, et al; PRACTICAL study team. Budesonide-formoterol reliever therapy versus maintenance budesonide plus terbutaline reliever therapy in adults with mild to moderate asthma (PRACTICAL): a 52-week, open-label, multicentre, superiority, randomised controlled trial. Lancet. 2019;394:919-928. Published correction appears in Lancet. 2020;395:1422.

2. Centers for Disease Control and Prevention. Summary Health Statistics: National Health Interview Survey, 2018. Accessed February 17, 2021. https://ftp.cdc.gov/pub/Health_Statistics/NCHS/NHIS/SHS/2018_SHS_Table_A-2.pdf

3. National Institutes of Health. National Heart, Lung, and Blood Institute. 2020 Focused Updates to the Asthma Management Guidelines: A Report from the National Asthma Education and Prevention Program Coordinating Committee Expert Panel Working Group. December 2020. Accessed February 17, 2021. www.nhlbi.nih.gov/health-topics/all-publications-and-resources/2020-focused-updates-asthma-management-guidelines

4. Global Initiative for Asthma (GINA). Global Strategy for Asthma Management and Prevention, 2020. Accessed February 17, 2021. www.ginasthma.org/

5. Sobieraj DM, Weeda ER, Nguyen E, et al. Association of inhaled corticosteroids and long-acting β-agonists as controller and quick relief therapy with exacerbations and symptom control in persistent asthma: a systematic review and meta-analysis. JAMA. 2018;319:1485-1496.

6. Beasley R, Holliday M, Reddel HK, et al; Novel START Study Team. Controlled trial of budesonide-formoterol as needed for mild asthma. N Engl J Med. 2019;380:2020-2030.

7. Albuterol. GoodRx. Accessed February 17, 2021. www.goodrx.com/albuterol

8. Budesonide/formoterol. GoodRx. Accessed February 17, 2021. www.goodrx.com/budesonide-formoterol

Issue
The Journal of Family Practice - 70(2)
Issue
The Journal of Family Practice - 70(2)
Page Number
90-92
Page Number
90-92
Publications
Publications
Topics
Article Type
Display Headline
An alternative regimen to reduce risk of asthma exacerbations
Display Headline
An alternative regimen to reduce risk of asthma exacerbations
Sections
PURLs Copyright
Copyright © 2021. The Family Physicians Inquiries Network. All rights reserved.
Inside the Article

PRACTICE CHANGER

Use an inhaled corticosteroid plus long-­acting beta-agonist (ICS/LABA) prn for intermittent, mild persistent, or moderate persistent asthma for fewer moderate and severe exacerbations and the same daily symptom control as scheduled ICS with a short-acting beta-agonist (SABA) prn.1

STRENGTH OF RECOMMENDATION

A: Based on a single, good-quality, multicenter, randomized controlled trial.1

Hardy J, Baggott C, Fingleton J, et al; PRACTICAL study team. Budesonide-formoterol reliever therapy versus maintenance budesonide plus terbutaline reliever therapy in adults with mild to moderate asthma (PRACTICAL): a 52-week, open-label, multicentre, superiority, randomised controlled trial. Lancet. 2019;394:919-928. Published correction appears in Lancet. 2020;395:1422.1

Disallow All Ads
Content Gating
No Gating (article Unlocked/Free)
Alternative CME
Disqus Comments
Default
Use ProPublica
Hide sidebar & use full width
render the right sidebar.
Conference Recap Checkbox
Not Conference Recap
Clinical Edge
Display the Slideshow in this Article
Medscape Article
Display survey writer
Reuters content
Article PDF Media
Media Files

A better approach to preventing active TB?

Article Type
Changed
Tue, 03/03/2020 - 15:13
Display Headline
A better approach to preventing active TB?

ILLUSTRATIVE CASE

A 27-year-old daycare worker was tested for tuberculosis (TB) as part of a recent work physical. She presents to your office for follow-up for her positive purified protein derivative (PPD) skin test. You confirm the result with a quantiferon gold test and ensure she does not have active TB. What medication should you prescribe to treat her latent TB infection (LTBI)?

In 2017, there were 9093 cases of new active TB in the United States.2 It’s estimated that one-fourth of the world’s population has latent TB.3 Identifying and treating latent TB ­infection is vital to achieving TB’s elimination.4,5

Primary care clinicians are at the forefront of screening high-risk populations for TB. Once identified, treating LTBI can be challenging for providers and patients. Treatment guidelines recommend 4 to 9 months of daily isoniazid.5-8 Shorter treatment regimens were recommended previously; they tended to be rigorous, to involve multiple drugs, and to require high adherence rates. As such, they included directly observed therapy, which prevented widespread adoption.

Consequently, the mainstay for treating LTBI has been 9 months of daily isoniazid. However, isoniazid use is limited by hepatoxicity and by suboptimal treatment completion rates. A 2018 retrospective analysis of patients treated for LTBI reported a completion rate of only 49% for 9 months of isoniazid.9 Additionally, a Cochrane review last updated in 2013 suggests that shorter courses of rifampin are similar in efficacy to isoniazid (although with a wide confidence interval [CI]), and likely have higher adherence rates.10

STUDY SUMMARY

Rifampin is as effective as isoniazid with fewer adverse effects

The study by Menzies et al1 was a multisite, 9-country, open-label, randomized controlled trial (RCT) that compared 4 months of daily rifampin to 9 months of daily isoniazid for the treatment of LTBI in adults. Participants were eligible if they had a positive tuberculin skin test or interferon-gamma-release assay, were ≥ 18 years of age, had an increased risk for reactivation of active TB, and if their health care provider had recommended treatment with isoniazid. Exclusion criteria included current pregnancy or plans to become pregnant, exposure to a patient with TB whose isolates were resistant to either trial drug, an allergy to either of the trial drugs, use of a medication with serious potential interactions with the trial drugs, or current active TB.

Method, outcomes, patient characteristics. Patients received either isoniazid 5 mg/kg body weight (maximum dose 300 mg) daily for 9 months or rifampin 10 mg/kg (maximum dose 600 mg) daily for 4 months and were followed for 28 months. Patients in the isoniazid group also received pyridoxine (vitamin B6) if they were at risk for neuropathy. The primary outcome was the rate of active TB. Secondary outcomes included adverse events, medication regimen completion rate, and drug resistance, among others.

This study found that a shorter rifampinbased regimen is associated with improved adherence and fewer adverse events than a longer isoniazid-based regimen for the treatment of latent TB infection.

A total of 2989 patients were treated with isoniazid; 3023 patients were treated with rifampin. The mean age of the participants was 38.4 years, 41% of the population was male, and 71% of the groups had confirmed active TB in close contacts.

Continue to: Results

 

 

Results. Overall, rates of active TB were low with 9 cases in the isoniazid group and 8 in the rifampin group. In the ­intention-to-treat analysis, the rate difference for confirmed active TB was < 0.01 cases per 100 person-years (95% CI; −0.14 to 0.16). This met the prespecified noninferiority endpoint, but did not show superiority. A total of 79% of patients treated with rifampin vs 63% treated with isoniazid completed their respective medication courses (difference of 15.1 percentage points; 95% CI, 12.7-17.4; P < .001). Compared with patients in the isoniazid group, those taking rifampin had fewer adverse events, leading to discontinuation (5.6% vs 2.8%).

WHAT’S NEW?

First high-quality study to show that less is more

This is the first large, high-quality study to show that a shorter (4 month) rifampin-based regimen is not inferior to a longer (9 months) isoniazid-based regimen for the treatment of LTBI, and that rifampin is associated with improved adherence and fewer adverse events.

CAVEATS

Low rate of active TB infection and potential bias

The current study had lower-than-­anticipated rates of active TB infection, which made the study’s conclusions less compelling. This may have been because of a small number of patients with human immunodeficiency virus enrolled in the study and/or that even participants who discontinued treatment received a median of 3 months of partial treatment.

In addition, the study was an open-label RCT, subjecting it to potential bias. However, the diagnosis of active TB and attribution of adverse events were made by an independent, blinded review panel.

CHALLENGES TO IMPLEMENTATION

No challenges to speak of

We see no challenges to implementing this recommendation.

ACKNOWLEDGEMENT

The PURLs Surveillance System was supported in part by Grant Number UL1RR024999 from the National Center For Research Resources, a Clinical Translational Science Award to the University of Chicago. The content is solely the responsibility of the authors and does not necessarily represent the official views of the National Center For Research Resources or the National Institutes of Health.

Files
References

1. Menzies D, Adjobimey M, Ruslami R, et al. Four months of rifampin or nine months of isoniazid for latent tuberculosis in adults. N Engl J Med. 2018;379:440-453.

2. Stewart RJ, Tsang CA, Pratt RH, et al. Tuberculosis — United States, 2017. MMWR Morb Mortal Wkly Rep. 2018;67:317-323.

3. Houben RM, Dodd PJ. The global burden of latent tuberculosis infection: a re-estimation using mathematical modeling. PLoS Med. 2016;13:e1002152.

4. Lönnroth K, Migliori GB, Abubakar I, et al. Towards tuberculosis elimination: an action framework for low-incidence countries. Eur Respir J. 2015;45:928-952.

5. Uplekar M, Weil D, Lonnroth K, et al. WHO’s new end TB strategy. Lancet. 2015;385:1799-1801.

6. Centers for Disease Control and Prevention. Treatment regimens for latent TB infection (LTBI). Last reviewed April 5, 2016. https://www.cdc.gov/tb/topic/treatment/ltbi.htm. Accessed January 15, 2020.

7. World Health Organization. Latent TB infection: updated and consolidated guidelines for programmatic management. 2018. Publication no. WHO/CDS/TB/2018.4. https://www.who.int/tb/publications/2018/latent-tuberculosis-infection/en/. Accessed January 15, 2020.

8. Borisov AS, Bamrah Morris S, Njie GJ, et al. Update of recommendations for use of once-weekly isoniazid-rifapentine regimen to treat latent Mycobacterium tuberculosis infection. MMWR Morb Mortal Wkly Rep. 2018;67:723-726.

9. Macaraig MM, Jalees M, Lam C, et al. Improved treatment completion with shorter treatment regimens for latent tuberculous infection. Int J Tuber Lung Dis. 2018;22:1344-1349. 10. Sharma SK, Sharma A, Kadhiravan T, et al. Rifamycins (rifampicin, rifabutin and rifapentine) compared to isoniazid for preventing tuberculosis in HIV-negative people at risk of active TB. Cochrane Database Syst Rev. 2013;(7):CD007545.

Article PDF
Author and Disclosure Information

UPMC St. Margaret Family Medicine Residency, Pittsburgh, PA

DEPUTY EDITOR
Shailey Prasad, MBBS, MPH

University of Minnesota, Department of Family Medicine and Community Health, Minneapolis

Issue
The Journal of Family Practice - 69(1)
Publications
Topics
Page Number
37-38
Sections
Files
Files
Author and Disclosure Information

UPMC St. Margaret Family Medicine Residency, Pittsburgh, PA

DEPUTY EDITOR
Shailey Prasad, MBBS, MPH

University of Minnesota, Department of Family Medicine and Community Health, Minneapolis

Author and Disclosure Information

UPMC St. Margaret Family Medicine Residency, Pittsburgh, PA

DEPUTY EDITOR
Shailey Prasad, MBBS, MPH

University of Minnesota, Department of Family Medicine and Community Health, Minneapolis

Article PDF
Article PDF

ILLUSTRATIVE CASE

A 27-year-old daycare worker was tested for tuberculosis (TB) as part of a recent work physical. She presents to your office for follow-up for her positive purified protein derivative (PPD) skin test. You confirm the result with a quantiferon gold test and ensure she does not have active TB. What medication should you prescribe to treat her latent TB infection (LTBI)?

In 2017, there were 9093 cases of new active TB in the United States.2 It’s estimated that one-fourth of the world’s population has latent TB.3 Identifying and treating latent TB ­infection is vital to achieving TB’s elimination.4,5

Primary care clinicians are at the forefront of screening high-risk populations for TB. Once identified, treating LTBI can be challenging for providers and patients. Treatment guidelines recommend 4 to 9 months of daily isoniazid.5-8 Shorter treatment regimens were recommended previously; they tended to be rigorous, to involve multiple drugs, and to require high adherence rates. As such, they included directly observed therapy, which prevented widespread adoption.

Consequently, the mainstay for treating LTBI has been 9 months of daily isoniazid. However, isoniazid use is limited by hepatoxicity and by suboptimal treatment completion rates. A 2018 retrospective analysis of patients treated for LTBI reported a completion rate of only 49% for 9 months of isoniazid.9 Additionally, a Cochrane review last updated in 2013 suggests that shorter courses of rifampin are similar in efficacy to isoniazid (although with a wide confidence interval [CI]), and likely have higher adherence rates.10

STUDY SUMMARY

Rifampin is as effective as isoniazid with fewer adverse effects

The study by Menzies et al1 was a multisite, 9-country, open-label, randomized controlled trial (RCT) that compared 4 months of daily rifampin to 9 months of daily isoniazid for the treatment of LTBI in adults. Participants were eligible if they had a positive tuberculin skin test or interferon-gamma-release assay, were ≥ 18 years of age, had an increased risk for reactivation of active TB, and if their health care provider had recommended treatment with isoniazid. Exclusion criteria included current pregnancy or plans to become pregnant, exposure to a patient with TB whose isolates were resistant to either trial drug, an allergy to either of the trial drugs, use of a medication with serious potential interactions with the trial drugs, or current active TB.

Method, outcomes, patient characteristics. Patients received either isoniazid 5 mg/kg body weight (maximum dose 300 mg) daily for 9 months or rifampin 10 mg/kg (maximum dose 600 mg) daily for 4 months and were followed for 28 months. Patients in the isoniazid group also received pyridoxine (vitamin B6) if they were at risk for neuropathy. The primary outcome was the rate of active TB. Secondary outcomes included adverse events, medication regimen completion rate, and drug resistance, among others.

This study found that a shorter rifampinbased regimen is associated with improved adherence and fewer adverse events than a longer isoniazid-based regimen for the treatment of latent TB infection.

A total of 2989 patients were treated with isoniazid; 3023 patients were treated with rifampin. The mean age of the participants was 38.4 years, 41% of the population was male, and 71% of the groups had confirmed active TB in close contacts.

Continue to: Results

 

 

Results. Overall, rates of active TB were low with 9 cases in the isoniazid group and 8 in the rifampin group. In the ­intention-to-treat analysis, the rate difference for confirmed active TB was < 0.01 cases per 100 person-years (95% CI; −0.14 to 0.16). This met the prespecified noninferiority endpoint, but did not show superiority. A total of 79% of patients treated with rifampin vs 63% treated with isoniazid completed their respective medication courses (difference of 15.1 percentage points; 95% CI, 12.7-17.4; P < .001). Compared with patients in the isoniazid group, those taking rifampin had fewer adverse events, leading to discontinuation (5.6% vs 2.8%).

WHAT’S NEW?

First high-quality study to show that less is more

This is the first large, high-quality study to show that a shorter (4 month) rifampin-based regimen is not inferior to a longer (9 months) isoniazid-based regimen for the treatment of LTBI, and that rifampin is associated with improved adherence and fewer adverse events.

CAVEATS

Low rate of active TB infection and potential bias

The current study had lower-than-­anticipated rates of active TB infection, which made the study’s conclusions less compelling. This may have been because of a small number of patients with human immunodeficiency virus enrolled in the study and/or that even participants who discontinued treatment received a median of 3 months of partial treatment.

In addition, the study was an open-label RCT, subjecting it to potential bias. However, the diagnosis of active TB and attribution of adverse events were made by an independent, blinded review panel.

CHALLENGES TO IMPLEMENTATION

No challenges to speak of

We see no challenges to implementing this recommendation.

ACKNOWLEDGEMENT

The PURLs Surveillance System was supported in part by Grant Number UL1RR024999 from the National Center For Research Resources, a Clinical Translational Science Award to the University of Chicago. The content is solely the responsibility of the authors and does not necessarily represent the official views of the National Center For Research Resources or the National Institutes of Health.

ILLUSTRATIVE CASE

A 27-year-old daycare worker was tested for tuberculosis (TB) as part of a recent work physical. She presents to your office for follow-up for her positive purified protein derivative (PPD) skin test. You confirm the result with a quantiferon gold test and ensure she does not have active TB. What medication should you prescribe to treat her latent TB infection (LTBI)?

In 2017, there were 9093 cases of new active TB in the United States.2 It’s estimated that one-fourth of the world’s population has latent TB.3 Identifying and treating latent TB ­infection is vital to achieving TB’s elimination.4,5

Primary care clinicians are at the forefront of screening high-risk populations for TB. Once identified, treating LTBI can be challenging for providers and patients. Treatment guidelines recommend 4 to 9 months of daily isoniazid.5-8 Shorter treatment regimens were recommended previously; they tended to be rigorous, to involve multiple drugs, and to require high adherence rates. As such, they included directly observed therapy, which prevented widespread adoption.

Consequently, the mainstay for treating LTBI has been 9 months of daily isoniazid. However, isoniazid use is limited by hepatoxicity and by suboptimal treatment completion rates. A 2018 retrospective analysis of patients treated for LTBI reported a completion rate of only 49% for 9 months of isoniazid.9 Additionally, a Cochrane review last updated in 2013 suggests that shorter courses of rifampin are similar in efficacy to isoniazid (although with a wide confidence interval [CI]), and likely have higher adherence rates.10

STUDY SUMMARY

Rifampin is as effective as isoniazid with fewer adverse effects

The study by Menzies et al1 was a multisite, 9-country, open-label, randomized controlled trial (RCT) that compared 4 months of daily rifampin to 9 months of daily isoniazid for the treatment of LTBI in adults. Participants were eligible if they had a positive tuberculin skin test or interferon-gamma-release assay, were ≥ 18 years of age, had an increased risk for reactivation of active TB, and if their health care provider had recommended treatment with isoniazid. Exclusion criteria included current pregnancy or plans to become pregnant, exposure to a patient with TB whose isolates were resistant to either trial drug, an allergy to either of the trial drugs, use of a medication with serious potential interactions with the trial drugs, or current active TB.

Method, outcomes, patient characteristics. Patients received either isoniazid 5 mg/kg body weight (maximum dose 300 mg) daily for 9 months or rifampin 10 mg/kg (maximum dose 600 mg) daily for 4 months and were followed for 28 months. Patients in the isoniazid group also received pyridoxine (vitamin B6) if they were at risk for neuropathy. The primary outcome was the rate of active TB. Secondary outcomes included adverse events, medication regimen completion rate, and drug resistance, among others.

This study found that a shorter rifampinbased regimen is associated with improved adherence and fewer adverse events than a longer isoniazid-based regimen for the treatment of latent TB infection.

A total of 2989 patients were treated with isoniazid; 3023 patients were treated with rifampin. The mean age of the participants was 38.4 years, 41% of the population was male, and 71% of the groups had confirmed active TB in close contacts.

Continue to: Results

 

 

Results. Overall, rates of active TB were low with 9 cases in the isoniazid group and 8 in the rifampin group. In the ­intention-to-treat analysis, the rate difference for confirmed active TB was < 0.01 cases per 100 person-years (95% CI; −0.14 to 0.16). This met the prespecified noninferiority endpoint, but did not show superiority. A total of 79% of patients treated with rifampin vs 63% treated with isoniazid completed their respective medication courses (difference of 15.1 percentage points; 95% CI, 12.7-17.4; P < .001). Compared with patients in the isoniazid group, those taking rifampin had fewer adverse events, leading to discontinuation (5.6% vs 2.8%).

WHAT’S NEW?

First high-quality study to show that less is more

This is the first large, high-quality study to show that a shorter (4 month) rifampin-based regimen is not inferior to a longer (9 months) isoniazid-based regimen for the treatment of LTBI, and that rifampin is associated with improved adherence and fewer adverse events.

CAVEATS

Low rate of active TB infection and potential bias

The current study had lower-than-­anticipated rates of active TB infection, which made the study’s conclusions less compelling. This may have been because of a small number of patients with human immunodeficiency virus enrolled in the study and/or that even participants who discontinued treatment received a median of 3 months of partial treatment.

In addition, the study was an open-label RCT, subjecting it to potential bias. However, the diagnosis of active TB and attribution of adverse events were made by an independent, blinded review panel.

CHALLENGES TO IMPLEMENTATION

No challenges to speak of

We see no challenges to implementing this recommendation.

ACKNOWLEDGEMENT

The PURLs Surveillance System was supported in part by Grant Number UL1RR024999 from the National Center For Research Resources, a Clinical Translational Science Award to the University of Chicago. The content is solely the responsibility of the authors and does not necessarily represent the official views of the National Center For Research Resources or the National Institutes of Health.

References

1. Menzies D, Adjobimey M, Ruslami R, et al. Four months of rifampin or nine months of isoniazid for latent tuberculosis in adults. N Engl J Med. 2018;379:440-453.

2. Stewart RJ, Tsang CA, Pratt RH, et al. Tuberculosis — United States, 2017. MMWR Morb Mortal Wkly Rep. 2018;67:317-323.

3. Houben RM, Dodd PJ. The global burden of latent tuberculosis infection: a re-estimation using mathematical modeling. PLoS Med. 2016;13:e1002152.

4. Lönnroth K, Migliori GB, Abubakar I, et al. Towards tuberculosis elimination: an action framework for low-incidence countries. Eur Respir J. 2015;45:928-952.

5. Uplekar M, Weil D, Lonnroth K, et al. WHO’s new end TB strategy. Lancet. 2015;385:1799-1801.

6. Centers for Disease Control and Prevention. Treatment regimens for latent TB infection (LTBI). Last reviewed April 5, 2016. https://www.cdc.gov/tb/topic/treatment/ltbi.htm. Accessed January 15, 2020.

7. World Health Organization. Latent TB infection: updated and consolidated guidelines for programmatic management. 2018. Publication no. WHO/CDS/TB/2018.4. https://www.who.int/tb/publications/2018/latent-tuberculosis-infection/en/. Accessed January 15, 2020.

8. Borisov AS, Bamrah Morris S, Njie GJ, et al. Update of recommendations for use of once-weekly isoniazid-rifapentine regimen to treat latent Mycobacterium tuberculosis infection. MMWR Morb Mortal Wkly Rep. 2018;67:723-726.

9. Macaraig MM, Jalees M, Lam C, et al. Improved treatment completion with shorter treatment regimens for latent tuberculous infection. Int J Tuber Lung Dis. 2018;22:1344-1349. 10. Sharma SK, Sharma A, Kadhiravan T, et al. Rifamycins (rifampicin, rifabutin and rifapentine) compared to isoniazid for preventing tuberculosis in HIV-negative people at risk of active TB. Cochrane Database Syst Rev. 2013;(7):CD007545.

References

1. Menzies D, Adjobimey M, Ruslami R, et al. Four months of rifampin or nine months of isoniazid for latent tuberculosis in adults. N Engl J Med. 2018;379:440-453.

2. Stewart RJ, Tsang CA, Pratt RH, et al. Tuberculosis — United States, 2017. MMWR Morb Mortal Wkly Rep. 2018;67:317-323.

3. Houben RM, Dodd PJ. The global burden of latent tuberculosis infection: a re-estimation using mathematical modeling. PLoS Med. 2016;13:e1002152.

4. Lönnroth K, Migliori GB, Abubakar I, et al. Towards tuberculosis elimination: an action framework for low-incidence countries. Eur Respir J. 2015;45:928-952.

5. Uplekar M, Weil D, Lonnroth K, et al. WHO’s new end TB strategy. Lancet. 2015;385:1799-1801.

6. Centers for Disease Control and Prevention. Treatment regimens for latent TB infection (LTBI). Last reviewed April 5, 2016. https://www.cdc.gov/tb/topic/treatment/ltbi.htm. Accessed January 15, 2020.

7. World Health Organization. Latent TB infection: updated and consolidated guidelines for programmatic management. 2018. Publication no. WHO/CDS/TB/2018.4. https://www.who.int/tb/publications/2018/latent-tuberculosis-infection/en/. Accessed January 15, 2020.

8. Borisov AS, Bamrah Morris S, Njie GJ, et al. Update of recommendations for use of once-weekly isoniazid-rifapentine regimen to treat latent Mycobacterium tuberculosis infection. MMWR Morb Mortal Wkly Rep. 2018;67:723-726.

9. Macaraig MM, Jalees M, Lam C, et al. Improved treatment completion with shorter treatment regimens for latent tuberculous infection. Int J Tuber Lung Dis. 2018;22:1344-1349. 10. Sharma SK, Sharma A, Kadhiravan T, et al. Rifamycins (rifampicin, rifabutin and rifapentine) compared to isoniazid for preventing tuberculosis in HIV-negative people at risk of active TB. Cochrane Database Syst Rev. 2013;(7):CD007545.

Issue
The Journal of Family Practice - 69(1)
Issue
The Journal of Family Practice - 69(1)
Page Number
37-38
Page Number
37-38
Publications
Publications
Topics
Article Type
Display Headline
A better approach to preventing active TB?
Display Headline
A better approach to preventing active TB?
Sections
PURLs Copyright
Copyright © 2020. The Family Physicians Inquiries Network. All rights reserved.
Inside the Article

PRACTICE CHANGER

Use 4 months of rifampin instead of 9 months of isoniazid to treat adults with latent tuberculosis; rifampin is associated with fewer adverse events and higher completion rates.

STRENGTH OF RECOMMENDATION

A: Based on a randomized controlled trial and a previous Cochrane review.

Menzies D, Adjobimey M, Ruslami R, et al. Four months of rifampin or nine months of isoniazid for latent tuberculosis in adults. N Engl J Med. 2018;379:440-453.

Disallow All Ads
Content Gating
No Gating (article Unlocked/Free)
Alternative CME
PubMed ID
32017832
Disqus Comments
Default
Use ProPublica
Hide sidebar & use full width
render the right sidebar.
Article PDF Media
Media Files

Antidepressant Tx for Anxiety Disorders: How Long?

Article Type
Changed
Tue, 12/03/2019 - 09:46
Display Headline
Antidepressant Tx for Anxiety Disorders: How Long?

PRACTICE CHANGER

Keep patients on antidepressant therapy for anxiety disorders for a year or longer before considering a taper.1

STRENGTH OF RECOMMENDATION

A: Based on a systematic review/meta-analysis of several good-quality randomized controlled trials.

A 42-year-old woman with generalized anxiety disorder (GAD) and panic attacks has been treated with sertraline (100 mg/d) for the past 8 months. She has also engaged in cognitive behavioral therapy (CBT) for 6 months. Her Generalized Anxiety Disorder-7 score has decreased from 19 prior to treatment to 5 at present. Now she would like to stop her antidepressant medication because she feels better. Would you recommend that she discontinue her medication at this point?

Anxiety disorders are common and often chronic and can cause significant morbidity and impairment.2,3 Firstline treatments for anxiety disorders include CBT and antidepressants, particularly selective serotonin reuptake inhibitors and serotonin-norepinephrine reuptake inhibitors.4-6

There is limited evidence regarding duration of antidepressant therapy for anxiety disorders. Previous studies have shown a high risk for relapse after discontinuation of antidepressants.6 A review of current practice patterns regarding pharmacologic treatment of depression and anxiety indicates an uptick in longer term antidepressant use for up to 2 years.7 However, long-term studies to guide treatment decisions are lacking.

STUDY SUMMARY

Clear benefit of continuing treatment

This systematic review and meta-analysis evaluated studies that looked at relapse rates and time to relapse in patients treated for anxiety disorders.1 The authors used PubMed, Cochrane, and Embase to identify studies involving patients treated for a variety of psychiatric disorders, including GAD, posttraumatic stress disorder (PTSD), panic disorder (PD), obsessive compulsive disorder (OCD), and social phobia. Eligible studies enrolled patients with anxiety disorders who had a positive response to an antidepressant and then randomized them in a double-blind fashion to either discontinuation of antidepressants and commencement of placebo (stopping group) or continuation of antidepressants (continuation group) for a duration of 8 to 52 weeks. The primary outcomes were relapse rate and time to relapse.

Twenty-eight studies met the inclusion criteria for the meta-analysis, with a total of 5233 patients (2625 patients in the continuation group and 2608 patients in the stopping group). A breakdown of the trials by indication included OCD (7), PD (6), GAD (6), social phobia (5), and PTSD (4). The authors graded the overall risk for bias to be low but noted that attrition bias was present in most studies.

Continue to: Results

 

 

Results. Relapse was more likely in the stopping group (odds ratio [OR], 3.11; n = 28 studies). Heterogeneity for relapse rate was low (I2 = 8.07%). Subgroup analyses by type of antidepressant, mode of discontinuation, and exclusion of patient comorbidities yielded similar results. Relapse prevalence was 16.4% in the continuation group and 36.4% in the stopping group. Additionally, time to relapse was shorter when antidepressants were discontinued (hazard ratio [HR], 3.63; n = 11 studies). Again, the heterogeneity for relapse rate was low (I2 = 0%). The original publications did not consistently report medication tolerance or withdrawal symptoms, preventing analysis of these. Dropout rates were higher in the stopping group (OR, 1.31; n = 27 studies).

WHAT’S NEW

No more guessing about how long to treat

Previously, there was limited evidence to guide decisions about the duration of antidepressant treatment for anxiety disorders. This study provides evidence that stopping antidepressants before completing 1 year of treatment increases the risk for relapse.

CAVEATS

In a word: Bias

While the authors used standard and appropriate methodologies for this type of study, some significant threats to validity remained. All but 2 studies in the analysis were industry funded. Publication bias is another potential issue, even though the authors identified and included 6 unpublished studies, 4 of which had negative results.

Additionally, the authors graded 11 of 28 trials as having a high likelihood of selective reporting bias, meaning that important portions of the original studies’ results may not have been published. Most studies were at high risk for attrition bias, resulting in loss of information when patients dropped out of the study. While this happened more often in the stopping groups, it is still possible that there are unidentified harms or unexpected outcomes in the medication groups.

While PTSD and OCD are no longer considered anxiety disorders, subgroup analyses found no difference in relapse rates between these diagnoses and the others included in the studies. Finally, a treatment duration longer than 52 weeks has not been studied, so the optimal treatment duration is unknown.

Continue to: CHALLENGES TO IMPLEMENTATION

 

 

CHALLENGES TO IMPLEMENTATION

Patient resistance to continuing treatment

Some patients may want to discontinue antidepressant treatment if their anxiety symptoms improve before completing 1 year. It may be difficult to convince them that continuing treatment will prevent relapse of their condition. Providing patients with information about the increased relapse rate associated with stopping their antidepressant early (with an estimated number needed to treat of 5) may help patients make a more informed decision.

ACKNOWLEDGMENT

The PURLs Surveillance System was supported in part by Grant Number UL1RR024999 from the National Center For Research Resources, a Clinical Translational Science Award to the University of Chicago. The content is solely the responsibility of the authors and does not necessarily represent the official views of the National Center For Research Resources or the National Institutes of Health.

Copyright © 2019. The Family Physicians Inquiries Network. All rights reserved.

Reprinted with permission from the Family Physicians Inquiries Network and The Journal of Family Practice (2019;68[7]:409-410).

References

1. Batelaan NM, Bosman RC, Muntingh A, et al. Risk of relapse after antidepressant discontinuation in anxiety disorders, obsessive-compulsive disorder, and post-traumatic stress disorder: systematic review and meta-analysis of relapse prevention trials. BMJ. 2017;358:j3927. Erratum in: BMJ. 2017;358:j4461.
2. National Institute of Mental Health. Prevalence of any anxiety disorder among adults. www.nimh.nih.gov/health/statistics/any-anxiety-disorder.shtml#part_155094. Updated November 2017. Accessed November 26, 2019.
3. Kessler RC, Petukhova M, Sampson NA, et al. Twelve-month and lifetime prevalence and lifetime morbid risk of anxiety and mood disorders in the United States. Int J Methods Psychiatr Res. 2012;21:169-184.
4. Bandelow B, Sher L, Bunevicius R, et al. Guidelines for the pharmacological treatment of anxiety disorders, obsessive-compulsive disorder and posttraumatic stress disorder in primary care. Int J Psychiatry Clin Pract. 2012;16:77-84.
5. Kaczkurkin AN, Foa EB. Cognitive-behavioral therapy for anxiety disorders: an update on the empirical evidence. Dialogues Clin Neurosci. 2015;17:337-346.
6. Donovan MR, Glue P, Kolluri S, et al. Comparative efficacy of antidepressants in preventing relapse in anxiety disorders—a meta-analysis. J Affect Disord. 2010;123:9-16.
7. Mojtabai R, Olfson M. National trends in long-term use of antidepressant medications: results from the U.S. National Health and Nutrition Examination Survey. J Clin Psychiatry. 2014;75:169-177.

Author and Disclosure Information

Patricia McGuire and Gregory Castelli are with UPMC St. Margaret in Pittsburgh.

Issue
Clinician Reviews - 29(12)
Publications
Topics
Sections
Author and Disclosure Information

Patricia McGuire and Gregory Castelli are with UPMC St. Margaret in Pittsburgh.

Author and Disclosure Information

Patricia McGuire and Gregory Castelli are with UPMC St. Margaret in Pittsburgh.

PRACTICE CHANGER

Keep patients on antidepressant therapy for anxiety disorders for a year or longer before considering a taper.1

STRENGTH OF RECOMMENDATION

A: Based on a systematic review/meta-analysis of several good-quality randomized controlled trials.

A 42-year-old woman with generalized anxiety disorder (GAD) and panic attacks has been treated with sertraline (100 mg/d) for the past 8 months. She has also engaged in cognitive behavioral therapy (CBT) for 6 months. Her Generalized Anxiety Disorder-7 score has decreased from 19 prior to treatment to 5 at present. Now she would like to stop her antidepressant medication because she feels better. Would you recommend that she discontinue her medication at this point?

Anxiety disorders are common and often chronic and can cause significant morbidity and impairment.2,3 Firstline treatments for anxiety disorders include CBT and antidepressants, particularly selective serotonin reuptake inhibitors and serotonin-norepinephrine reuptake inhibitors.4-6

There is limited evidence regarding duration of antidepressant therapy for anxiety disorders. Previous studies have shown a high risk for relapse after discontinuation of antidepressants.6 A review of current practice patterns regarding pharmacologic treatment of depression and anxiety indicates an uptick in longer term antidepressant use for up to 2 years.7 However, long-term studies to guide treatment decisions are lacking.

STUDY SUMMARY

Clear benefit of continuing treatment

This systematic review and meta-analysis evaluated studies that looked at relapse rates and time to relapse in patients treated for anxiety disorders.1 The authors used PubMed, Cochrane, and Embase to identify studies involving patients treated for a variety of psychiatric disorders, including GAD, posttraumatic stress disorder (PTSD), panic disorder (PD), obsessive compulsive disorder (OCD), and social phobia. Eligible studies enrolled patients with anxiety disorders who had a positive response to an antidepressant and then randomized them in a double-blind fashion to either discontinuation of antidepressants and commencement of placebo (stopping group) or continuation of antidepressants (continuation group) for a duration of 8 to 52 weeks. The primary outcomes were relapse rate and time to relapse.

Twenty-eight studies met the inclusion criteria for the meta-analysis, with a total of 5233 patients (2625 patients in the continuation group and 2608 patients in the stopping group). A breakdown of the trials by indication included OCD (7), PD (6), GAD (6), social phobia (5), and PTSD (4). The authors graded the overall risk for bias to be low but noted that attrition bias was present in most studies.

Continue to: Results

 

 

Results. Relapse was more likely in the stopping group (odds ratio [OR], 3.11; n = 28 studies). Heterogeneity for relapse rate was low (I2 = 8.07%). Subgroup analyses by type of antidepressant, mode of discontinuation, and exclusion of patient comorbidities yielded similar results. Relapse prevalence was 16.4% in the continuation group and 36.4% in the stopping group. Additionally, time to relapse was shorter when antidepressants were discontinued (hazard ratio [HR], 3.63; n = 11 studies). Again, the heterogeneity for relapse rate was low (I2 = 0%). The original publications did not consistently report medication tolerance or withdrawal symptoms, preventing analysis of these. Dropout rates were higher in the stopping group (OR, 1.31; n = 27 studies).

WHAT’S NEW

No more guessing about how long to treat

Previously, there was limited evidence to guide decisions about the duration of antidepressant treatment for anxiety disorders. This study provides evidence that stopping antidepressants before completing 1 year of treatment increases the risk for relapse.

CAVEATS

In a word: Bias

While the authors used standard and appropriate methodologies for this type of study, some significant threats to validity remained. All but 2 studies in the analysis were industry funded. Publication bias is another potential issue, even though the authors identified and included 6 unpublished studies, 4 of which had negative results.

Additionally, the authors graded 11 of 28 trials as having a high likelihood of selective reporting bias, meaning that important portions of the original studies’ results may not have been published. Most studies were at high risk for attrition bias, resulting in loss of information when patients dropped out of the study. While this happened more often in the stopping groups, it is still possible that there are unidentified harms or unexpected outcomes in the medication groups.

While PTSD and OCD are no longer considered anxiety disorders, subgroup analyses found no difference in relapse rates between these diagnoses and the others included in the studies. Finally, a treatment duration longer than 52 weeks has not been studied, so the optimal treatment duration is unknown.

Continue to: CHALLENGES TO IMPLEMENTATION

 

 

CHALLENGES TO IMPLEMENTATION

Patient resistance to continuing treatment

Some patients may want to discontinue antidepressant treatment if their anxiety symptoms improve before completing 1 year. It may be difficult to convince them that continuing treatment will prevent relapse of their condition. Providing patients with information about the increased relapse rate associated with stopping their antidepressant early (with an estimated number needed to treat of 5) may help patients make a more informed decision.

ACKNOWLEDGMENT

The PURLs Surveillance System was supported in part by Grant Number UL1RR024999 from the National Center For Research Resources, a Clinical Translational Science Award to the University of Chicago. The content is solely the responsibility of the authors and does not necessarily represent the official views of the National Center For Research Resources or the National Institutes of Health.

Copyright © 2019. The Family Physicians Inquiries Network. All rights reserved.

Reprinted with permission from the Family Physicians Inquiries Network and The Journal of Family Practice (2019;68[7]:409-410).

PRACTICE CHANGER

Keep patients on antidepressant therapy for anxiety disorders for a year or longer before considering a taper.1

STRENGTH OF RECOMMENDATION

A: Based on a systematic review/meta-analysis of several good-quality randomized controlled trials.

A 42-year-old woman with generalized anxiety disorder (GAD) and panic attacks has been treated with sertraline (100 mg/d) for the past 8 months. She has also engaged in cognitive behavioral therapy (CBT) for 6 months. Her Generalized Anxiety Disorder-7 score has decreased from 19 prior to treatment to 5 at present. Now she would like to stop her antidepressant medication because she feels better. Would you recommend that she discontinue her medication at this point?

Anxiety disorders are common and often chronic and can cause significant morbidity and impairment.2,3 Firstline treatments for anxiety disorders include CBT and antidepressants, particularly selective serotonin reuptake inhibitors and serotonin-norepinephrine reuptake inhibitors.4-6

There is limited evidence regarding duration of antidepressant therapy for anxiety disorders. Previous studies have shown a high risk for relapse after discontinuation of antidepressants.6 A review of current practice patterns regarding pharmacologic treatment of depression and anxiety indicates an uptick in longer term antidepressant use for up to 2 years.7 However, long-term studies to guide treatment decisions are lacking.

STUDY SUMMARY

Clear benefit of continuing treatment

This systematic review and meta-analysis evaluated studies that looked at relapse rates and time to relapse in patients treated for anxiety disorders.1 The authors used PubMed, Cochrane, and Embase to identify studies involving patients treated for a variety of psychiatric disorders, including GAD, posttraumatic stress disorder (PTSD), panic disorder (PD), obsessive compulsive disorder (OCD), and social phobia. Eligible studies enrolled patients with anxiety disorders who had a positive response to an antidepressant and then randomized them in a double-blind fashion to either discontinuation of antidepressants and commencement of placebo (stopping group) or continuation of antidepressants (continuation group) for a duration of 8 to 52 weeks. The primary outcomes were relapse rate and time to relapse.

Twenty-eight studies met the inclusion criteria for the meta-analysis, with a total of 5233 patients (2625 patients in the continuation group and 2608 patients in the stopping group). A breakdown of the trials by indication included OCD (7), PD (6), GAD (6), social phobia (5), and PTSD (4). The authors graded the overall risk for bias to be low but noted that attrition bias was present in most studies.

Continue to: Results

 

 

Results. Relapse was more likely in the stopping group (odds ratio [OR], 3.11; n = 28 studies). Heterogeneity for relapse rate was low (I2 = 8.07%). Subgroup analyses by type of antidepressant, mode of discontinuation, and exclusion of patient comorbidities yielded similar results. Relapse prevalence was 16.4% in the continuation group and 36.4% in the stopping group. Additionally, time to relapse was shorter when antidepressants were discontinued (hazard ratio [HR], 3.63; n = 11 studies). Again, the heterogeneity for relapse rate was low (I2 = 0%). The original publications did not consistently report medication tolerance or withdrawal symptoms, preventing analysis of these. Dropout rates were higher in the stopping group (OR, 1.31; n = 27 studies).

WHAT’S NEW

No more guessing about how long to treat

Previously, there was limited evidence to guide decisions about the duration of antidepressant treatment for anxiety disorders. This study provides evidence that stopping antidepressants before completing 1 year of treatment increases the risk for relapse.

CAVEATS

In a word: Bias

While the authors used standard and appropriate methodologies for this type of study, some significant threats to validity remained. All but 2 studies in the analysis were industry funded. Publication bias is another potential issue, even though the authors identified and included 6 unpublished studies, 4 of which had negative results.

Additionally, the authors graded 11 of 28 trials as having a high likelihood of selective reporting bias, meaning that important portions of the original studies’ results may not have been published. Most studies were at high risk for attrition bias, resulting in loss of information when patients dropped out of the study. While this happened more often in the stopping groups, it is still possible that there are unidentified harms or unexpected outcomes in the medication groups.

While PTSD and OCD are no longer considered anxiety disorders, subgroup analyses found no difference in relapse rates between these diagnoses and the others included in the studies. Finally, a treatment duration longer than 52 weeks has not been studied, so the optimal treatment duration is unknown.

Continue to: CHALLENGES TO IMPLEMENTATION

 

 

CHALLENGES TO IMPLEMENTATION

Patient resistance to continuing treatment

Some patients may want to discontinue antidepressant treatment if their anxiety symptoms improve before completing 1 year. It may be difficult to convince them that continuing treatment will prevent relapse of their condition. Providing patients with information about the increased relapse rate associated with stopping their antidepressant early (with an estimated number needed to treat of 5) may help patients make a more informed decision.

ACKNOWLEDGMENT

The PURLs Surveillance System was supported in part by Grant Number UL1RR024999 from the National Center For Research Resources, a Clinical Translational Science Award to the University of Chicago. The content is solely the responsibility of the authors and does not necessarily represent the official views of the National Center For Research Resources or the National Institutes of Health.

Copyright © 2019. The Family Physicians Inquiries Network. All rights reserved.

Reprinted with permission from the Family Physicians Inquiries Network and The Journal of Family Practice (2019;68[7]:409-410).

References

1. Batelaan NM, Bosman RC, Muntingh A, et al. Risk of relapse after antidepressant discontinuation in anxiety disorders, obsessive-compulsive disorder, and post-traumatic stress disorder: systematic review and meta-analysis of relapse prevention trials. BMJ. 2017;358:j3927. Erratum in: BMJ. 2017;358:j4461.
2. National Institute of Mental Health. Prevalence of any anxiety disorder among adults. www.nimh.nih.gov/health/statistics/any-anxiety-disorder.shtml#part_155094. Updated November 2017. Accessed November 26, 2019.
3. Kessler RC, Petukhova M, Sampson NA, et al. Twelve-month and lifetime prevalence and lifetime morbid risk of anxiety and mood disorders in the United States. Int J Methods Psychiatr Res. 2012;21:169-184.
4. Bandelow B, Sher L, Bunevicius R, et al. Guidelines for the pharmacological treatment of anxiety disorders, obsessive-compulsive disorder and posttraumatic stress disorder in primary care. Int J Psychiatry Clin Pract. 2012;16:77-84.
5. Kaczkurkin AN, Foa EB. Cognitive-behavioral therapy for anxiety disorders: an update on the empirical evidence. Dialogues Clin Neurosci. 2015;17:337-346.
6. Donovan MR, Glue P, Kolluri S, et al. Comparative efficacy of antidepressants in preventing relapse in anxiety disorders—a meta-analysis. J Affect Disord. 2010;123:9-16.
7. Mojtabai R, Olfson M. National trends in long-term use of antidepressant medications: results from the U.S. National Health and Nutrition Examination Survey. J Clin Psychiatry. 2014;75:169-177.

References

1. Batelaan NM, Bosman RC, Muntingh A, et al. Risk of relapse after antidepressant discontinuation in anxiety disorders, obsessive-compulsive disorder, and post-traumatic stress disorder: systematic review and meta-analysis of relapse prevention trials. BMJ. 2017;358:j3927. Erratum in: BMJ. 2017;358:j4461.
2. National Institute of Mental Health. Prevalence of any anxiety disorder among adults. www.nimh.nih.gov/health/statistics/any-anxiety-disorder.shtml#part_155094. Updated November 2017. Accessed November 26, 2019.
3. Kessler RC, Petukhova M, Sampson NA, et al. Twelve-month and lifetime prevalence and lifetime morbid risk of anxiety and mood disorders in the United States. Int J Methods Psychiatr Res. 2012;21:169-184.
4. Bandelow B, Sher L, Bunevicius R, et al. Guidelines for the pharmacological treatment of anxiety disorders, obsessive-compulsive disorder and posttraumatic stress disorder in primary care. Int J Psychiatry Clin Pract. 2012;16:77-84.
5. Kaczkurkin AN, Foa EB. Cognitive-behavioral therapy for anxiety disorders: an update on the empirical evidence. Dialogues Clin Neurosci. 2015;17:337-346.
6. Donovan MR, Glue P, Kolluri S, et al. Comparative efficacy of antidepressants in preventing relapse in anxiety disorders—a meta-analysis. J Affect Disord. 2010;123:9-16.
7. Mojtabai R, Olfson M. National trends in long-term use of antidepressant medications: results from the U.S. National Health and Nutrition Examination Survey. J Clin Psychiatry. 2014;75:169-177.

Issue
Clinician Reviews - 29(12)
Issue
Clinician Reviews - 29(12)
Publications
Publications
Topics
Article Type
Display Headline
Antidepressant Tx for Anxiety Disorders: How Long?
Display Headline
Antidepressant Tx for Anxiety Disorders: How Long?
Sections
Disallow All Ads
Content Gating
No Gating (article Unlocked/Free)
Alternative CME
Disqus Comments
Default
Gate On Date
Tue, 12/03/2019 - 09:15
Un-Gate On Date
Tue, 12/03/2019 - 09:15
Use ProPublica
CFC Schedule Remove Status
Tue, 12/03/2019 - 09:15
Hide sidebar & use full width
render the right sidebar.

Antidepressant Tx for anxiety disorders: How long?

Article Type
Changed
Thu, 09/26/2019 - 12:01
Display Headline
Antidepressant Tx for anxiety disorders: How long?

ILLUSTRATIVE CASE

A 42-year-old woman with generalized anxiety disorder and panic attacks has been treated with sertraline 100 mg/d for the past 8 months. She has also engaged in cognitive behavioral therapy (CBT) for 6 months. Her Generalized Anxiety Disorder-7 score has decreased from 19 prior to treatment to 5 at present. Now she would like to stop her antidepressant medication because she feels better. Would you recommend that she discontinue her medication at this point?

Anxiety disorders are common, often chronic, and can cause significant morbidity and impairment.2,3 First-line treatments for anxiety disorders include CBT and antidepressants, particularly selective serotonin reuptake inhibitors and serotonin-norepinephrine reuptake inhibitors.4-6

There is limited evidence regarding duration of antidepressant therapy for anxiety disorders. Previous studies have shown a high risk of relapse after discontinuation of antidepressants.6 A review of current practice patterns regarding pharmacologic treatment of depression and anxiety indicates an uptick in longer term antidepressant use for up to 2 years.7 However, long-term studies to guide treatment decisions are lacking.

 

STUDY SUMMARY

Clear benefit of continuing treatment up to 1 year

This systematic review and meta-analysis evaluated studies that looked at relapse rates and time to relapse in patients treated for anxiety disorders.1 The authors used PubMed, Cochrane, and Embase to identify studies involving patients treated for a variety of disorders, including generalized anxiety disorder (GAD), posttraumatic stress disorder (PTSD), panic disorder (PD), obsessive-compulsive disorder (OCD), and social phobia. Eligible studies enrolled patients with anxiety disorders who had a positive response to an antidepressant and then randomized them in a double-blind fashion to either discontinuation of antidepressants and starting placebo (stopping group) or continuation of antidepressants (continuation group) for a duration of 8 to 52 weeks. The primary outcomes were relapse rate and time to relapse.

Twenty-eight studies met the inclusion criteria for the meta-analysis, with a total of 5233 patients (2625 patients in the antidepressant group and 2608 patients in the placebo group). A breakdown of the trials by indiication included OCD (7), PD (6), GAD (6), social phobia (5), and PTSD (4). The authors graded the overall risk of bias to be low but noted that attrition bias was present in most studies.

Results. Relapse was more likely in the stopping group (odds ratio [OR] = 3.11; 95% confidence interval [CI], 2.48-3.89; n = 28 studies). Heterogeneity for relapse rate was low (I2 = 8.07%). Subgroup analyses by type of antidepressant, mode of discontinuation, and exclusion of patient comorbidities yielded similar results. Relapse prevalence was 16.4% in the antidepressant group and 36.4% in the stopping group. Additionally, time to relapse was shorter when antidepressants were discontinued (hazard ratio [HR] = 3.63; 95% CI, 2.58-5.10; n = 11 studies). Again, the heterogeneity for relapse rate was low (I2 = 0%). The original publications did not consistently report medication tolerability or withdrawal symptoms, preventing analysis of these. Dropout rates were higher in the stopping group (OR = 1.31; 95% CI, 1.06-1.63; n = 27 studies).

WHAT’S NEW

No more guessing about how long to treat

Previously, there was limited evidence to guide decisions about the duration of antidepressant treatment for anxiety disorders. This study provides evidence that stopping antidepressant treatment before 1 year increases the risk of relapse.

Continue to: CAVEATS

 

 

CAVEATS

Potential bias … bias … and more bias

While the authors used standard and appropriate methodologies for this type of study, some significant threats to validity remained. All but 2 studies in the analysis were industry funded. Publication bias is another potential issue, even though the authors identified and included 6 unpublished studies, 4 of which had negative results.

This study provides evidence that stopping antidepressant treatment for anxiety disorders before 1 year increases the risk of relapse.

Additionally, the authors graded 11 of 28 trials as having a high likelihood of selective reporting bias, meaning that important portions of the original studies’ results may not have been published. Most studies were at high risk for attrition bias, resulting in loss of information when patients dropped out of the study. While this happened more often in the stopping groups, it is still possible that there are unidentified harms or unexpected outcomes in the medication groups.

While PTSD and OCD are no longer considered anxiety disorders, subgroup analyses found no difference in relapse rates between these diagnoses and the others included in the studies. Finally, treatment duration longer than 52 weeks has not been studied, so the optimal treatment duration is unknown.

CHALLENGES TO IMPLEMENTATION

Patients may resist continuing treatment once symptoms abate

Some patients may want to discontinue antidepressant treatment if their anxiety symptoms improve prior to 1 year. It may be difficult to convince them that continuing treatment will prevent relapse of their condition. Providing patients with information about the increased relapse rate with stopping medication early (with an estimated number needed to treat of 5) may help patients make a more informed decision.

ACKNOWLEDGMENT

The PURLs Surveillance System was supported in part by Grant Number UL1RR024999 from the National Center For Research Resources, a Clinical Translational Science Award to the University of Chicago. The content is solely the responsibility of the authors and does not necessarily represent the official views of the National Center For Research Resources or the National Institutes of Health.

Files
References

1. Batelaan NM, Bosman RC, Muntingh A, et al. Risk of relapse after antidepressant discontinuation in anxiety disorders, obsessive-compulsive disorder, and post-traumatic stress disorder: systematic review and meta-analysis of relapse prevention trials. BMJ. 2017;358:j3927. Erratum in: BMJ. 2017;358:j4461.

2. National Institute of Mental Health. Prevalence of any anxiety disorder among adults. https://www.nimh.nih.gov/health/statistics/any-anxiety-disorder.shtml#part_155094. Updated November 2017. Accessed July 11, 2019.

3. Kessler RC, Petukhova M, Sampson NA, et al. Twelve-month and lifetime prevalence and lifetime morbid risk of anxiety and mood disorders in the United States. Int J Methods Psychiatr Res. 2012;21:169-184.

4. Bandelow B, Sher L, Bunevicius R, et al. Guidelines for the pharmacological treatment of anxiety disorders, obsessive-compulsive disorder and posttraumatic stress disorder in primary care. Int J Psychiatry Clin Pract. 2012;16:77-84.

5. Kaczkurkin AN, Foa EB. Cognitive-behavioral therapy for anxiety disorders: an update on the empirical evidence. Dialogues Clin Neurosci. 2015;17:337-346.

6. Donovan MR, Glue P, Kolluri S, et al. Comparative efficacy of antidepressants in preventing relapse in anxiety disorders—a meta-analysis. J Affect Disord. 2010;123:9-16.

7. Mojtabai R, Olfson M. National trends in long-term use of antidepressant medications: results from the U.S. National Health and Nutrition Examination Survey. J Clin Psychiatry. 2014;75:169-177.

Article PDF
Author and Disclosure Information

UPMC St. Margaret, Pittsburgh, Pa

DEPUTY EDITOR
James J. Stevermer, MD, MSPH

Department of Family and Community Medicine, University of Missouri-Columbia

Issue
The Journal of Family Practice - 68(7)
Publications
Topics
Page Number
409-410
Sections
Files
Files
Author and Disclosure Information

UPMC St. Margaret, Pittsburgh, Pa

DEPUTY EDITOR
James J. Stevermer, MD, MSPH

Department of Family and Community Medicine, University of Missouri-Columbia

Author and Disclosure Information

UPMC St. Margaret, Pittsburgh, Pa

DEPUTY EDITOR
James J. Stevermer, MD, MSPH

Department of Family and Community Medicine, University of Missouri-Columbia

Article PDF
Article PDF

ILLUSTRATIVE CASE

A 42-year-old woman with generalized anxiety disorder and panic attacks has been treated with sertraline 100 mg/d for the past 8 months. She has also engaged in cognitive behavioral therapy (CBT) for 6 months. Her Generalized Anxiety Disorder-7 score has decreased from 19 prior to treatment to 5 at present. Now she would like to stop her antidepressant medication because she feels better. Would you recommend that she discontinue her medication at this point?

Anxiety disorders are common, often chronic, and can cause significant morbidity and impairment.2,3 First-line treatments for anxiety disorders include CBT and antidepressants, particularly selective serotonin reuptake inhibitors and serotonin-norepinephrine reuptake inhibitors.4-6

There is limited evidence regarding duration of antidepressant therapy for anxiety disorders. Previous studies have shown a high risk of relapse after discontinuation of antidepressants.6 A review of current practice patterns regarding pharmacologic treatment of depression and anxiety indicates an uptick in longer term antidepressant use for up to 2 years.7 However, long-term studies to guide treatment decisions are lacking.

 

STUDY SUMMARY

Clear benefit of continuing treatment up to 1 year

This systematic review and meta-analysis evaluated studies that looked at relapse rates and time to relapse in patients treated for anxiety disorders.1 The authors used PubMed, Cochrane, and Embase to identify studies involving patients treated for a variety of disorders, including generalized anxiety disorder (GAD), posttraumatic stress disorder (PTSD), panic disorder (PD), obsessive-compulsive disorder (OCD), and social phobia. Eligible studies enrolled patients with anxiety disorders who had a positive response to an antidepressant and then randomized them in a double-blind fashion to either discontinuation of antidepressants and starting placebo (stopping group) or continuation of antidepressants (continuation group) for a duration of 8 to 52 weeks. The primary outcomes were relapse rate and time to relapse.

Twenty-eight studies met the inclusion criteria for the meta-analysis, with a total of 5233 patients (2625 patients in the antidepressant group and 2608 patients in the placebo group). A breakdown of the trials by indiication included OCD (7), PD (6), GAD (6), social phobia (5), and PTSD (4). The authors graded the overall risk of bias to be low but noted that attrition bias was present in most studies.

Results. Relapse was more likely in the stopping group (odds ratio [OR] = 3.11; 95% confidence interval [CI], 2.48-3.89; n = 28 studies). Heterogeneity for relapse rate was low (I2 = 8.07%). Subgroup analyses by type of antidepressant, mode of discontinuation, and exclusion of patient comorbidities yielded similar results. Relapse prevalence was 16.4% in the antidepressant group and 36.4% in the stopping group. Additionally, time to relapse was shorter when antidepressants were discontinued (hazard ratio [HR] = 3.63; 95% CI, 2.58-5.10; n = 11 studies). Again, the heterogeneity for relapse rate was low (I2 = 0%). The original publications did not consistently report medication tolerability or withdrawal symptoms, preventing analysis of these. Dropout rates were higher in the stopping group (OR = 1.31; 95% CI, 1.06-1.63; n = 27 studies).

WHAT’S NEW

No more guessing about how long to treat

Previously, there was limited evidence to guide decisions about the duration of antidepressant treatment for anxiety disorders. This study provides evidence that stopping antidepressant treatment before 1 year increases the risk of relapse.

Continue to: CAVEATS

 

 

CAVEATS

Potential bias … bias … and more bias

While the authors used standard and appropriate methodologies for this type of study, some significant threats to validity remained. All but 2 studies in the analysis were industry funded. Publication bias is another potential issue, even though the authors identified and included 6 unpublished studies, 4 of which had negative results.

This study provides evidence that stopping antidepressant treatment for anxiety disorders before 1 year increases the risk of relapse.

Additionally, the authors graded 11 of 28 trials as having a high likelihood of selective reporting bias, meaning that important portions of the original studies’ results may not have been published. Most studies were at high risk for attrition bias, resulting in loss of information when patients dropped out of the study. While this happened more often in the stopping groups, it is still possible that there are unidentified harms or unexpected outcomes in the medication groups.

While PTSD and OCD are no longer considered anxiety disorders, subgroup analyses found no difference in relapse rates between these diagnoses and the others included in the studies. Finally, treatment duration longer than 52 weeks has not been studied, so the optimal treatment duration is unknown.

CHALLENGES TO IMPLEMENTATION

Patients may resist continuing treatment once symptoms abate

Some patients may want to discontinue antidepressant treatment if their anxiety symptoms improve prior to 1 year. It may be difficult to convince them that continuing treatment will prevent relapse of their condition. Providing patients with information about the increased relapse rate with stopping medication early (with an estimated number needed to treat of 5) may help patients make a more informed decision.

ACKNOWLEDGMENT

The PURLs Surveillance System was supported in part by Grant Number UL1RR024999 from the National Center For Research Resources, a Clinical Translational Science Award to the University of Chicago. The content is solely the responsibility of the authors and does not necessarily represent the official views of the National Center For Research Resources or the National Institutes of Health.

ILLUSTRATIVE CASE

A 42-year-old woman with generalized anxiety disorder and panic attacks has been treated with sertraline 100 mg/d for the past 8 months. She has also engaged in cognitive behavioral therapy (CBT) for 6 months. Her Generalized Anxiety Disorder-7 score has decreased from 19 prior to treatment to 5 at present. Now she would like to stop her antidepressant medication because she feels better. Would you recommend that she discontinue her medication at this point?

Anxiety disorders are common, often chronic, and can cause significant morbidity and impairment.2,3 First-line treatments for anxiety disorders include CBT and antidepressants, particularly selective serotonin reuptake inhibitors and serotonin-norepinephrine reuptake inhibitors.4-6

There is limited evidence regarding duration of antidepressant therapy for anxiety disorders. Previous studies have shown a high risk of relapse after discontinuation of antidepressants.6 A review of current practice patterns regarding pharmacologic treatment of depression and anxiety indicates an uptick in longer term antidepressant use for up to 2 years.7 However, long-term studies to guide treatment decisions are lacking.

 

STUDY SUMMARY

Clear benefit of continuing treatment up to 1 year

This systematic review and meta-analysis evaluated studies that looked at relapse rates and time to relapse in patients treated for anxiety disorders.1 The authors used PubMed, Cochrane, and Embase to identify studies involving patients treated for a variety of disorders, including generalized anxiety disorder (GAD), posttraumatic stress disorder (PTSD), panic disorder (PD), obsessive-compulsive disorder (OCD), and social phobia. Eligible studies enrolled patients with anxiety disorders who had a positive response to an antidepressant and then randomized them in a double-blind fashion to either discontinuation of antidepressants and starting placebo (stopping group) or continuation of antidepressants (continuation group) for a duration of 8 to 52 weeks. The primary outcomes were relapse rate and time to relapse.

Twenty-eight studies met the inclusion criteria for the meta-analysis, with a total of 5233 patients (2625 patients in the antidepressant group and 2608 patients in the placebo group). A breakdown of the trials by indiication included OCD (7), PD (6), GAD (6), social phobia (5), and PTSD (4). The authors graded the overall risk of bias to be low but noted that attrition bias was present in most studies.

Results. Relapse was more likely in the stopping group (odds ratio [OR] = 3.11; 95% confidence interval [CI], 2.48-3.89; n = 28 studies). Heterogeneity for relapse rate was low (I2 = 8.07%). Subgroup analyses by type of antidepressant, mode of discontinuation, and exclusion of patient comorbidities yielded similar results. Relapse prevalence was 16.4% in the antidepressant group and 36.4% in the stopping group. Additionally, time to relapse was shorter when antidepressants were discontinued (hazard ratio [HR] = 3.63; 95% CI, 2.58-5.10; n = 11 studies). Again, the heterogeneity for relapse rate was low (I2 = 0%). The original publications did not consistently report medication tolerability or withdrawal symptoms, preventing analysis of these. Dropout rates were higher in the stopping group (OR = 1.31; 95% CI, 1.06-1.63; n = 27 studies).

WHAT’S NEW

No more guessing about how long to treat

Previously, there was limited evidence to guide decisions about the duration of antidepressant treatment for anxiety disorders. This study provides evidence that stopping antidepressant treatment before 1 year increases the risk of relapse.

Continue to: CAVEATS

 

 

CAVEATS

Potential bias … bias … and more bias

While the authors used standard and appropriate methodologies for this type of study, some significant threats to validity remained. All but 2 studies in the analysis were industry funded. Publication bias is another potential issue, even though the authors identified and included 6 unpublished studies, 4 of which had negative results.

This study provides evidence that stopping antidepressant treatment for anxiety disorders before 1 year increases the risk of relapse.

Additionally, the authors graded 11 of 28 trials as having a high likelihood of selective reporting bias, meaning that important portions of the original studies’ results may not have been published. Most studies were at high risk for attrition bias, resulting in loss of information when patients dropped out of the study. While this happened more often in the stopping groups, it is still possible that there are unidentified harms or unexpected outcomes in the medication groups.

While PTSD and OCD are no longer considered anxiety disorders, subgroup analyses found no difference in relapse rates between these diagnoses and the others included in the studies. Finally, treatment duration longer than 52 weeks has not been studied, so the optimal treatment duration is unknown.

CHALLENGES TO IMPLEMENTATION

Patients may resist continuing treatment once symptoms abate

Some patients may want to discontinue antidepressant treatment if their anxiety symptoms improve prior to 1 year. It may be difficult to convince them that continuing treatment will prevent relapse of their condition. Providing patients with information about the increased relapse rate with stopping medication early (with an estimated number needed to treat of 5) may help patients make a more informed decision.

ACKNOWLEDGMENT

The PURLs Surveillance System was supported in part by Grant Number UL1RR024999 from the National Center For Research Resources, a Clinical Translational Science Award to the University of Chicago. The content is solely the responsibility of the authors and does not necessarily represent the official views of the National Center For Research Resources or the National Institutes of Health.

References

1. Batelaan NM, Bosman RC, Muntingh A, et al. Risk of relapse after antidepressant discontinuation in anxiety disorders, obsessive-compulsive disorder, and post-traumatic stress disorder: systematic review and meta-analysis of relapse prevention trials. BMJ. 2017;358:j3927. Erratum in: BMJ. 2017;358:j4461.

2. National Institute of Mental Health. Prevalence of any anxiety disorder among adults. https://www.nimh.nih.gov/health/statistics/any-anxiety-disorder.shtml#part_155094. Updated November 2017. Accessed July 11, 2019.

3. Kessler RC, Petukhova M, Sampson NA, et al. Twelve-month and lifetime prevalence and lifetime morbid risk of anxiety and mood disorders in the United States. Int J Methods Psychiatr Res. 2012;21:169-184.

4. Bandelow B, Sher L, Bunevicius R, et al. Guidelines for the pharmacological treatment of anxiety disorders, obsessive-compulsive disorder and posttraumatic stress disorder in primary care. Int J Psychiatry Clin Pract. 2012;16:77-84.

5. Kaczkurkin AN, Foa EB. Cognitive-behavioral therapy for anxiety disorders: an update on the empirical evidence. Dialogues Clin Neurosci. 2015;17:337-346.

6. Donovan MR, Glue P, Kolluri S, et al. Comparative efficacy of antidepressants in preventing relapse in anxiety disorders—a meta-analysis. J Affect Disord. 2010;123:9-16.

7. Mojtabai R, Olfson M. National trends in long-term use of antidepressant medications: results from the U.S. National Health and Nutrition Examination Survey. J Clin Psychiatry. 2014;75:169-177.

References

1. Batelaan NM, Bosman RC, Muntingh A, et al. Risk of relapse after antidepressant discontinuation in anxiety disorders, obsessive-compulsive disorder, and post-traumatic stress disorder: systematic review and meta-analysis of relapse prevention trials. BMJ. 2017;358:j3927. Erratum in: BMJ. 2017;358:j4461.

2. National Institute of Mental Health. Prevalence of any anxiety disorder among adults. https://www.nimh.nih.gov/health/statistics/any-anxiety-disorder.shtml#part_155094. Updated November 2017. Accessed July 11, 2019.

3. Kessler RC, Petukhova M, Sampson NA, et al. Twelve-month and lifetime prevalence and lifetime morbid risk of anxiety and mood disorders in the United States. Int J Methods Psychiatr Res. 2012;21:169-184.

4. Bandelow B, Sher L, Bunevicius R, et al. Guidelines for the pharmacological treatment of anxiety disorders, obsessive-compulsive disorder and posttraumatic stress disorder in primary care. Int J Psychiatry Clin Pract. 2012;16:77-84.

5. Kaczkurkin AN, Foa EB. Cognitive-behavioral therapy for anxiety disorders: an update on the empirical evidence. Dialogues Clin Neurosci. 2015;17:337-346.

6. Donovan MR, Glue P, Kolluri S, et al. Comparative efficacy of antidepressants in preventing relapse in anxiety disorders—a meta-analysis. J Affect Disord. 2010;123:9-16.

7. Mojtabai R, Olfson M. National trends in long-term use of antidepressant medications: results from the U.S. National Health and Nutrition Examination Survey. J Clin Psychiatry. 2014;75:169-177.

Issue
The Journal of Family Practice - 68(7)
Issue
The Journal of Family Practice - 68(7)
Page Number
409-410
Page Number
409-410
Publications
Publications
Topics
Article Type
Display Headline
Antidepressant Tx for anxiety disorders: How long?
Display Headline
Antidepressant Tx for anxiety disorders: How long?
Sections
PURLs Copyright
Copyright © 2019. The Family Physicians Inquiries Network. All rights reserved.
Inside the Article

PRACTICE CHANGER

Keep patients on antidepressant therapy for anxiety disorders for a year or longer before considering a taper.

STRENGTH OF RECOMMENDATION

A: Based on a systematic review/meta-analysis of several good quality randomized controlled trials.1

Batelaan NM, Bosman RC, Muntingh A, et al. Risk of relapse after antidepressant discontinuation in anxiety disorders, obsessive-compulsive disorder, and post-traumatic stress disorder: systematic review and meta-analysis of relapse prevention trials. BMJ. 2017;358:j3927. Erratum in: BMJ. 2017;358:j4461.

Disallow All Ads
Content Gating
No Gating (article Unlocked/Free)
Alternative CME
PubMed ID
31532817
Disqus Comments
Default
Use ProPublica
Hide sidebar & use full width
render the right sidebar.
Article PDF Media
Media Files

What’s the Best Treatment Setting for Stable Pulmonary Embolism?

Article Type
Changed
Mon, 05/06/2019 - 00:01
Display Headline
What’s the Best Treatment Setting for Stable Pulmonary Embolism?

Practice Changer

A 63-year-old woman with a history of hypertension presents to the emergency department (ED) with acute-onset shortness of breath and pleuritic chest pain after traveling across the country for a work conference. She has no history of cancer, liver disease, or renal disease. Her blood pressure is 140/80 mm Hg, and her heart rate, 90 beats/min. You diagnose an acute PE in this patient and start anticoagulation. Should you admit her to the hospital to decrease morbidity and mortality?

According to the CDC, venous thromboembolism (VTE) affects about 900,000 people each year, and about 60,000 to 100,000 of these patients die annually.2 Pulmonary embolism is the third leading cause of death from cardiovascular disease, following heart attack and stroke.3 Prompt diagnosis and treatment with systemic anticoagulation improves patient outcomes and decreases the risk for long-term complications.

The 2016 American College of Chest Physicians (CHEST) guideline on antithrombotic therapy for VTE recommends home treatment or early discharge over standard discharge (after the first 5 days of treatment) for patients who meet the following clinical criteria: “clinically stable with good cardiopulmonary reserve; no contraindications such as recent bleeding, severe renal or liver disease, or severe thrombocytopenia (ie, < 70,000/mm3); expected to be compliant with treatment; and feels well enough to be treated at home.”3

The guideline states that various clinical decision tools, such as the Pulmonary Embolism Severity Index (PESI), can aid in identifying low-risk patients to be considered for treatment at home. The PESI uses age, gender, vital signs, mental status, and a history of cancer, lung, and cardiac disease to stratify patients by risk.4

A systematic review of 1 randomized controlled trial (RCT) and 7 observational studies found that in low-risk patients, outpatient treatment was as safe as inpatient treatment.5 This more recent study determines the net clinical benefit of hospitalized versus outpatient management in a wider range of patients with acute PE, regardless of initial risk.1

STUDY SUMMARY

Hospitalization confers no benefit to stable PE patients

This retrospective, propensity-matched cohort study compared rates of adverse events in 1127 patients with acute PE managed in the hospital versus outpatient setting.1 Patients were classified as outpatients if they were discharged from the ED or discharged from the hospital within 48 hours of admission. Patients were included if a symptomatic acute PE was diagnosed via CT or high-probability ventilation-perfusion scan and excluded if they were younger than 19, were diagnosed with a PE during hospitalization, had chronic PE, or were hemodynamically unstable, among other factors. The investigators calculated PESI scores for all patients.

Propensity scores matched patients on 28 characteristics and known risk factors for adverse events to ensure the groups were similar. The primary outcome was rate of adverse events, including recurrent VTE, major bleeding, or death at 14 days. The secondary outcome included rates of the above during the 3-month follow-up period.

Continue to: Of the 1127 eligible patients...

 

 

Of the 1127 eligible patients, 1081 were included in the matched cohort, with 576 (53%) treated as hospitalized patients and 505 (47%) treated as outpatients. The mean age of the matched cohorts was 63.2 years for the inpatient group and 63.6 years for the outpatient group. Overall, the cohorts were well matched.

The 14-day rate of adverse events was higher in hospitalized patients than in outpatients (13% vs 3.3%; odds ratio [OR], 5.07), with each of the adverse events that made up the primary outcome occurring more frequently in the hospitalized group (see Table). The rate of adverse events at 3 months was also greater for hospitalized patients compared with outpatients (21.7% vs 6.9%; OR, 4.9). The results remained similar for high-risk patients (Class III-V) based on their PESI score.

Comparison of Inpatient vs Outpatient Treatment of Patients With Pulmonary Embolism

WHAT’S NEW

Higher rate of adverse events in inpatients

This trial supports the CHEST guideline recommendations to manage hemodynamically stable patients with acute PE as outpatients.3 It adds to the conversation by demonstrating higher rates of adverse events with hospitalization, even in high-risk subgroups (PESI Class III-V).

 

CAVEATS

Good study, but it wasn’t an RCT

While this is a well-designed cohort study, it is not an RCT. This study defined outpatient management as patients discharged from the ED or hospitalized for < 48 hours. However, only 59 of the 544 patients in the outpatient group were early hospital discharges; the rest were never admitted. Finally, a specialized thrombosis clinic followed up with the patients within 24 hours of discharge, and patients had telephone access to specialized health care professionals; such organization of care contributed to the safe outpatient management of these PE patients.

CHALLENGES TO IMPLEMENTATION

Insurance coverage may present an issue

Medication coverage of direct oral anticoagulants and low-molecular-weight heparin may present a barrier to patients treated in the outpatient setting who have no insurance or are insured by certain carriers.

ACKNOWLEDGEMENT

The PURLs Surveillance System was supported in part by Grant Number UL1RR024999 from the National Center For Research Resources, a Clinical Translational Science Award to the University of Chicago. The content is solely the responsibility of the authors and does not necessarily represent the official views of the National Center For Research Resources or the National Institutes of Health.

Copyright © 2018. The Family Physicians Inquiries Network. All rights reserved.

Reprinted with permission from the Family Physicians Inquiries Network and The Journal of Family Practice (2018;67[12]:777-779).

References

1. Roy PM, Corsi DJ, Carrier M, et al. Net clinical benefit of hospitalization versus outpatient management of patients with acute pulmonary embolism. J Thromb Haemost. 2017;15:685-694.
2. CDC. Venous thromboembolism data & statistics. www.cdc.gov/ncbddd/dvt/data.html. Accessed April 26, 2019.
3. Kearon C, Akl EA, Ornelas J, et al. Antithrombotic therapy for VTE disease: CHEST guideline and expert panel report. CHEST. 2016;149:315-352.
4. Aujesky D, Obrosky DS, Stone RA, et al. Derivation and validation of a prognostic model for pulmonary embolism. Am J Respir Crit Care Med. 2005;172:1041-1046.
5. Vinson DR, Zehtabchi S, Yealy DM. Can selected patients with newly diagnosed pulmonary embolism be safely treated without hospitalization? A systematic review. Ann Emerg Med. 2012;60:651-662.

Article PDF
Author and Disclosure Information

Monica Schaffer and Gregory Castelli are with University of Pittsburgh Medical Center St. Margaret.

Issue
Clinician Reviews - 29(5)
Publications
Topics
Page Number
3e-4e
Sections
Author and Disclosure Information

Monica Schaffer and Gregory Castelli are with University of Pittsburgh Medical Center St. Margaret.

Author and Disclosure Information

Monica Schaffer and Gregory Castelli are with University of Pittsburgh Medical Center St. Margaret.

Article PDF
Article PDF

Practice Changer

A 63-year-old woman with a history of hypertension presents to the emergency department (ED) with acute-onset shortness of breath and pleuritic chest pain after traveling across the country for a work conference. She has no history of cancer, liver disease, or renal disease. Her blood pressure is 140/80 mm Hg, and her heart rate, 90 beats/min. You diagnose an acute PE in this patient and start anticoagulation. Should you admit her to the hospital to decrease morbidity and mortality?

According to the CDC, venous thromboembolism (VTE) affects about 900,000 people each year, and about 60,000 to 100,000 of these patients die annually.2 Pulmonary embolism is the third leading cause of death from cardiovascular disease, following heart attack and stroke.3 Prompt diagnosis and treatment with systemic anticoagulation improves patient outcomes and decreases the risk for long-term complications.

The 2016 American College of Chest Physicians (CHEST) guideline on antithrombotic therapy for VTE recommends home treatment or early discharge over standard discharge (after the first 5 days of treatment) for patients who meet the following clinical criteria: “clinically stable with good cardiopulmonary reserve; no contraindications such as recent bleeding, severe renal or liver disease, or severe thrombocytopenia (ie, < 70,000/mm3); expected to be compliant with treatment; and feels well enough to be treated at home.”3

The guideline states that various clinical decision tools, such as the Pulmonary Embolism Severity Index (PESI), can aid in identifying low-risk patients to be considered for treatment at home. The PESI uses age, gender, vital signs, mental status, and a history of cancer, lung, and cardiac disease to stratify patients by risk.4

A systematic review of 1 randomized controlled trial (RCT) and 7 observational studies found that in low-risk patients, outpatient treatment was as safe as inpatient treatment.5 This more recent study determines the net clinical benefit of hospitalized versus outpatient management in a wider range of patients with acute PE, regardless of initial risk.1

STUDY SUMMARY

Hospitalization confers no benefit to stable PE patients

This retrospective, propensity-matched cohort study compared rates of adverse events in 1127 patients with acute PE managed in the hospital versus outpatient setting.1 Patients were classified as outpatients if they were discharged from the ED or discharged from the hospital within 48 hours of admission. Patients were included if a symptomatic acute PE was diagnosed via CT or high-probability ventilation-perfusion scan and excluded if they were younger than 19, were diagnosed with a PE during hospitalization, had chronic PE, or were hemodynamically unstable, among other factors. The investigators calculated PESI scores for all patients.

Propensity scores matched patients on 28 characteristics and known risk factors for adverse events to ensure the groups were similar. The primary outcome was rate of adverse events, including recurrent VTE, major bleeding, or death at 14 days. The secondary outcome included rates of the above during the 3-month follow-up period.

Continue to: Of the 1127 eligible patients...

 

 

Of the 1127 eligible patients, 1081 were included in the matched cohort, with 576 (53%) treated as hospitalized patients and 505 (47%) treated as outpatients. The mean age of the matched cohorts was 63.2 years for the inpatient group and 63.6 years for the outpatient group. Overall, the cohorts were well matched.

The 14-day rate of adverse events was higher in hospitalized patients than in outpatients (13% vs 3.3%; odds ratio [OR], 5.07), with each of the adverse events that made up the primary outcome occurring more frequently in the hospitalized group (see Table). The rate of adverse events at 3 months was also greater for hospitalized patients compared with outpatients (21.7% vs 6.9%; OR, 4.9). The results remained similar for high-risk patients (Class III-V) based on their PESI score.

Comparison of Inpatient vs Outpatient Treatment of Patients With Pulmonary Embolism

WHAT’S NEW

Higher rate of adverse events in inpatients

This trial supports the CHEST guideline recommendations to manage hemodynamically stable patients with acute PE as outpatients.3 It adds to the conversation by demonstrating higher rates of adverse events with hospitalization, even in high-risk subgroups (PESI Class III-V).

 

CAVEATS

Good study, but it wasn’t an RCT

While this is a well-designed cohort study, it is not an RCT. This study defined outpatient management as patients discharged from the ED or hospitalized for < 48 hours. However, only 59 of the 544 patients in the outpatient group were early hospital discharges; the rest were never admitted. Finally, a specialized thrombosis clinic followed up with the patients within 24 hours of discharge, and patients had telephone access to specialized health care professionals; such organization of care contributed to the safe outpatient management of these PE patients.

CHALLENGES TO IMPLEMENTATION

Insurance coverage may present an issue

Medication coverage of direct oral anticoagulants and low-molecular-weight heparin may present a barrier to patients treated in the outpatient setting who have no insurance or are insured by certain carriers.

ACKNOWLEDGEMENT

The PURLs Surveillance System was supported in part by Grant Number UL1RR024999 from the National Center For Research Resources, a Clinical Translational Science Award to the University of Chicago. The content is solely the responsibility of the authors and does not necessarily represent the official views of the National Center For Research Resources or the National Institutes of Health.

Copyright © 2018. The Family Physicians Inquiries Network. All rights reserved.

Reprinted with permission from the Family Physicians Inquiries Network and The Journal of Family Practice (2018;67[12]:777-779).

Practice Changer

A 63-year-old woman with a history of hypertension presents to the emergency department (ED) with acute-onset shortness of breath and pleuritic chest pain after traveling across the country for a work conference. She has no history of cancer, liver disease, or renal disease. Her blood pressure is 140/80 mm Hg, and her heart rate, 90 beats/min. You diagnose an acute PE in this patient and start anticoagulation. Should you admit her to the hospital to decrease morbidity and mortality?

According to the CDC, venous thromboembolism (VTE) affects about 900,000 people each year, and about 60,000 to 100,000 of these patients die annually.2 Pulmonary embolism is the third leading cause of death from cardiovascular disease, following heart attack and stroke.3 Prompt diagnosis and treatment with systemic anticoagulation improves patient outcomes and decreases the risk for long-term complications.

The 2016 American College of Chest Physicians (CHEST) guideline on antithrombotic therapy for VTE recommends home treatment or early discharge over standard discharge (after the first 5 days of treatment) for patients who meet the following clinical criteria: “clinically stable with good cardiopulmonary reserve; no contraindications such as recent bleeding, severe renal or liver disease, or severe thrombocytopenia (ie, < 70,000/mm3); expected to be compliant with treatment; and feels well enough to be treated at home.”3

The guideline states that various clinical decision tools, such as the Pulmonary Embolism Severity Index (PESI), can aid in identifying low-risk patients to be considered for treatment at home. The PESI uses age, gender, vital signs, mental status, and a history of cancer, lung, and cardiac disease to stratify patients by risk.4

A systematic review of 1 randomized controlled trial (RCT) and 7 observational studies found that in low-risk patients, outpatient treatment was as safe as inpatient treatment.5 This more recent study determines the net clinical benefit of hospitalized versus outpatient management in a wider range of patients with acute PE, regardless of initial risk.1

STUDY SUMMARY

Hospitalization confers no benefit to stable PE patients

This retrospective, propensity-matched cohort study compared rates of adverse events in 1127 patients with acute PE managed in the hospital versus outpatient setting.1 Patients were classified as outpatients if they were discharged from the ED or discharged from the hospital within 48 hours of admission. Patients were included if a symptomatic acute PE was diagnosed via CT or high-probability ventilation-perfusion scan and excluded if they were younger than 19, were diagnosed with a PE during hospitalization, had chronic PE, or were hemodynamically unstable, among other factors. The investigators calculated PESI scores for all patients.

Propensity scores matched patients on 28 characteristics and known risk factors for adverse events to ensure the groups were similar. The primary outcome was rate of adverse events, including recurrent VTE, major bleeding, or death at 14 days. The secondary outcome included rates of the above during the 3-month follow-up period.

Continue to: Of the 1127 eligible patients...

 

 

Of the 1127 eligible patients, 1081 were included in the matched cohort, with 576 (53%) treated as hospitalized patients and 505 (47%) treated as outpatients. The mean age of the matched cohorts was 63.2 years for the inpatient group and 63.6 years for the outpatient group. Overall, the cohorts were well matched.

The 14-day rate of adverse events was higher in hospitalized patients than in outpatients (13% vs 3.3%; odds ratio [OR], 5.07), with each of the adverse events that made up the primary outcome occurring more frequently in the hospitalized group (see Table). The rate of adverse events at 3 months was also greater for hospitalized patients compared with outpatients (21.7% vs 6.9%; OR, 4.9). The results remained similar for high-risk patients (Class III-V) based on their PESI score.

Comparison of Inpatient vs Outpatient Treatment of Patients With Pulmonary Embolism

WHAT’S NEW

Higher rate of adverse events in inpatients

This trial supports the CHEST guideline recommendations to manage hemodynamically stable patients with acute PE as outpatients.3 It adds to the conversation by demonstrating higher rates of adverse events with hospitalization, even in high-risk subgroups (PESI Class III-V).

 

CAVEATS

Good study, but it wasn’t an RCT

While this is a well-designed cohort study, it is not an RCT. This study defined outpatient management as patients discharged from the ED or hospitalized for < 48 hours. However, only 59 of the 544 patients in the outpatient group were early hospital discharges; the rest were never admitted. Finally, a specialized thrombosis clinic followed up with the patients within 24 hours of discharge, and patients had telephone access to specialized health care professionals; such organization of care contributed to the safe outpatient management of these PE patients.

CHALLENGES TO IMPLEMENTATION

Insurance coverage may present an issue

Medication coverage of direct oral anticoagulants and low-molecular-weight heparin may present a barrier to patients treated in the outpatient setting who have no insurance or are insured by certain carriers.

ACKNOWLEDGEMENT

The PURLs Surveillance System was supported in part by Grant Number UL1RR024999 from the National Center For Research Resources, a Clinical Translational Science Award to the University of Chicago. The content is solely the responsibility of the authors and does not necessarily represent the official views of the National Center For Research Resources or the National Institutes of Health.

Copyright © 2018. The Family Physicians Inquiries Network. All rights reserved.

Reprinted with permission from the Family Physicians Inquiries Network and The Journal of Family Practice (2018;67[12]:777-779).

References

1. Roy PM, Corsi DJ, Carrier M, et al. Net clinical benefit of hospitalization versus outpatient management of patients with acute pulmonary embolism. J Thromb Haemost. 2017;15:685-694.
2. CDC. Venous thromboembolism data & statistics. www.cdc.gov/ncbddd/dvt/data.html. Accessed April 26, 2019.
3. Kearon C, Akl EA, Ornelas J, et al. Antithrombotic therapy for VTE disease: CHEST guideline and expert panel report. CHEST. 2016;149:315-352.
4. Aujesky D, Obrosky DS, Stone RA, et al. Derivation and validation of a prognostic model for pulmonary embolism. Am J Respir Crit Care Med. 2005;172:1041-1046.
5. Vinson DR, Zehtabchi S, Yealy DM. Can selected patients with newly diagnosed pulmonary embolism be safely treated without hospitalization? A systematic review. Ann Emerg Med. 2012;60:651-662.

References

1. Roy PM, Corsi DJ, Carrier M, et al. Net clinical benefit of hospitalization versus outpatient management of patients with acute pulmonary embolism. J Thromb Haemost. 2017;15:685-694.
2. CDC. Venous thromboembolism data & statistics. www.cdc.gov/ncbddd/dvt/data.html. Accessed April 26, 2019.
3. Kearon C, Akl EA, Ornelas J, et al. Antithrombotic therapy for VTE disease: CHEST guideline and expert panel report. CHEST. 2016;149:315-352.
4. Aujesky D, Obrosky DS, Stone RA, et al. Derivation and validation of a prognostic model for pulmonary embolism. Am J Respir Crit Care Med. 2005;172:1041-1046.
5. Vinson DR, Zehtabchi S, Yealy DM. Can selected patients with newly diagnosed pulmonary embolism be safely treated without hospitalization? A systematic review. Ann Emerg Med. 2012;60:651-662.

Issue
Clinician Reviews - 29(5)
Issue
Clinician Reviews - 29(5)
Page Number
3e-4e
Page Number
3e-4e
Publications
Publications
Topics
Article Type
Display Headline
What’s the Best Treatment Setting for Stable Pulmonary Embolism?
Display Headline
What’s the Best Treatment Setting for Stable Pulmonary Embolism?
Sections
Disallow All Ads
Content Gating
No Gating (article Unlocked/Free)
Alternative CME
Disqus Comments
Default
Use ProPublica
Hide sidebar & use full width
render the right sidebar.
Article PDF Media

What’s the best treatment setting for stable PE patients?

Article Type
Changed
Fri, 01/18/2019 - 10:45
Display Headline
What’s the best treatment setting for stable PE patients?

ILLUSTRATIVE CASE

A 63-year-old woman with a history of hypertension presents to the emergency department (ED) with acute onset shortness of breath and pleuritic chest pain after traveling across the country for a work conference. She has no history of cancer, liver disease, or renal disease. Her blood pressure is 140/80 mm Hg, and her heart rate is 90 bpm. You diagnose an acute PE in this patient and start anticoagulation. Should you admit her to the hospital to decrease morbidity and mortality?

According to the Centers for Disease Control and Prevention, venous thromboembolism (VTE) affects approximately 900,000 people each year, and approximately 60,000 to 100,000 of these patients die annually.2 Pulmonary embolism is the third leading cause of death from cardiovascular disease, following heart attacks and strokes.3 Prompt diagnosis and treatment with systemic anticoagulation improves patient outcomes and decreases the risk of long-term complications.

The 2016 American College of Chest Physicians (CHEST) guideline on antithrombotic therapy for VTE disease recommends home treatment or early discharge over standard discharge (after the first 5 days of treatment) for patients who meet the following clinical criteria: “clinically stable with good cardiopulmonary reserve; no contraindications such as recent bleeding, severe renal or liver disease, or severe thrombocytopenia (ie, <70,000/mm3); expected to be compliant with treatment; and the patient feels well enough to be treated at home.”3

The guideline states that various clinical decision tools, such as the Pulmonary Embolism Severity Index (PESI), can aid in identifying low-risk patients to be considered for treatment at home. The PESI uses age, gender, vital signs, mental status, and a history of cancer, lung, and cardiac disease to stratify patients by risk.4

A systematic review of 1 randomized controlled trial (RCT) and 7 observational studies found that in low-risk patients, outpatient treatment was as safe as inpatient treatment.5 This more recent study determines the net clinical benefit of hospitalized vs outpatient management in a wider range of patients with acute PE, regardless of initial risk.1

STUDY SUMMARY

Hospitalization confers no benefit to stable patients with acute PE

This retrospective, propensity-matched cohort study compared rates of adverse events in 1127 patients with acute PE managed in the hospital vs outpatient setting.1 Patients were classified as outpatients if they were discharged from the ED or discharged from the hospital within 48 hours of admission. Patients were included if a symptomatic acute PE was diagnosed via computed tomography scan or high-probability ventilation-perfusion scan and excluded if they were <19 years of age, diagnosed with a PE during hospitalization, had chronic PE, or were hemodynamically unstable, among other factors. The investigators calculated PESI scores for all patients.

This trial supports guideline recommendations to manage hemodynamically stable patients with acute PE as outpatients.

Propensity scores matched patients on 28 patient characteristics and known risk factors for adverse events to ensure the groups were similar. The primary outcome was rate of adverse events, including recurrent VTE, major bleeding, or death at 14 days. The secondary outcome was rates of the above during the 3-month follow-up period.

Continue to: Of the 1127 eligible patients...

 

 

Of the 1127 eligible patients, 1081 were included in the matched cohort, with 576 (53%) treated as hospitalized patients and 505 (47%) treated as outpatients. The mean age of the matched cohorts was 63.2 years for the inpatient group and 63.6 years for the outpatient group. Overall, the cohorts were well matched.

The 14-day rate of adverse events was higher in hospitalized patients than in outpatients (13% vs 3.3%; odds ratio [OR] = 5.07; 95% confidence interval [CI], 1.68-15.28), with each of the adverse events that made up the primary outcome occurring more frequently in the hospitalized group (TABLE). The rate of adverse events at 3 months was also greater for hospitalized patients compared with outpatients (21.7% vs 6.9%; OR = 4.9; 95% CI, 2.62-9.17). The results remained similar for high-risk patients (Class III-V) based on their PESI score.

Comparison of inpatient vs outpatient treatment of patients with PE

WHAT’S NEW

A higher rate of AEs in those treated as inpatients vs outpatients

This trial supports the CHEST guideline recommendations3 to manage hemodynamically stable patients with acute PE as outpatients. It adds to the conversation by demonstrating higher rates of adverse events with hospitalization, even in high-risk subgroups (PESI Class III-V).

 

CAVEATS

A good study, but it wasn’t an RCT

While this is a well-designed cohort study, it is not a randomized controlled trial (RCT). This study defined outpatient management as patients discharged from the ED or hospitalized for <48 hours. However, only 59 of the 544 patients in the outpatient group were early hospital discharges, while the rest were never admitted. Finally, a specialized thrombosis clinic followed up with the patients within 24 hours of discharge, and patients had telephone access to specialized health care professionals; such organization of care contributed to the safe outpatient management of these PE patients.

Continue to: CHALLENGES TO IMPLEMENTATION

 

 

CHALLENGES TO IMPLEMENTATION

Insurance coverage may present an issue

Medication coverage of direct oral anticoagulants and low molecular weight heparin may present a barrier to patients treated in the outpatient setting who have no insurance or are insured by certain insurance carriers.

ACKNOWLEDGEMENT

The PURLs Surveillance System was supported in part by Grant Number UL1RR024999 from the National Center For Research Resources, a Clinical Translational Science Award to the University of Chicago. The content is solely the responsibility of the authors and does not necessarily represent the official views of the National Center For Research Resources or the National Institutes of Health.

Files
References

1. Roy PM, Corsi DJ, Carrier M, et al. Net clinical benefit of hospitalization versus outpatient management of patients with acute pulmonary embolism. J Thromb Haemost. 2017;15:685-694.

2. Centers for Disease Control and Prevention. Venous Thromboembolism Data & Statistics. February 5, 2018. https://www.cdc.gov/ncbddd/dvt/data.html. Accessed July 6, 2018.

3. Kearon C, Akl EA, Ornelas J, et al. Antithrombotic therapy for VTE disease: CHEST guideline and expert panel report. CHEST. 2016;149:315-352.

4. Aujesky D, Obrosky DS, Stone RA, et al. Derivation and validation of a prognostic model for pulmonary embolism. Am J Respir Crit Care Med. 2005;172:1041-1046.

5. Vinson DR, Zehtabchi S, Yealy DM. Can selected patients with newly diagnosed pulmonary embolism be safely treated without hospitalization? A systematic review. Ann Emerg Med. 2012;60:651-662.

Article PDF
Author and Disclosure Information

UPMC St. Margaret, Pittsburgh, Pa

DEPUTY EDITOR
Anne Mounsey, MD

Department of Family Medicine, University of North Carolina, Chapel Hill

Issue
The Journal of Family Practice - 67(12)
Publications
Topics
Page Number
777-779
Sections
Files
Files
Author and Disclosure Information

UPMC St. Margaret, Pittsburgh, Pa

DEPUTY EDITOR
Anne Mounsey, MD

Department of Family Medicine, University of North Carolina, Chapel Hill

Author and Disclosure Information

UPMC St. Margaret, Pittsburgh, Pa

DEPUTY EDITOR
Anne Mounsey, MD

Department of Family Medicine, University of North Carolina, Chapel Hill

Article PDF
Article PDF

ILLUSTRATIVE CASE

A 63-year-old woman with a history of hypertension presents to the emergency department (ED) with acute onset shortness of breath and pleuritic chest pain after traveling across the country for a work conference. She has no history of cancer, liver disease, or renal disease. Her blood pressure is 140/80 mm Hg, and her heart rate is 90 bpm. You diagnose an acute PE in this patient and start anticoagulation. Should you admit her to the hospital to decrease morbidity and mortality?

According to the Centers for Disease Control and Prevention, venous thromboembolism (VTE) affects approximately 900,000 people each year, and approximately 60,000 to 100,000 of these patients die annually.2 Pulmonary embolism is the third leading cause of death from cardiovascular disease, following heart attacks and strokes.3 Prompt diagnosis and treatment with systemic anticoagulation improves patient outcomes and decreases the risk of long-term complications.

The 2016 American College of Chest Physicians (CHEST) guideline on antithrombotic therapy for VTE disease recommends home treatment or early discharge over standard discharge (after the first 5 days of treatment) for patients who meet the following clinical criteria: “clinically stable with good cardiopulmonary reserve; no contraindications such as recent bleeding, severe renal or liver disease, or severe thrombocytopenia (ie, <70,000/mm3); expected to be compliant with treatment; and the patient feels well enough to be treated at home.”3

The guideline states that various clinical decision tools, such as the Pulmonary Embolism Severity Index (PESI), can aid in identifying low-risk patients to be considered for treatment at home. The PESI uses age, gender, vital signs, mental status, and a history of cancer, lung, and cardiac disease to stratify patients by risk.4

A systematic review of 1 randomized controlled trial (RCT) and 7 observational studies found that in low-risk patients, outpatient treatment was as safe as inpatient treatment.5 This more recent study determines the net clinical benefit of hospitalized vs outpatient management in a wider range of patients with acute PE, regardless of initial risk.1

STUDY SUMMARY

Hospitalization confers no benefit to stable patients with acute PE

This retrospective, propensity-matched cohort study compared rates of adverse events in 1127 patients with acute PE managed in the hospital vs outpatient setting.1 Patients were classified as outpatients if they were discharged from the ED or discharged from the hospital within 48 hours of admission. Patients were included if a symptomatic acute PE was diagnosed via computed tomography scan or high-probability ventilation-perfusion scan and excluded if they were <19 years of age, diagnosed with a PE during hospitalization, had chronic PE, or were hemodynamically unstable, among other factors. The investigators calculated PESI scores for all patients.

This trial supports guideline recommendations to manage hemodynamically stable patients with acute PE as outpatients.

Propensity scores matched patients on 28 patient characteristics and known risk factors for adverse events to ensure the groups were similar. The primary outcome was rate of adverse events, including recurrent VTE, major bleeding, or death at 14 days. The secondary outcome was rates of the above during the 3-month follow-up period.

Continue to: Of the 1127 eligible patients...

 

 

Of the 1127 eligible patients, 1081 were included in the matched cohort, with 576 (53%) treated as hospitalized patients and 505 (47%) treated as outpatients. The mean age of the matched cohorts was 63.2 years for the inpatient group and 63.6 years for the outpatient group. Overall, the cohorts were well matched.

The 14-day rate of adverse events was higher in hospitalized patients than in outpatients (13% vs 3.3%; odds ratio [OR] = 5.07; 95% confidence interval [CI], 1.68-15.28), with each of the adverse events that made up the primary outcome occurring more frequently in the hospitalized group (TABLE). The rate of adverse events at 3 months was also greater for hospitalized patients compared with outpatients (21.7% vs 6.9%; OR = 4.9; 95% CI, 2.62-9.17). The results remained similar for high-risk patients (Class III-V) based on their PESI score.

Comparison of inpatient vs outpatient treatment of patients with PE

WHAT’S NEW

A higher rate of AEs in those treated as inpatients vs outpatients

This trial supports the CHEST guideline recommendations3 to manage hemodynamically stable patients with acute PE as outpatients. It adds to the conversation by demonstrating higher rates of adverse events with hospitalization, even in high-risk subgroups (PESI Class III-V).

 

CAVEATS

A good study, but it wasn’t an RCT

While this is a well-designed cohort study, it is not a randomized controlled trial (RCT). This study defined outpatient management as patients discharged from the ED or hospitalized for <48 hours. However, only 59 of the 544 patients in the outpatient group were early hospital discharges, while the rest were never admitted. Finally, a specialized thrombosis clinic followed up with the patients within 24 hours of discharge, and patients had telephone access to specialized health care professionals; such organization of care contributed to the safe outpatient management of these PE patients.

Continue to: CHALLENGES TO IMPLEMENTATION

 

 

CHALLENGES TO IMPLEMENTATION

Insurance coverage may present an issue

Medication coverage of direct oral anticoagulants and low molecular weight heparin may present a barrier to patients treated in the outpatient setting who have no insurance or are insured by certain insurance carriers.

ACKNOWLEDGEMENT

The PURLs Surveillance System was supported in part by Grant Number UL1RR024999 from the National Center For Research Resources, a Clinical Translational Science Award to the University of Chicago. The content is solely the responsibility of the authors and does not necessarily represent the official views of the National Center For Research Resources or the National Institutes of Health.

ILLUSTRATIVE CASE

A 63-year-old woman with a history of hypertension presents to the emergency department (ED) with acute onset shortness of breath and pleuritic chest pain after traveling across the country for a work conference. She has no history of cancer, liver disease, or renal disease. Her blood pressure is 140/80 mm Hg, and her heart rate is 90 bpm. You diagnose an acute PE in this patient and start anticoagulation. Should you admit her to the hospital to decrease morbidity and mortality?

According to the Centers for Disease Control and Prevention, venous thromboembolism (VTE) affects approximately 900,000 people each year, and approximately 60,000 to 100,000 of these patients die annually.2 Pulmonary embolism is the third leading cause of death from cardiovascular disease, following heart attacks and strokes.3 Prompt diagnosis and treatment with systemic anticoagulation improves patient outcomes and decreases the risk of long-term complications.

The 2016 American College of Chest Physicians (CHEST) guideline on antithrombotic therapy for VTE disease recommends home treatment or early discharge over standard discharge (after the first 5 days of treatment) for patients who meet the following clinical criteria: “clinically stable with good cardiopulmonary reserve; no contraindications such as recent bleeding, severe renal or liver disease, or severe thrombocytopenia (ie, <70,000/mm3); expected to be compliant with treatment; and the patient feels well enough to be treated at home.”3

The guideline states that various clinical decision tools, such as the Pulmonary Embolism Severity Index (PESI), can aid in identifying low-risk patients to be considered for treatment at home. The PESI uses age, gender, vital signs, mental status, and a history of cancer, lung, and cardiac disease to stratify patients by risk.4

A systematic review of 1 randomized controlled trial (RCT) and 7 observational studies found that in low-risk patients, outpatient treatment was as safe as inpatient treatment.5 This more recent study determines the net clinical benefit of hospitalized vs outpatient management in a wider range of patients with acute PE, regardless of initial risk.1

STUDY SUMMARY

Hospitalization confers no benefit to stable patients with acute PE

This retrospective, propensity-matched cohort study compared rates of adverse events in 1127 patients with acute PE managed in the hospital vs outpatient setting.1 Patients were classified as outpatients if they were discharged from the ED or discharged from the hospital within 48 hours of admission. Patients were included if a symptomatic acute PE was diagnosed via computed tomography scan or high-probability ventilation-perfusion scan and excluded if they were <19 years of age, diagnosed with a PE during hospitalization, had chronic PE, or were hemodynamically unstable, among other factors. The investigators calculated PESI scores for all patients.

This trial supports guideline recommendations to manage hemodynamically stable patients with acute PE as outpatients.

Propensity scores matched patients on 28 patient characteristics and known risk factors for adverse events to ensure the groups were similar. The primary outcome was rate of adverse events, including recurrent VTE, major bleeding, or death at 14 days. The secondary outcome was rates of the above during the 3-month follow-up period.

Continue to: Of the 1127 eligible patients...

 

 

Of the 1127 eligible patients, 1081 were included in the matched cohort, with 576 (53%) treated as hospitalized patients and 505 (47%) treated as outpatients. The mean age of the matched cohorts was 63.2 years for the inpatient group and 63.6 years for the outpatient group. Overall, the cohorts were well matched.

The 14-day rate of adverse events was higher in hospitalized patients than in outpatients (13% vs 3.3%; odds ratio [OR] = 5.07; 95% confidence interval [CI], 1.68-15.28), with each of the adverse events that made up the primary outcome occurring more frequently in the hospitalized group (TABLE). The rate of adverse events at 3 months was also greater for hospitalized patients compared with outpatients (21.7% vs 6.9%; OR = 4.9; 95% CI, 2.62-9.17). The results remained similar for high-risk patients (Class III-V) based on their PESI score.

Comparison of inpatient vs outpatient treatment of patients with PE

WHAT’S NEW

A higher rate of AEs in those treated as inpatients vs outpatients

This trial supports the CHEST guideline recommendations3 to manage hemodynamically stable patients with acute PE as outpatients. It adds to the conversation by demonstrating higher rates of adverse events with hospitalization, even in high-risk subgroups (PESI Class III-V).

 

CAVEATS

A good study, but it wasn’t an RCT

While this is a well-designed cohort study, it is not a randomized controlled trial (RCT). This study defined outpatient management as patients discharged from the ED or hospitalized for <48 hours. However, only 59 of the 544 patients in the outpatient group were early hospital discharges, while the rest were never admitted. Finally, a specialized thrombosis clinic followed up with the patients within 24 hours of discharge, and patients had telephone access to specialized health care professionals; such organization of care contributed to the safe outpatient management of these PE patients.

Continue to: CHALLENGES TO IMPLEMENTATION

 

 

CHALLENGES TO IMPLEMENTATION

Insurance coverage may present an issue

Medication coverage of direct oral anticoagulants and low molecular weight heparin may present a barrier to patients treated in the outpatient setting who have no insurance or are insured by certain insurance carriers.

ACKNOWLEDGEMENT

The PURLs Surveillance System was supported in part by Grant Number UL1RR024999 from the National Center For Research Resources, a Clinical Translational Science Award to the University of Chicago. The content is solely the responsibility of the authors and does not necessarily represent the official views of the National Center For Research Resources or the National Institutes of Health.

References

1. Roy PM, Corsi DJ, Carrier M, et al. Net clinical benefit of hospitalization versus outpatient management of patients with acute pulmonary embolism. J Thromb Haemost. 2017;15:685-694.

2. Centers for Disease Control and Prevention. Venous Thromboembolism Data & Statistics. February 5, 2018. https://www.cdc.gov/ncbddd/dvt/data.html. Accessed July 6, 2018.

3. Kearon C, Akl EA, Ornelas J, et al. Antithrombotic therapy for VTE disease: CHEST guideline and expert panel report. CHEST. 2016;149:315-352.

4. Aujesky D, Obrosky DS, Stone RA, et al. Derivation and validation of a prognostic model for pulmonary embolism. Am J Respir Crit Care Med. 2005;172:1041-1046.

5. Vinson DR, Zehtabchi S, Yealy DM. Can selected patients with newly diagnosed pulmonary embolism be safely treated without hospitalization? A systematic review. Ann Emerg Med. 2012;60:651-662.

References

1. Roy PM, Corsi DJ, Carrier M, et al. Net clinical benefit of hospitalization versus outpatient management of patients with acute pulmonary embolism. J Thromb Haemost. 2017;15:685-694.

2. Centers for Disease Control and Prevention. Venous Thromboembolism Data & Statistics. February 5, 2018. https://www.cdc.gov/ncbddd/dvt/data.html. Accessed July 6, 2018.

3. Kearon C, Akl EA, Ornelas J, et al. Antithrombotic therapy for VTE disease: CHEST guideline and expert panel report. CHEST. 2016;149:315-352.

4. Aujesky D, Obrosky DS, Stone RA, et al. Derivation and validation of a prognostic model for pulmonary embolism. Am J Respir Crit Care Med. 2005;172:1041-1046.

5. Vinson DR, Zehtabchi S, Yealy DM. Can selected patients with newly diagnosed pulmonary embolism be safely treated without hospitalization? A systematic review. Ann Emerg Med. 2012;60:651-662.

Issue
The Journal of Family Practice - 67(12)
Issue
The Journal of Family Practice - 67(12)
Page Number
777-779
Page Number
777-779
Publications
Publications
Topics
Article Type
Display Headline
What’s the best treatment setting for stable PE patients?
Display Headline
What’s the best treatment setting for stable PE patients?
Sections
PURLs Copyright
Copyright © 2018. The Family Physicians Inquiries Network. All rights reserved.
Inside the Article

PRACTICE CHANGER

Manage patients with acute pulmonary embolism (PE) who are hemodynamically stable in the outpatient setting to decrease adverse events—regardless of their initial risk category.1

STRENGTH OF RECOMMENDATION

B: Based upon a good-quality retrospective cohort propensity score analysis.

Roy PM, Corsi DJ, Carrier M, et al. Net clinical benefit of hospitalization versus outpatient management of patients with acute pulmonary embolism. J Thromb Haemost. 2017;15:685-694.

Disallow All Ads
Content Gating
No Gating (article Unlocked/Free)
Alternative CME
PubMed ID
30566115
Disqus Comments
Default
Use ProPublica
Article PDF Media
Media Files

The (Sterile) Gloves Are Coming Off

Article Type
Changed
Thu, 03/28/2019 - 14:33
Display Headline
The (Sterile) Gloves Are Coming Off

Practice Changer

Your practice manager, on a quest to reduce expenses, asks whether your practice could reduce the amount of money spent on gloves for procedures. How do you reply?

The effect of a small difference spread over a large number of events can be sizable. For example, the added cost of using sterile, as opposed to nonsterile, gloves for minor procedures is relatively modest and certainly worthwhile if the sterile gloves reduce the number of surgical site infections (SSIs). However, if there is no difference in SSIs, the extra cost becomes a large unnecessary expense, given the volume of minor procedures performed.

The decision to use sterile gloves often stems from habit, product availability, or perceived benefit of fewer SSIs.2 Providers’ choice of gloves varies widely, despite some evidence comparing sterile and nonsterile gloves.3-5

STUDY SUMMARY

Sterile no better than nonsterile gloves

This systematic review and meta-analysis of 13 RCTs and observational (prospective or retrospective) studies compared infection rates using sterile versus nonsterile gloves in 11,071 unique patients. The methods used in the review followed the Cochrane collaboration guidelines.6 Patients included in each study underwent outpatient cutaneous or mucosal surgical procedures, including laceration repair, standard excisions, Mohs micrographic surgery, or tooth extractions. In addition to glove type, documentation of postoperative SSI was necessary for inclusion.

 

Methodology. A total of 512 publications were reviewed for inclusion; 14 met the criteria but one study was removed due to incomplete data, leaving 13 trials with a total of 11,071 patients for the analysis. In the RCTs, 1,360 patients were randomly assigned to treatment with sterile gloves and 1,381 to treatment with nonsterile gloves as the intervention. In the prospective or retrospective observational trials, 4,680 patients were treated with sterile gloves, and 3,650 were treated with nonsterile gloves. Heterogeneity was low. Of note, the researchers performed a subgroup analysis on nine studies (4 RCTs and 5 observational studies) involving only cutaneous surgeries; these represented procedures most likely performed in the primary care setting.

The primary outcome of this review was postoperative wound infection. The results did not show any difference in SSIs between sterile and nonsterile gloves in all trials (2% vs 2.1%; relative risk [RR], 1.06). There was also no difference in infection rate in the subgroup analysis (2.2% vs 2.2%, respectively; RR, 1.02) or an analysis limited to only RCTs.

WHAT’S NEW

Highest-quality evidence shows no difference

This systematic review found no difference in SSI rates when using sterile versus nonsterile gloves. Given that the analysis represents the highest-quality level of evidence (a systematic review of RCTs) and that sterile gloves are several times more expensive per pair than nonsterile gloves, the findings should impact future practice.

Continue to: CAVEATS

 

 

CAVEATS

Risk for bias and limited applicability

Not every trial in this meta-analysis was an RCT, and the inclusion of observational studies increases the risk for bias. However, the results of the observational studies were similar to those of the RCTs, somewhat alleviating this potential threat to validity.

It is worth noting that more extensive surgeries and more complicated repairs were not included in the trials, meaning that the findings are limited to oral surgery, Mohs micrographic surgery, standard incisions, and laceration repairs.

CHALLENGES TO IMPLEMENTATION

Inertia, medicolegal concerns, and personal preference

Clinical inertia may lead to slow adoption of these recommendations. Providers may worry about potential medicolegal ramifications from this change.1 Lastly, some providers may prefer the fit and feel of sterile gloves for their procedures.

 

ACKNOWLEDGEMENT

The PURLs Surveillance System was supported in part by Grant Number UL1RR024999 from the National Center For Research Resources, a Clinical Translational Science Award to the University of Chicago. The content is solely the responsibility of the authors and does not necessarily represent the official views of the National Center For Research Resources or the National Institutes of Health.

Copyright © 2018. The Family Physicians Inquiries Network. All rights reserved.

Reprinted with permission from the Family Physicians Inquiries Network and The Journal of Family Practice (2018; 67[8]:507-508).

References

1. Brewer JD, Gonzalez AB, Baum CL, et al. Comparison of sterile vs nonsterile gloves in cutaneous surgery and common outpatient dental procedures: a systematic review and meta-analysis. JAMA Dermatol. 2016; 152(9): 1008-1014.
2. Creamer J, Davis K, Rice W. Sterile gloves: do they make a difference? Am J Surg. 2012;204(6):976-979.
3. Heal C, Sriharan S, Buttner PG, et al. Comparing non-sterile to sterile gloves for minor surgery: a prospective randomised controlled non-inferiority trial. Med J Aust. 2015(1);202:27-31.
4. Ghafouri HB, Zoofaghari SJ, Kasnavieh MH, et al. A pilot study on the repair of contaminated traumatic wounds in the emergency department using sterile versus non-sterile gloves. Hong Kong J Emerg Med. 2014;21(3):148-152.
5. Rogers HD, Desciak EB, Marcus RP, et al. Prospective study of wound infections in Mohs micrographic surgery using clean surgical technique in the absence of prophylactic antibiotics. J Am Acad Dermatol. 2010;63:842-851.
6. Cochrane Methods. London, UK: The Cochrane Collaboration. 2018. http://methods.cochrane.org/. Accessed August 24, 2018.

Article PDF
Author and Disclosure Information

Gregory Castelli and Mary Pat Friedlander practice at University of Pittsburgh Medical Center St. Margaret in Pennsylvania.

Issue
Clinician Reviews - 28(9)
Publications
Topics
Page Number
e1-e2
Sections
Author and Disclosure Information

Gregory Castelli and Mary Pat Friedlander practice at University of Pittsburgh Medical Center St. Margaret in Pennsylvania.

Author and Disclosure Information

Gregory Castelli and Mary Pat Friedlander practice at University of Pittsburgh Medical Center St. Margaret in Pennsylvania.

Article PDF
Article PDF

Practice Changer

Your practice manager, on a quest to reduce expenses, asks whether your practice could reduce the amount of money spent on gloves for procedures. How do you reply?

The effect of a small difference spread over a large number of events can be sizable. For example, the added cost of using sterile, as opposed to nonsterile, gloves for minor procedures is relatively modest and certainly worthwhile if the sterile gloves reduce the number of surgical site infections (SSIs). However, if there is no difference in SSIs, the extra cost becomes a large unnecessary expense, given the volume of minor procedures performed.

The decision to use sterile gloves often stems from habit, product availability, or perceived benefit of fewer SSIs.2 Providers’ choice of gloves varies widely, despite some evidence comparing sterile and nonsterile gloves.3-5

STUDY SUMMARY

Sterile no better than nonsterile gloves

This systematic review and meta-analysis of 13 RCTs and observational (prospective or retrospective) studies compared infection rates using sterile versus nonsterile gloves in 11,071 unique patients. The methods used in the review followed the Cochrane collaboration guidelines.6 Patients included in each study underwent outpatient cutaneous or mucosal surgical procedures, including laceration repair, standard excisions, Mohs micrographic surgery, or tooth extractions. In addition to glove type, documentation of postoperative SSI was necessary for inclusion.

 

Methodology. A total of 512 publications were reviewed for inclusion; 14 met the criteria but one study was removed due to incomplete data, leaving 13 trials with a total of 11,071 patients for the analysis. In the RCTs, 1,360 patients were randomly assigned to treatment with sterile gloves and 1,381 to treatment with nonsterile gloves as the intervention. In the prospective or retrospective observational trials, 4,680 patients were treated with sterile gloves, and 3,650 were treated with nonsterile gloves. Heterogeneity was low. Of note, the researchers performed a subgroup analysis on nine studies (4 RCTs and 5 observational studies) involving only cutaneous surgeries; these represented procedures most likely performed in the primary care setting.

The primary outcome of this review was postoperative wound infection. The results did not show any difference in SSIs between sterile and nonsterile gloves in all trials (2% vs 2.1%; relative risk [RR], 1.06). There was also no difference in infection rate in the subgroup analysis (2.2% vs 2.2%, respectively; RR, 1.02) or an analysis limited to only RCTs.

WHAT’S NEW

Highest-quality evidence shows no difference

This systematic review found no difference in SSI rates when using sterile versus nonsterile gloves. Given that the analysis represents the highest-quality level of evidence (a systematic review of RCTs) and that sterile gloves are several times more expensive per pair than nonsterile gloves, the findings should impact future practice.

Continue to: CAVEATS

 

 

CAVEATS

Risk for bias and limited applicability

Not every trial in this meta-analysis was an RCT, and the inclusion of observational studies increases the risk for bias. However, the results of the observational studies were similar to those of the RCTs, somewhat alleviating this potential threat to validity.

It is worth noting that more extensive surgeries and more complicated repairs were not included in the trials, meaning that the findings are limited to oral surgery, Mohs micrographic surgery, standard incisions, and laceration repairs.

CHALLENGES TO IMPLEMENTATION

Inertia, medicolegal concerns, and personal preference

Clinical inertia may lead to slow adoption of these recommendations. Providers may worry about potential medicolegal ramifications from this change.1 Lastly, some providers may prefer the fit and feel of sterile gloves for their procedures.

 

ACKNOWLEDGEMENT

The PURLs Surveillance System was supported in part by Grant Number UL1RR024999 from the National Center For Research Resources, a Clinical Translational Science Award to the University of Chicago. The content is solely the responsibility of the authors and does not necessarily represent the official views of the National Center For Research Resources or the National Institutes of Health.

Copyright © 2018. The Family Physicians Inquiries Network. All rights reserved.

Reprinted with permission from the Family Physicians Inquiries Network and The Journal of Family Practice (2018; 67[8]:507-508).

Practice Changer

Your practice manager, on a quest to reduce expenses, asks whether your practice could reduce the amount of money spent on gloves for procedures. How do you reply?

The effect of a small difference spread over a large number of events can be sizable. For example, the added cost of using sterile, as opposed to nonsterile, gloves for minor procedures is relatively modest and certainly worthwhile if the sterile gloves reduce the number of surgical site infections (SSIs). However, if there is no difference in SSIs, the extra cost becomes a large unnecessary expense, given the volume of minor procedures performed.

The decision to use sterile gloves often stems from habit, product availability, or perceived benefit of fewer SSIs.2 Providers’ choice of gloves varies widely, despite some evidence comparing sterile and nonsterile gloves.3-5

STUDY SUMMARY

Sterile no better than nonsterile gloves

This systematic review and meta-analysis of 13 RCTs and observational (prospective or retrospective) studies compared infection rates using sterile versus nonsterile gloves in 11,071 unique patients. The methods used in the review followed the Cochrane collaboration guidelines.6 Patients included in each study underwent outpatient cutaneous or mucosal surgical procedures, including laceration repair, standard excisions, Mohs micrographic surgery, or tooth extractions. In addition to glove type, documentation of postoperative SSI was necessary for inclusion.

 

Methodology. A total of 512 publications were reviewed for inclusion; 14 met the criteria but one study was removed due to incomplete data, leaving 13 trials with a total of 11,071 patients for the analysis. In the RCTs, 1,360 patients were randomly assigned to treatment with sterile gloves and 1,381 to treatment with nonsterile gloves as the intervention. In the prospective or retrospective observational trials, 4,680 patients were treated with sterile gloves, and 3,650 were treated with nonsterile gloves. Heterogeneity was low. Of note, the researchers performed a subgroup analysis on nine studies (4 RCTs and 5 observational studies) involving only cutaneous surgeries; these represented procedures most likely performed in the primary care setting.

The primary outcome of this review was postoperative wound infection. The results did not show any difference in SSIs between sterile and nonsterile gloves in all trials (2% vs 2.1%; relative risk [RR], 1.06). There was also no difference in infection rate in the subgroup analysis (2.2% vs 2.2%, respectively; RR, 1.02) or an analysis limited to only RCTs.

WHAT’S NEW

Highest-quality evidence shows no difference

This systematic review found no difference in SSI rates when using sterile versus nonsterile gloves. Given that the analysis represents the highest-quality level of evidence (a systematic review of RCTs) and that sterile gloves are several times more expensive per pair than nonsterile gloves, the findings should impact future practice.

Continue to: CAVEATS

 

 

CAVEATS

Risk for bias and limited applicability

Not every trial in this meta-analysis was an RCT, and the inclusion of observational studies increases the risk for bias. However, the results of the observational studies were similar to those of the RCTs, somewhat alleviating this potential threat to validity.

It is worth noting that more extensive surgeries and more complicated repairs were not included in the trials, meaning that the findings are limited to oral surgery, Mohs micrographic surgery, standard incisions, and laceration repairs.

CHALLENGES TO IMPLEMENTATION

Inertia, medicolegal concerns, and personal preference

Clinical inertia may lead to slow adoption of these recommendations. Providers may worry about potential medicolegal ramifications from this change.1 Lastly, some providers may prefer the fit and feel of sterile gloves for their procedures.

 

ACKNOWLEDGEMENT

The PURLs Surveillance System was supported in part by Grant Number UL1RR024999 from the National Center For Research Resources, a Clinical Translational Science Award to the University of Chicago. The content is solely the responsibility of the authors and does not necessarily represent the official views of the National Center For Research Resources or the National Institutes of Health.

Copyright © 2018. The Family Physicians Inquiries Network. All rights reserved.

Reprinted with permission from the Family Physicians Inquiries Network and The Journal of Family Practice (2018; 67[8]:507-508).

References

1. Brewer JD, Gonzalez AB, Baum CL, et al. Comparison of sterile vs nonsterile gloves in cutaneous surgery and common outpatient dental procedures: a systematic review and meta-analysis. JAMA Dermatol. 2016; 152(9): 1008-1014.
2. Creamer J, Davis K, Rice W. Sterile gloves: do they make a difference? Am J Surg. 2012;204(6):976-979.
3. Heal C, Sriharan S, Buttner PG, et al. Comparing non-sterile to sterile gloves for minor surgery: a prospective randomised controlled non-inferiority trial. Med J Aust. 2015(1);202:27-31.
4. Ghafouri HB, Zoofaghari SJ, Kasnavieh MH, et al. A pilot study on the repair of contaminated traumatic wounds in the emergency department using sterile versus non-sterile gloves. Hong Kong J Emerg Med. 2014;21(3):148-152.
5. Rogers HD, Desciak EB, Marcus RP, et al. Prospective study of wound infections in Mohs micrographic surgery using clean surgical technique in the absence of prophylactic antibiotics. J Am Acad Dermatol. 2010;63:842-851.
6. Cochrane Methods. London, UK: The Cochrane Collaboration. 2018. http://methods.cochrane.org/. Accessed August 24, 2018.

References

1. Brewer JD, Gonzalez AB, Baum CL, et al. Comparison of sterile vs nonsterile gloves in cutaneous surgery and common outpatient dental procedures: a systematic review and meta-analysis. JAMA Dermatol. 2016; 152(9): 1008-1014.
2. Creamer J, Davis K, Rice W. Sterile gloves: do they make a difference? Am J Surg. 2012;204(6):976-979.
3. Heal C, Sriharan S, Buttner PG, et al. Comparing non-sterile to sterile gloves for minor surgery: a prospective randomised controlled non-inferiority trial. Med J Aust. 2015(1);202:27-31.
4. Ghafouri HB, Zoofaghari SJ, Kasnavieh MH, et al. A pilot study on the repair of contaminated traumatic wounds in the emergency department using sterile versus non-sterile gloves. Hong Kong J Emerg Med. 2014;21(3):148-152.
5. Rogers HD, Desciak EB, Marcus RP, et al. Prospective study of wound infections in Mohs micrographic surgery using clean surgical technique in the absence of prophylactic antibiotics. J Am Acad Dermatol. 2010;63:842-851.
6. Cochrane Methods. London, UK: The Cochrane Collaboration. 2018. http://methods.cochrane.org/. Accessed August 24, 2018.

Issue
Clinician Reviews - 28(9)
Issue
Clinician Reviews - 28(9)
Page Number
e1-e2
Page Number
e1-e2
Publications
Publications
Topics
Article Type
Display Headline
The (Sterile) Gloves Are Coming Off
Display Headline
The (Sterile) Gloves Are Coming Off
Sections
Disallow All Ads
Content Gating
No Gating (article Unlocked/Free)
Alternative CME
Disqus Comments
Default
Use ProPublica
Article PDF Media

Time to switch to nonsterile gloves for these procedures?

Article Type
Changed
Thu, 03/28/2019 - 14:35
Display Headline
Time to switch to nonsterile gloves for these procedures?

ILLUSTRATIVE CASE

Your practice manager comes to you to discuss ways that you can reduce expenses. He asks whether the practice could reduce the amount of money spent on gloves for procedures. How do you reply?

A decision involving a small difference, spread over a larger number of events, can have a sizable effect. An example is whether to use sterile vs nonsterile gloves for minor procedures. The cost difference between a box of sterile gloves and a box of nonsterile gloves is relatively small, and certainly worth the difference if the more expensive sterile gloves reduce the number of surgical site infections (SSIs).

However, if there is no difference in the number of SSIs, there may be no value to the extra cost, which, given the number of such procedures, becomes a large unnecessary expense. The choice to use sterile gloves often stems from habit, product availability, or the perceived benefit of fewer SSIs.2 While some evidence exists comparing glove choice, there is wide variability in physicians’ choice of gloves.3-5 This large systematic review compared rates of SSIs using sterile vs nonsterile gloves.

STUDY SUMMARY

RCTs/observational studies find sterile no better than nonsterile gloves

This systematic review and meta-analysis of 13 randomized controlled trials (RCTs) and observational (prospective or retrospective) studies compared infection rates using sterile vs nonsterile gloves in 11,071 unique patients undergoing cutaneous surgery, including Mohs microsurgery or outpatient dental procedures. The methods used in the review followed the Cochrane collaboration guidelines.6 The inclusion criteria were that the studies had to be either RCTs or observational studies. Patients included in each study underwent outpatient cutaneous or mucosal surgical procedures, including laceration repair, standard excisions, Mohs micrographic surgery, or tooth extractions. In addition to glove type, documentation of postoperative SSI was necessary for inclusion.

Methodology. The authors of the analysis reviewed a total of 512 publications for inclusion; of these, 14 met the inclusion criteria. One study was later removed due to incomplete data, leaving a total of 13 trials for the analysis. Of the 11,071 patients included in the final analysis, 1360 patients were randomly assigned to treatment with sterile gloves, while 1381 patients were assigned to treatment with nonsterile gloves as the intervention in a clinical trial. The remaining patients participated in either prospective or retrospective observational trials; 4680 patients were treated with sterile gloves, and 3650 patients were treated with nonsterile gloves. Heterogeneity was low for the included studies. Of note, the researchers performed a subgroup analysis on 9 total studies (4 RCTs and 5 observational studies) involving cutaneous surgeries only. These represented procedures most likely performed in the primary care setting.

 

The primary outcome of this review was postoperative wound infection. The results did not show any difference in SSIs between sterile vs nonsterile gloves in all trials (2% vs 2.1%; relative risk [RR]=1.06; 95% confidence interval [CI], 0.81-1.39). There was also no difference in infection rates in the subgroup analysis of 9 trials limited to cutaneous surgery (2.2% vs 2.2%, respectively; RR=1.02; 95% CI, 0.78-1.34) or when the analysis was limited to only RCTs.

[polldaddy:10063798]

Continue to: WHAT'S NEW

 

 

WHAT’S NEW

Highest-quality evidence shows no difference in SSIs

This systematic review found no difference in SSI rates when using sterile vs nonsterile gloves. Given that the analysis represents the highest-quality level of evidence (a systematic review of RCTs) and that sterile gloves are several times more expensive per pair than nonsterile gloves, the findings should impact future practice.

CAVEATS

A risk of bias and limited applicability

Not every trial in this meta-analysis was an RCT, and the inclusion of observational studies increases the risk of bias. However, the results of the observational studies were similar to those of the RCTs, somewhat alleviating this potential threat to validity.

The results did not show any difference in surgical site infections between sterile and nonsterile gloves.

It is worth noting that more extensive surgeries and more complicated repairs were not included in the trials, meaning that the findings are limited to oral surgery, Mohs micrographic surgery, standard incisions, and laceration repairs.

 

CHALLENGES TO IMPLEMENTATION

Inertia, medicolegal concerns, and personal preference

Clinical inertia may lead to slow adoption of these recommendations. Physicians may worry about potential medicolegal ramifications from this change.1 Lastly, some physicians may prefer the fit and feel of sterile gloves for their procedures.

ACKNOWLEDGEMENT

The PURLs Surveillance System was supported in part by Grant Number UL1RR024999 from the National Center For Research Resources, a Clinical Translational Science Award to the University of Chicago. The content is solely the responsibility of the authors and does not necessarily represent the official views of the National Center For Research Resources or the National Institutes of Health.

Files
References

1. Brewer JD, Gonzalez AB, Baum CL, et al. Comparison of sterile vs nonsterile gloves in cutaneous surgery and common outpatient dental procedures: a systematic review and meta-analysis. JAMA Dermatol. 2016;152:1008-1014.

2. Creamer J, Davis K, Rice W. Sterile gloves: do they make a difference? Am J Surg. 2012;204:976-979.

3. Heal C, Sriharan S, Buttner PG, et al. Comparing non-sterile to sterile gloves for minor surgery: a prospective randomised controlled non-inferiority trial. Med J Aust. 2015;202:27-31.

4. Ghafouri HB, Zoofaghari SJ, Kasnavieh MH, et al. A pilot study on the repair of contaminated traumatic wounds in the emergency department using sterile versus non-sterile gloves. Hong Kong J Emerg Med. 2014;21:148-152.

5. Rogers HD, Desciak EB, Marcus RP, et al. Prospective study of wound infections in Mohs micrographic surgery using clean surgical technique in the absence of prophylactic antibiotics. J Am Acad Dermatol. 2010;63:842-851.

6. Cochrane Methods. London, UK: The Cochrane Collaboration. 2018. Available at: http://methods.cochrane.org/. Accessed July 15, 2018.

Article PDF
Author and Disclosure Information

UPMC St. Margaret, Pittsburgh, Pa

DEPUTY EDITOR
James J. Stevermer, MD, MSPH

Department of Family and Community Medicine, University of Missouri-Columbia

Issue
The Journal of Family Practice - 67(8)
Publications
Topics
Page Number
507-508
Sections
Files
Files
Author and Disclosure Information

UPMC St. Margaret, Pittsburgh, Pa

DEPUTY EDITOR
James J. Stevermer, MD, MSPH

Department of Family and Community Medicine, University of Missouri-Columbia

Author and Disclosure Information

UPMC St. Margaret, Pittsburgh, Pa

DEPUTY EDITOR
James J. Stevermer, MD, MSPH

Department of Family and Community Medicine, University of Missouri-Columbia

Article PDF
Article PDF

ILLUSTRATIVE CASE

Your practice manager comes to you to discuss ways that you can reduce expenses. He asks whether the practice could reduce the amount of money spent on gloves for procedures. How do you reply?

A decision involving a small difference, spread over a larger number of events, can have a sizable effect. An example is whether to use sterile vs nonsterile gloves for minor procedures. The cost difference between a box of sterile gloves and a box of nonsterile gloves is relatively small, and certainly worth the difference if the more expensive sterile gloves reduce the number of surgical site infections (SSIs).

However, if there is no difference in the number of SSIs, there may be no value to the extra cost, which, given the number of such procedures, becomes a large unnecessary expense. The choice to use sterile gloves often stems from habit, product availability, or the perceived benefit of fewer SSIs.2 While some evidence exists comparing glove choice, there is wide variability in physicians’ choice of gloves.3-5 This large systematic review compared rates of SSIs using sterile vs nonsterile gloves.

STUDY SUMMARY

RCTs/observational studies find sterile no better than nonsterile gloves

This systematic review and meta-analysis of 13 randomized controlled trials (RCTs) and observational (prospective or retrospective) studies compared infection rates using sterile vs nonsterile gloves in 11,071 unique patients undergoing cutaneous surgery, including Mohs microsurgery or outpatient dental procedures. The methods used in the review followed the Cochrane collaboration guidelines.6 The inclusion criteria were that the studies had to be either RCTs or observational studies. Patients included in each study underwent outpatient cutaneous or mucosal surgical procedures, including laceration repair, standard excisions, Mohs micrographic surgery, or tooth extractions. In addition to glove type, documentation of postoperative SSI was necessary for inclusion.

Methodology. The authors of the analysis reviewed a total of 512 publications for inclusion; of these, 14 met the inclusion criteria. One study was later removed due to incomplete data, leaving a total of 13 trials for the analysis. Of the 11,071 patients included in the final analysis, 1360 patients were randomly assigned to treatment with sterile gloves, while 1381 patients were assigned to treatment with nonsterile gloves as the intervention in a clinical trial. The remaining patients participated in either prospective or retrospective observational trials; 4680 patients were treated with sterile gloves, and 3650 patients were treated with nonsterile gloves. Heterogeneity was low for the included studies. Of note, the researchers performed a subgroup analysis on 9 total studies (4 RCTs and 5 observational studies) involving cutaneous surgeries only. These represented procedures most likely performed in the primary care setting.

 

The primary outcome of this review was postoperative wound infection. The results did not show any difference in SSIs between sterile vs nonsterile gloves in all trials (2% vs 2.1%; relative risk [RR]=1.06; 95% confidence interval [CI], 0.81-1.39). There was also no difference in infection rates in the subgroup analysis of 9 trials limited to cutaneous surgery (2.2% vs 2.2%, respectively; RR=1.02; 95% CI, 0.78-1.34) or when the analysis was limited to only RCTs.

[polldaddy:10063798]

Continue to: WHAT'S NEW

 

 

WHAT’S NEW

Highest-quality evidence shows no difference in SSIs

This systematic review found no difference in SSI rates when using sterile vs nonsterile gloves. Given that the analysis represents the highest-quality level of evidence (a systematic review of RCTs) and that sterile gloves are several times more expensive per pair than nonsterile gloves, the findings should impact future practice.

CAVEATS

A risk of bias and limited applicability

Not every trial in this meta-analysis was an RCT, and the inclusion of observational studies increases the risk of bias. However, the results of the observational studies were similar to those of the RCTs, somewhat alleviating this potential threat to validity.

The results did not show any difference in surgical site infections between sterile and nonsterile gloves.

It is worth noting that more extensive surgeries and more complicated repairs were not included in the trials, meaning that the findings are limited to oral surgery, Mohs micrographic surgery, standard incisions, and laceration repairs.

 

CHALLENGES TO IMPLEMENTATION

Inertia, medicolegal concerns, and personal preference

Clinical inertia may lead to slow adoption of these recommendations. Physicians may worry about potential medicolegal ramifications from this change.1 Lastly, some physicians may prefer the fit and feel of sterile gloves for their procedures.

ACKNOWLEDGEMENT

The PURLs Surveillance System was supported in part by Grant Number UL1RR024999 from the National Center For Research Resources, a Clinical Translational Science Award to the University of Chicago. The content is solely the responsibility of the authors and does not necessarily represent the official views of the National Center For Research Resources or the National Institutes of Health.

ILLUSTRATIVE CASE

Your practice manager comes to you to discuss ways that you can reduce expenses. He asks whether the practice could reduce the amount of money spent on gloves for procedures. How do you reply?

A decision involving a small difference, spread over a larger number of events, can have a sizable effect. An example is whether to use sterile vs nonsterile gloves for minor procedures. The cost difference between a box of sterile gloves and a box of nonsterile gloves is relatively small, and certainly worth the difference if the more expensive sterile gloves reduce the number of surgical site infections (SSIs).

However, if there is no difference in the number of SSIs, there may be no value to the extra cost, which, given the number of such procedures, becomes a large unnecessary expense. The choice to use sterile gloves often stems from habit, product availability, or the perceived benefit of fewer SSIs.2 While some evidence exists comparing glove choice, there is wide variability in physicians’ choice of gloves.3-5 This large systematic review compared rates of SSIs using sterile vs nonsterile gloves.

STUDY SUMMARY

RCTs/observational studies find sterile no better than nonsterile gloves

This systematic review and meta-analysis of 13 randomized controlled trials (RCTs) and observational (prospective or retrospective) studies compared infection rates using sterile vs nonsterile gloves in 11,071 unique patients undergoing cutaneous surgery, including Mohs microsurgery or outpatient dental procedures. The methods used in the review followed the Cochrane collaboration guidelines.6 The inclusion criteria were that the studies had to be either RCTs or observational studies. Patients included in each study underwent outpatient cutaneous or mucosal surgical procedures, including laceration repair, standard excisions, Mohs micrographic surgery, or tooth extractions. In addition to glove type, documentation of postoperative SSI was necessary for inclusion.

Methodology. The authors of the analysis reviewed a total of 512 publications for inclusion; of these, 14 met the inclusion criteria. One study was later removed due to incomplete data, leaving a total of 13 trials for the analysis. Of the 11,071 patients included in the final analysis, 1360 patients were randomly assigned to treatment with sterile gloves, while 1381 patients were assigned to treatment with nonsterile gloves as the intervention in a clinical trial. The remaining patients participated in either prospective or retrospective observational trials; 4680 patients were treated with sterile gloves, and 3650 patients were treated with nonsterile gloves. Heterogeneity was low for the included studies. Of note, the researchers performed a subgroup analysis on 9 total studies (4 RCTs and 5 observational studies) involving cutaneous surgeries only. These represented procedures most likely performed in the primary care setting.

 

The primary outcome of this review was postoperative wound infection. The results did not show any difference in SSIs between sterile vs nonsterile gloves in all trials (2% vs 2.1%; relative risk [RR]=1.06; 95% confidence interval [CI], 0.81-1.39). There was also no difference in infection rates in the subgroup analysis of 9 trials limited to cutaneous surgery (2.2% vs 2.2%, respectively; RR=1.02; 95% CI, 0.78-1.34) or when the analysis was limited to only RCTs.

[polldaddy:10063798]

Continue to: WHAT'S NEW

 

 

WHAT’S NEW

Highest-quality evidence shows no difference in SSIs

This systematic review found no difference in SSI rates when using sterile vs nonsterile gloves. Given that the analysis represents the highest-quality level of evidence (a systematic review of RCTs) and that sterile gloves are several times more expensive per pair than nonsterile gloves, the findings should impact future practice.

CAVEATS

A risk of bias and limited applicability

Not every trial in this meta-analysis was an RCT, and the inclusion of observational studies increases the risk of bias. However, the results of the observational studies were similar to those of the RCTs, somewhat alleviating this potential threat to validity.

The results did not show any difference in surgical site infections between sterile and nonsterile gloves.

It is worth noting that more extensive surgeries and more complicated repairs were not included in the trials, meaning that the findings are limited to oral surgery, Mohs micrographic surgery, standard incisions, and laceration repairs.

 

CHALLENGES TO IMPLEMENTATION

Inertia, medicolegal concerns, and personal preference

Clinical inertia may lead to slow adoption of these recommendations. Physicians may worry about potential medicolegal ramifications from this change.1 Lastly, some physicians may prefer the fit and feel of sterile gloves for their procedures.

ACKNOWLEDGEMENT

The PURLs Surveillance System was supported in part by Grant Number UL1RR024999 from the National Center For Research Resources, a Clinical Translational Science Award to the University of Chicago. The content is solely the responsibility of the authors and does not necessarily represent the official views of the National Center For Research Resources or the National Institutes of Health.

References

1. Brewer JD, Gonzalez AB, Baum CL, et al. Comparison of sterile vs nonsterile gloves in cutaneous surgery and common outpatient dental procedures: a systematic review and meta-analysis. JAMA Dermatol. 2016;152:1008-1014.

2. Creamer J, Davis K, Rice W. Sterile gloves: do they make a difference? Am J Surg. 2012;204:976-979.

3. Heal C, Sriharan S, Buttner PG, et al. Comparing non-sterile to sterile gloves for minor surgery: a prospective randomised controlled non-inferiority trial. Med J Aust. 2015;202:27-31.

4. Ghafouri HB, Zoofaghari SJ, Kasnavieh MH, et al. A pilot study on the repair of contaminated traumatic wounds in the emergency department using sterile versus non-sterile gloves. Hong Kong J Emerg Med. 2014;21:148-152.

5. Rogers HD, Desciak EB, Marcus RP, et al. Prospective study of wound infections in Mohs micrographic surgery using clean surgical technique in the absence of prophylactic antibiotics. J Am Acad Dermatol. 2010;63:842-851.

6. Cochrane Methods. London, UK: The Cochrane Collaboration. 2018. Available at: http://methods.cochrane.org/. Accessed July 15, 2018.

References

1. Brewer JD, Gonzalez AB, Baum CL, et al. Comparison of sterile vs nonsterile gloves in cutaneous surgery and common outpatient dental procedures: a systematic review and meta-analysis. JAMA Dermatol. 2016;152:1008-1014.

2. Creamer J, Davis K, Rice W. Sterile gloves: do they make a difference? Am J Surg. 2012;204:976-979.

3. Heal C, Sriharan S, Buttner PG, et al. Comparing non-sterile to sterile gloves for minor surgery: a prospective randomised controlled non-inferiority trial. Med J Aust. 2015;202:27-31.

4. Ghafouri HB, Zoofaghari SJ, Kasnavieh MH, et al. A pilot study on the repair of contaminated traumatic wounds in the emergency department using sterile versus non-sterile gloves. Hong Kong J Emerg Med. 2014;21:148-152.

5. Rogers HD, Desciak EB, Marcus RP, et al. Prospective study of wound infections in Mohs micrographic surgery using clean surgical technique in the absence of prophylactic antibiotics. J Am Acad Dermatol. 2010;63:842-851.

6. Cochrane Methods. London, UK: The Cochrane Collaboration. 2018. Available at: http://methods.cochrane.org/. Accessed July 15, 2018.

Issue
The Journal of Family Practice - 67(8)
Issue
The Journal of Family Practice - 67(8)
Page Number
507-508
Page Number
507-508
Publications
Publications
Topics
Article Type
Display Headline
Time to switch to nonsterile gloves for these procedures?
Display Headline
Time to switch to nonsterile gloves for these procedures?
Sections
PURLs Copyright
Copyright © 2018. The Family Physicians Inquiries Network. All rights reserved.
Inside the Article

PRACTICE CHANGER

Using nonsterile gloves for common primary care skin procedures causes no more infections than using sterile gloves.1

STRENGTH OF RECOMMENDATION

A: Based on a systematic review and meta-analysis of 13 randomized controlled trials.

Brewer JD, Gonzalez AB, Baum CL, et al. Comparison of sterile vs nonsterile gloves in cutaneous surgery and common outpatient dental procedures: a systematic review and meta-analysis. JAMA Dermatol. 2016;152:1008-1014.

Disallow All Ads
Content Gating
No Gating (article Unlocked/Free)
Alternative CME
PubMed ID
30110498
Disqus Comments
Default
Use ProPublica
Article PDF Media
Media Files